You are on page 1of 209

More Advance Praise for

Neuro-Ophthalmology

“This superbly written and illustrated book provides the reader with the three most important
aspects of medical knowledge in general and neuro-ophthalmology in particular: a differential diag-
nosis of specific symptoms and signs, the pathophysiology of specific disorders, and the appropriate
management for disorders for which a diagnosis has been made. This is the way teaching should be.
The book is, quite simply, a real winner, and those who read it will be winners for themselves and
their patients!”
—Neil R. Miller, MD, Frank B. Walsh Professor of Neuro-Ophthalmology,
Johns Hopkins Medical Institutions, Baltimore, MD

“This is a concise, straightforward, and well written book in which the authors use a novel approach,
case histories, to cover a wide spectrum of Neuro-Ophthalmological topics. The book is divided into
five sections dealing with disorders of the afferent visual system: eye movements including nystag-
mus, pupils, eyelids, and complex ‘combination’ syndromes involving the orbital apex, superior
orbital fissure, and cavernous sinus.
“Each case presentation is followed by a ‘What do you do now?’ discussion that includes obtain-
ing further pertinent history and examination, a discussion of the pathogenesis, the differential
diagnosis and then management. The key points of each condition are highlighted. There are 27
illustrations including patient and fundus photographs, visual fields, imaging studies, and dia-
grams, and 12 tables, that complement the case histories. Each topic is followed by a brief list of
references for further reading.
“This enjoyable work is ideal for students and residents rotating through neuro-ophthalmology, neuro-
ophthalmology fellows, and practicing neurologists, ophthalmologists, and neuro-ophthalmologists.”
—Patrick Lavin, MB, BCh, BAO, MRCPI, Professor of Neurology, Professor of Ophthalmology
and Visual Sciences, Vanderbilt University School of Medicine, Nashville, TN

“It is a pleasure to write this review of the welcome text, Neuro-Ophthalmology, authored by
Thurtell, Tomsak, and Daroff. The authors bring a wealth of experience to this project and are to
be commended for producing a superb text.
“The table of contents is complete and easy to use with sections based on diagnosis; the topics
include not only the common, but indeed the vast majority of diagnoses likely to be encountered
in N-O practice.
“The text is well written and a pleasure to read. The recommendations are sage and sound,
embodied within a readable, case-based text. The encapsulated cases with highlighted key points are
a strong point. This text can be recommended strongly with ease.
“The tables are well thought-out and useful, efficiently covering such difficult topics as higher
cortical dysfunction-related terminology. The illustrations are excellent, with high-resolution detail
that adds immeasurably to the cases.
“This will be a welcome and useful addition to the library of not only the neuro-ophthalmic neo-
phyte, but also the seasoned veteran looking for an enjoyable refresher in a readable, well-illustrated
package.”
—Eric Eggenberger, DO, MSEpi, FAAN, Professor and Vice-Chair,
Department of Neurology and Ophthalmology, Michigan State University, East Lansing, MI
What Do I Do Now?
S ER IES CO -E D I TORS-I N-CHIEF

Lawrence C. Newman, MD
Director of the Headache Institute
Department of Neurology
St. Luke’s-Roosevelt Hospital Center
New York, NY

Morris Levin, MD
Co-director of the Dartmouth Headache Center
Director of the Dartmouth Neurology Residency Training Program
Section of Neurology
Dartmouth Hitchcock Medical Center
Lebanon, NH

PR EV IO US VO LUM ES I N TH E S ER IES

Headache and Facial Pain


Peripheral Nerve and Muscle Disease
Pediatric Neurology
Stroke
Epilepsy
Neurocritical Care

ii WHAT DO I DO NOW? PERIPHERAL NERVE AND MUSCLE DISEASE


Neuro-Ophthalmology

Matthew J. Thurtell, MBBS, FRACP


Assistant Professor of Ophthalmology and Neurology
Department of Ophthalmology and Visual Sciences
University of Iowa
Iowa City, IA

Robert L. Tomsak, MD, PhD


Professor of Ophthalmology and Neurology
Kresge Eye Institute
Wayne State University
Detroit, MI

Robert B. Daroff, MD
Professor of Neurology
Associate Dean for Development
School of Medicine
Case Western Reserve University
Cleveland, OH

1
1
Oxford University Press, Inc., publishes works that further Oxford University’s objective of excellence
in research, scholarship, and education.

Oxford New York


Auckland Cape Town Dar es Salaam Hong Kong Karachi Kuala Lumpur Madrid
Melbourne Mexico City Nairobi New Delhi Shanghai Taipei Toronto

With offices in
Argentina Austria Brazil Chile Czech Republic France Greece Guatemala Hungary Italy
Japan Poland Portugal Singapore South Korea Switzerland Thailand Turkey Ukraine Vietnam

Copyright © 2012 by Oxford University Press, Inc.

Published by Oxford University Press, Inc.


198 Madison Avenue, New York, New York 10016
www.oup.com

First issued as an Oxford University Press paperback, 2012

Oxford is a registered trademark of Oxford University Press

All rights reserved. No part of this publication may be reproduced, stored in a retrieval system, or
transmitted, in any form or by any means, electronic, mechanical, photocopying, recording, or otherwise,
without the prior permission of Oxford University Press.
____________________________________________
Library of Congress Cataloging-in-Publication Data
Thurtell, Matthew J.
Neuro-ophthalmology / Matthew J. Thurtell, Robert L. Tomsak, Robert B. Daroff.
p. ; cm. — (What do I do now?)
Includes bibliographical references and index.
ISBN 978-0-19-539084-1 (pbk) 1. Neuroophthalmology—Case studies. I. Tomsak, Robert L.
II. Daroff, Robert B. III. Title. IV. Series: What do I do now?
[DNLM: 1. Eye Diseases—diagnosis—Case Reports. 2. Nervous System Diseases—complications—Case
Reports. 3. Cranial Nerve Diseases—Case Reports. 4. Diagnosis, Differential—Case Reports.
5. Eye Diseases—therapy—Case Reports. WW 460]
RE725.T48 2012
617.7—dc23 2011016787
____________________________________________

The science of medicine is a rapidly changing field. As new research and clinical experience broaden our
knowledge, changes in treatment and drug therapy occur. The author and publisher of this work have checked
with sources believed to be reliable in their efforts to provide information that is accurate and complete, and in
accordance with the standards accepted at the time of publication. However, in light of the possibility of human
error or changes in the practice of medicine, neither the author, nor the publisher, nor any other party who has
been involved in the preparation or publication of this work warrants that the information contained herein is
in every respect accurate or complete. Readers are encouraged to confirm the information contained herein with
other reliable sources, and are strongly advised to check the product information sheet provided by the
pharmaceutical company for each drug they plan to administer.

987654321

Printed in the United States of America on acid-free paper

iv WHAT DO I DO NOW? PERIPHERAL NERVE AND MUSCLE DISEASE


This book is dedicated to:

My mentors: R. John Leigh, Robert L. Tomsak, Nancy J. Newman,


Valérie Biousse, and G. Michael Halmagyi.
—MJT
J. Lawton Smith, MD: a superb teacher, remarkable man, and brilliant
neuro-ophthalmologist.
—RLT and RBD
This page intentionally left blank
Preface

Patients with neuro-ophthalmic conditions are commonly encountered in


clinical practice, yet many clinicians feel ill-prepared or uncomfortable when
dealing with them. Those trained in neurology often feel uneasy when evalu-
ating patients who have predominantly visual or ocular complaints, whereas
those trained in ophthalmology often feel uncomfortable when evaluating
those with predominantly neurologic complaints. A neuro-ophthalmologist
is not always available for consultation, so that the clinician is left wonder-
ing, “What do I do now?” when encountering a challenging case.
In this installment of the What Do I Do Now? series, we aim to provide
a user-friendly manual that clinicians can use when dealing with neuro-
ophthalmic problems on the ward or in the clinic. The volume is divided
into five sections that cover the main aspects of neuro-ophthalmic practice:
(1) afferent (visual) disorders, (2) efferent (eye movement) disorders,
(3) eyelid disorders, (4) pupil disorders, and (5) combination syndromes.
Each chapter includes a practical, case-based discussion on how to approach
and manage a patient with a particular disorder. We discuss mainly com-
mon disorders, although we also consider some less common yet important
conditions, as well as neuro-ophthalmic presentations of systemic and psy-
chiatric disease. We have included a larger proportion of efferent (eye move-
ment) cases, because many other handbooks focus on afferent (visual)
disorders. We base our recommendations on current evidence whenever
possible. A list of key clinical points appears at the end of each chapter as
well as a list of important references. In most chapters, tables and boxes
summarize pertinent information. We include figures in many chapters to
illustrate abnormal clinical signs or relevant imaging findings.
We designed the volume as a resource for neurologists and ophthalmolo-
gists at all levels of training. We hope that it will serve as a useful handbook
in caring for patients with neuro-ophthalmic disorders.

MJT, Iowa City, IA


RLT, Detroit, MI
RBD, Cleveland, OH

vii
This page intentionally left blank
Acknowledgments

Many of the cases described in this book touch on controversial aspects of


neuro-ophthalmology. We acknowledge those experts in the field who
agreed to discuss their personal approaches to such cases with us. In par-
ticular, we thank Drs. R. John Leigh, Michael Wall, Randy H. Kardon,
Christopher A. Johnson, and Mark Wilkinson for their useful advice. We
also thank the team at Oxford University Press, Craig Panner and Kathryn
Winder, for their help in bringing this volume to completion.

Matthew J. Thurtell, MBBS, FRACP


Robert L. Tomsak, MD, PhD
Robert B. Daroff, MD

ix
This page intentionally left blank
Contents

SECTION I AFFERENT DISORDERS

1 Optic Neuritis 5
Optic neuritis is the most frequent cause of optic neuropathy in young adults and
is often encountered in clinical practice. In this chapter, we summarize the
cardinal signs of optic neuropathy, and discuss the diagnostic evaluation and
management of idiopathic optic neuritis.

2 Arteritic Anterior Ischemic Optic Neuropathy 11


Arteritic anterior ischemic optic neuropathy occurs in the setting of giant cell
arteritis and is a medical emergency, because there is a high risk of fellow eye
involvement if corticosteroid treatment is not initiated in a timely fashion. In this
chapter, we review the clinical features of arteritic anterior ischemic optic
neuropathy. We also discuss the evaluation and treatment of patients with
suspected giant cell arteritis.

3 Nonarteritic Anterior Ischemic Optic Neuropathy 16


Nonarteritic anterior ischemic optic neuropathy is the most frequent cause
of optic neuropathy in older adults. Its pathogenesis remains uncertain,
although it often occurs in patients with small, structurally congested optic
discs. We summarize the symptoms, signs, possible precipitating factors, and
management of this common optic neuropathy.

4 Compressive Optic Neuropathy 21


Optic nerve compression results in progressive, and often painless, monocular
vision loss. In this chapter, we review the clinical signs and common causes of
compressive optic neuropathy. We discuss in more detail the imaging
characteristics and management of optic nerve sheath meningioma.

5 Hereditary Optic Neuropathy 25


Monocular and binocular vision loss can occasionally be caused by hereditary
optic neuropathy. While progressive painless binocular central vision loss is
characteristic of dominant optic atrophy, acute painless monocular vision loss is
characteristic of Leber’s hereditary optic neuropathy. We discuss the clinical
features and evaluation of Leber’s hereditary optic neuropathy and briefly
mention promising treatment options.

xi
6 Idiopathic Intracranial Hypertension 29
Idiopathic intracranial hypertension is a syndrome of raised intracranial pressure
of unknown cause that most often occurs in obese young women. Bilateral
papilledema is usually present and can cause severe, irreversible vision loss if left
untreated. In this chapter, we review the symptoms, signs, evaluation, and
management of idiopathic intracranial hypertension.

7 Pseudopapilledema 36
A diagnostic dilemma often arises when a patient with headaches is found to have
optic nerve head elevation. Anomalous optic nerve head elevation often mimics
papilledema and is therefore known as pseudopapilledema. In this chapter, we
review the features that help to distinguish pseudopapilledema from papilledema
and we discuss common causes of pseudopapilledema, such as optic nerve head
drusen.

8 Chiasmal Syndromes 43
Dysfunction of the optic chiasm typically produces bitemporal visual field
defects. Chiasmal dysfunction most frequently results from compression by
extrinsic lesions, such as pituitary macroadenomas and suprasellar meningiomas.
We describe the clinical signs of chiasmal dysfunction in this chapter. We also
discuss the evaluation and management of pituitary apoplexy.

9 Homonymous Hemianopia 48
Homonymous hemianopia is caused by lesions involving the retrochiasmal visual
pathways or primary visual cortex. The most common cause of homonymous
hemianopia is stroke. We discuss the approach to the patient with homonymous
hemianopia, with specific reference to prognosis, implications for driving, and
rehabilitation.

10 Disorders of Higher Visual Function 52


A disorder of higher visual function should be considered when visual complaints
are out of proportion to examination findings. Such disorders can remain
undiagnosed until other cognitive deficits develop. In this chapter, we review
common disorders of higher visual function, with specific reference to the visual
(posterior) variant of Alzheimer’s disease.

11 Transient Visual Loss 57


Transient visual loss is common and often due to a transient loss of blood supply
to the afferent visual system, although there are many other potential causes. We
review the approach to the patient with transient visual loss in this chapter, with
special attention to vascular causes.

xii CONTENTS
12 Migraine Aura 63
Migraine aura is a common cause of transient positive and negative visual
phenomena. Similar symptoms can occasionally occur with occipital lesions or
as a manifestation of occipital seizures. In this chapter, we review the clinical
features of migraine aura, with specific reference to those features that help
distinguish it from occipital seizures.

13 Nonorganic Vision Loss 68


Nonorganic vision loss is common, but can be challenging to diagnose and treat.
We discuss the approach to the patient with suspected nonorganic vision loss in
this chapter, and we describe the maneuvers that can be used to demonstrate
intact visual function.

SECTION II EFFERENT DISORDERS

14 Third Nerve Palsy 75


Acute painful pupil-involving third nerve palsy requires urgent investigation,
because it can be due to third nerve compression by a rapidly enlarging aneurysm
of the posterior communicating artery. We summarize the approach to the patient
with a third nerve palsy in this chapter, with emphasis on the imaging evaluation
of acute pupil-involving third nerve palsy.

15 Fourth Nerve Palsy 79


Fourth nerve palsy is a common cause of binocular vertical diplopia but can be
difficult to diagnose, because examination findings are often subtle. In this
chapter, we review the symptoms, signs, causes, differential diagnosis, and
evaluation of fourth nerve palsy.

16 Sixth Nerve Palsy 84


Binocular horizontal diplopia is often due to sixth nerve palsy but can be caused
by other conditions, such as restriction of the medial rectus muscle in thyroid eye
disease. We review the approach to the patient with sixth nerve palsy in this
chapter. We briefly discuss the role of imaging in patients with sixth nerve palsy,
as this remains a controversial topic.

17 Ocular Myasthenia 88
Ocular myasthenia typically causes intermittent or fluctuating ptosis and diplopia
but is often difficult to diagnose definitively. We discuss the approach to the
patient with suspected ocular myasthenia but no acetylcholine-receptor
antibodies, and review the treatment options for ocular myasthenia.

CONTENTS xiii
18 Complete Bilateral External Ophthalmoplegia 93
Complete bilateral external ophthalmoplegia is characterized by global weakness
of the extraocular and levator muscles. It has a broad differential diagnosis, which
varies depending on the tempo of onset. In this chapter, we review the approach
to the patient with complete bilateral external ophthalmoplegia and discuss the
common causes.

19 Superior Oblique Myokymia 98


Monocular oscillopsia is an uncommon symptom that is often due to superior
oblique myokymia. The diagnosis is frequently missed, because the eye
oscillations in this condition are intermittent and often subtle. We discuss the
evaluation and management of patients with suspected superior oblique
myokymia in this chapter.

20 Internuclear Ophthalmoplegia 102


Internuclear ophthalmoplegia is a classic ocular motor syndrome that is caused by
a lesion affecting the medial longitudinal fasciculus in the brainstem. In this
chapter, we review the signs, causes, and differential diagnosis of internuclear
ophthalmoplegia.

21 Progressive Supranuclear Palsy 106


Patients with progressive supranuclear palsy, a sporadic akinetic-rigid syndrome,
often have visual complaints and abnormal eye movements. We review the
neuro-ophthalmic signs of progressive supranuclear palsy and present a practical
approach to the management of its common visual symptoms, such as reading
difficulty.

22 Gaze-Evoked Nystagmus 110


Gaze-evoked nystagmus is the most common type of nystagmus encountered in
clinical practice but is poorly localizing. It is often confused with “end-point”
nystagmus, which is physiologic and of no concern. In this chapter, we discuss
the approach to the patient with gaze-evoked nystagmus.

23 Downbeat Nystagmus 114


Downbeat nystagmus is a common type of central vestibular nystagmus that
often produces oscillopsia or blurred vision. We review the signs, causes, and
investigation of downbeat nystagmus. We also discuss available treatment options.

24 Upbeat Nystagmus 118


Upbeat nystagmus is a less common type of central vestibular nystagmus that is
often transient. However, it can be persistent and responsible for visual
symptoms. We review the characteristics and causes of upbeat nystagmus in this
chapter. We also discuss the management of upbeat nystagmus occurring in the
setting of Wernicke’s encephalopathy.

xiv CONTENTS
25 Acquired Pendular Nystagmus 122
Acquired pendular nystagmus often occurs in the setting of multiple sclerosis
but can also occur in the syndrome of oculopalatal tremor. Because it often causes
disabling oscillopsia, many affected patients request treatment. In this chapter,
we discuss the clinical features and treatment of acquired pendular nystagmus.

26 Infantile Nystagmus Syndrome 126


Formerly known as congenital nystagmus, this form of nystagmus can occur in
isolation or in association with other ophthalmic, neurologic, or endocrine
abnormalities. While it does not usually cause oscillopsia, it can cause blurred
vision. Consequently, affected patients sometimes request treatment. We review
the clinical features of infantile nystagmus syndrome and present a contemporary
approach to treatment.

27 Saccadic Intrusions and Dysmetria 131


Saccadic intrusions and dysmetria are often encountered in association with
certain cerebellar and neurodegenerative diseases. When severe, they can give rise
to difficulty reading or oscillopsia. In this chapter, we review the approach to the
patient with saccadic intrusions and dysmetria. We also briefly discuss proposed
medical treatments.

SECTION III EYELID DISORDERS

28 Eyelid Ptosis 137


Upper-eyelid ptosis is frequently encountered in clinical practice but has a broad
differential diagnosis. We discuss the approach to the patient with upper-eyelid
ptosis in this chapter, with special attention to levator dehiscence.

29 Benign Essential Blepharospasm 141


Blepharospasm is an involuntary closure of the eyes that is caused by spasm of the
orbicularis oculi. It can be isolated or associated with certain ophthalmic and
neurologic disorders. We review the approach to the patient with blepharospasm
in this chapter, with emphasis on treatment strategies.

SECTION IV PUPIL DISORDERS

30 Physiologic Anisocoria 147


A difference in the size of the pupils (anisocoria) is a frequent finding on physical
examination. While often a cause for alarm, it is not uncommonly physiologic
and of no concern. In this chapter, we summarize the approach to the patient
with anisocoria.

CONTENTS xv
31 Horner’s Syndrome 152
Horner’s syndrome can be caused by a lesion anywhere along the oculosympathetic
pathway. Although there may be other signs that might help with localization of
the lesion, the syndrome often occurs in isolation. We review the approach to the
patient with suspected Horner’s syndrome, with special attention to the role of
investigations such as pharmacologic pupil testing and imaging.

32 Tonic Pupil 158


A tonic pupil is caused by a lesion affecting the postganglionic parasympathetic
innervation of the pupil. It can be an incidental finding on examination or
associated with visual symptoms, such as photophobia or blurred vision at near.
We discuss the common causes and diagnostic evaluation of a tonic pupil in this
chapter.

33 Pharmacologic Mydriasis 162


A dilated unreactive pupil is a dramatic clinical finding. While it can be seen in
the setting of compressive third nerve palsy, it can also be due to pharmacologic
mydriasis. We discuss the approach to the patient with suspected pharmacologic
mydriasis, with an emphasis on the role of pharmacologic pupil testing.

S ECTI O N V CO MB I N ATION SY N DROMES

34 Syndromes of the Orbital Apex, Superior Orbital Fissure,


and Cavernous Sinus 169
Lesions in the orbital apex, superior orbital fissure, and cavernous sinus can give
rise to characteristic combinations of cranial nerve palsies. In this chapter, we
briefly review the possible causes of these syndromes. We discuss rhino-orbital
mucormycosis in detail, because it has a grave prognosis if it is not diagnosed and
treated in a timely fashion.

35 Dorsal Midbrain Syndrome 175


Lesions of the dorsal midbrain give rise to a characteristic and highly localizing
constellation of neuro-ophthalmic signs. We review the clinical features and
common causes of the dorsal midbrain syndrome in this chapter, with special
reference to the clinical features and management of ventriculoperitoneal shunt
malfunction.

36 Thyroid Eye Disease 178


Thyroid eye disease is the most common cause of orbital disease encountered in
clinical practice. It often occurs in patients with Graves’ disease, but it is not
always associated with abnormal thyroid function. In this chapter, we review the
clinical signs, investigation, and treatment of thyroid eye disease.

Index 183

xvi CONTENTS
Neuro-Ophthalmology
This page intentionally left blank
SECTION I

AFFERENT DISORDERS
This page intentionally left blank
1 Optic Neuritis

You are called to see a 22-year-old woman who has had a


subacute onset of vision loss in her right eye over several
days with associated pain on eye movements. She has
no past medical history and denies other neurologic
symptoms. Examination reveals visual acuities of 20/100
in the right eye and 20/15 in the left eye. She can identify
only the control Ishihara color plate with the right eye,
but she correctly identifies all plates with the left eye.
Confrontation visual fields reveal a central scotoma in
the right eye. Her pupils are equal, but there is a right
relative afferent pupillary defect. Her funduscopic
examination is normal.

What do you do now?

5
T he patient in this scenario has had a subacute onset of painful monocu-
lar vision loss and exhibits the cardinal signs of an optic neuropathy:
decreased visual acuity, a central visual field defect, dyschromatopsia, and a
relative afferent pupillary defect. Her presentation is classic for optic neuri-
tis. The clinical manifestations of optic neuritis vary depending on the por-
tion of the optic nerve that is inflamed. In retrobulbar optic neuritis, the
most common subtype of optic neuritis, there is minimal, if any, optic disc
edema. In papillitis, the optic nerve head itself is inflamed and there is optic
disc edema. In neuroretinitis, the peripapillary retina is also inflamed, giving
rise to changes in the macula (e.g., a macular star), as well as optic disc
edema.
Causes for optic neuritis vary depending on the portion of the optic
nerve affected. Causes for retrobulbar optic neuritis include demyelinating
diseases (e.g., multiple sclerosis and neuromyelitis optica), autoimmune
diseases (e.g., systemic lupus erythematosus), inflammatory diseases (e.g.,
sarcoidosis), infections (e.g., Lyme disease and syphilis), and vaccinations
(e.g., influenza vaccine). In many patients, however, a cause cannot be
identified and the optic neuritis is considered idiopathic. Nevertheless,
the first step in the evaluation of this patient would be to obtain further
history (e.g., inquiring about recent travel, vaccinations, and tick bites).
The ophthalmic and neurologic examinations should be completed, because
there may be abnormalities that suggest the diagnosis. For instance, the
presence of granulomatous uveitis on ophthalmic examination might sug-
gest an inflammatory disorder, such as sarcoidosis, whereas the presence
of nystagmus or internuclear ophthalmoplegia might suggest multiple scle-
rosis (MS). In the absence of any other significant history or abnormal
examination findings, however, the optic neuritis should be considered
idiopathic.
Diagnostic studies should be obtained in patients with idiopathic optic
neuritis to confirm the presence of optic nerve inflammation, to look for
central nervous system lesions, and to exclude other causes for the optic
neuropathy (e.g., compressive optic neuropathy; see case 4). The most
appropriate initial diagnostic study is magnetic resonance imaging (MRI)
of the orbits and brain. MRI of the orbits typically demonstrates increased
signal in the affected optic nerve, with associated contrast enhancement

6 WHAT DO I DO NOW? NEURO-OPHTHALMOLOGY


FIGURE 1-1 Magnetic resonance imaging (MRI) of the orbits (top row) and brain (bottom
row) in a patient with optic neuritis, demonstrating left optic nerve enhancement and
multiple ovoid periventricular white matter lesions.

that is best appreciated on fat-suppressed images (see Figure 1-1). However,


such changes are nonspecific and cannot be considered diagnostic of
idiopathic optic neuritis. MRI of the brain should be obtained to assess for
periventricular white matter lesions, which are most obvious on T2-weighted
imaging and FLAIR (fluid-attenuated inversion recovery) sequences (see
Figure 1-1), because the presence of one or more lesions (especially in the
corpus callosum) portends an increased risk of developing MS.
Laboratory investigations can be used to screen for other causes of optic
neuritis. However, these investigations, which include neuromyelitis optica
(aquaporin-4) antibody, antinuclear antibody, angiotensin-converting
enzyme level, syphilis serology, and Lyme disease serology, are often unreveal-
ing or can give misleading false-positive results in patients with idiopathic

1. OPTIC NEURITIS 7
optic neuritis. Thus, further laboratory investigations should not be obtained
routinely in every patient but should be tailored to the individual patient.
Cerebrospinal fluid (CSF) analysis can be useful to exclude other causes of
optic neuritis in selected patients. However, it is not routinely required to
look for oligoclonal bands or other CSF markers of MS, because an increased
risk of MS is more reliably predicted by the presence of white matter lesions
on MRI.
The recommended treatment of idiopathic optic neuritis is based on the
findings of the Optic Neuritis Treatment Trial (Beck et al., 1992, 1993),
which was a randomized controlled treatment trial comparing outcomes in
patients who received intravenous and then oral steroids, oral steroids alone,
or placebo. The group receiving intravenous and then oral steroids showed
a more rapid recovery of vision than the placebo group, although the final
visual outcome was similar. The former group also showed a lower risk of
developing clinically definite MS in the first 2 years following treatment.
The group receiving oral steroids alone did not show a faster recovery com-
pared with placebo, but did show an increased rate of recurrent optic neu-
ritis attacks compared with the other two groups. Thus, the recommended
treatment for idiopathic optic neuritis is intravenous methylprednisone
(1 g daily) for 3 days followed by oral prednisone (1 mg/kg per day) for
11 days. The prognosis for visual recovery is excellent, with vision recovering
to near normal in most patients over weeks to months, although there can
be persisting minor visual deficits (e.g., in contrast and color vision) and
clinical signs of optic nerve dysfunction (e.g., a relative afferent pupillary
defect).
In patients who have one or more white matter lesions on MRI, the risk
of developing MS is greater than 50% in the 10 years following an attack of
idiopathic optic neuritis, compared with about 20% in those patients who
do not have white matter lesions. Treatment with disease-modifying therapy
for MS (e.g., beta-interferon) can reduce the risk of developing MS in
patients with idiopathic optic neuritis who have white matter lesions on
MRI (Jacobs et al., 2000). Although not all optic neuritis patients with
white matter lesions will develop clinically definite MS, initiation of dis-
ease-modifying therapy should be carefully considered.

8 WHAT DO I DO NOW? NEURO-OPHTHALMOLOGY


K EY P O I N TS TO R E M E M B E R

■ The cardinal signs of an optic neuropathy are decreased visual


acuity, a central visual field defect, dyschromatopsia, and a relative
afferent pupillary defect.
■ Optic neuritis is characterized by a subacute onset of painful
monocular vision loss with a relative afferent pupillary defect; optic
disc edema may or may not be present, depending on the portion of
the nerve affected.
■ Optic neuritis is often idiopathic, but can be caused by
demyelinating disease, autoimmune disease, inflammatory disease,
infections, and vaccinations.
■ MRI of the orbits and brain should be obtained to confirm the
presence of optic nerve inflammation, assess for white matter
lesions, and exclude other causes of optic neuropathy.
■ The 10-year risk of MS is over 50% in optic neuritis patients with
one or more white matter lesions on MRI versus about 20% in
those with no white matter lesions.
■ Treatment of optic neuritis with intravenous methylprednisone
(1 g daily) for 3 days and then oral prednisone (1 mg/kg per day) for
11 days leads to a faster recovery of vision and decreased risk of
developing MS in the 2 years following treatment.

Further Reading
Beck RW, Cleary PA, Anderson MM Jr, et al. A randomized, controlled trial of
corticosteroids in the treatment of acute optic neuritis. N Engl J Med. 1992;326:
581–588.
Beck RW, Cleary PA, Trobe JD, et al. The effect of corticosteroids for acute optic neuritis
on the subsequent development of multiple sclerosis. N Engl J Med. 1993;329:
1764–1769.
Beck RW, Trobe JD, Moke PS, et al. High- and low-risk profiles for the development of
multiple sclerosis within 10 years after optic neuritis: experience of the Optic
Neuritis Treatment Trial. Arch Ophthalmol. 2003;121:944–949.
Biousse V, Calvetti O, Drews-Botsch CD, Atkins EJ, Sathornsumetee B, Newman NJ.
Management of optic neuritis and impact of clinical trials: an international survey.
J Neurol Sci. 2009;276:69–74.

1. OPTIC NEURITIS 9
Jacobs LD, Beck RW, Simon JH, et al. Intramuscular interferon beta-1a therapy initiated
during a first demyelinating event in multiple sclerosis. N Engl J Med.
2000;343:898–904.
Jafari N, Kreft KL, Flach HZ, Janssens AC, Hintzen RQ. Callosal lesion predicts future
attacks after clinically isolated syndrome. Neurology. 2009;73:1837–1841.
Polman CH, Reingold SC, Banwell B, et al. Diagnostic criteria for multiple sclerosis: 2010
revisions to the McDonald criteria. Ann Neurol. 2011;69:292–302.

10 WHAT DO I DO NOW? NEURO-OPHTHALMOLOGY


2 Arteritic Anterior Ischemic
Optic Neuropathy

A 75-year-old white woman presents to your clinic after


suddenly developing vision loss in her right eye. She
reports having had several brief episodes of transient
vision loss in her right eye over the past week, as well as
a persistent dull right-sided temporal headache.
Examination reveals visual acuities of count fingers in
the right eye and 20/25 in the left eye. She cannot
identify the control Ishihara color plate with the right
eye, but she correctly identifies all plates with the left
eye. Her pupils are equal, but there is a right relative
afferent pupillary defect. Funduscopic examination
reveals pallid optic disc edema in the right eye.

What do you do now?

11
T he acute onset of severe monocular vision loss with associated optic
disc edema in this older woman with temporal headaches should
immediately suggest anterior ischemic optic neuropathy due to giant cell
arteritis (GCA). Ischemic optic neuropathies occur as a result of hypoperfu-
sion of the optic nerve. Ischemia to the retrobulbar portion of the nerve
results in posterior ischemic optic neuropathy; because the anterior portion
of the optic nerve is not affected, there is no associated optic disc edema.
Ischemia to the anterior portion of the nerve results in anterior ischemic
optic neuropathy; because the optic nerve head is affected, there is, by defi-
nition, optic disc edema, which can be hyperemic or pallid. Anterior isch-
emic optic neuropathy is common compared to posterior ischemic optic
neuropathy. It can be divided into two types: arteritic anterior ischemic
optic neuropathy (AAION), in which the ischemia results from inflamma-
tory occlusion of the posterior ciliary arteries by vasculitis (typically GCA),
and nonarteritic anterior ischemic optic neuropathy (NAION), in which
the ischemia occurs because of other factors (see case 3).
GCA is a rare granulomatous vasculitis that affects medium- to large-
sized arteries, especially the cranial branches of the aortic arch. It occurs
most commonly in white women who are aged 65 years or more and does
not occur in children or adults who are aged 50 years or less. Up to 50% of
patients present with visual symptoms, mostly secondary to AAION. The
AAION in GCA is characterized by a rapid onset of severe monocular
vision loss with diffuse optic disc edema (see Figure 2-1) and a dense relative
afferent pupillary defect. The vision loss is often devastating, with the initial
visual acuity being count fingers or worse in over 50% of patients. The optic
disc edema is typically pallid but can be hyperemic, and it gradually resolves
over several weeks, with the optic disc ultimately becoming pale, atrophic,
and cupped. In contrast with optic neuritis and NAION (see cases 1 and 3),
there is rarely any recovery of vision.
AAION can sometimes be difficult to distinguish from NAION in the
acute setting, but the distinction is clinically important, because 25%–50%
of patients with AAION will go on to develop AAION in the fellow eye
within 2 weeks if left untreated. The presence of pallid optic disc edema is
highly suggestive of AAION, whereas hyperemic optic disc edema with
hemorrhages in the retinal nerve fiber layer is more characteristic of NAION
(see case 3). Since GCA can cause inflammatory occlusion of the posterior

12 WHAT DO I DO NOW? NEURO-OPHTHALMOLOGY


FIGURE 2-1 Fundus photograph demonstrating optic disc edema due to arteritic anterior
ischemic optic neuropathy, with cilioretinal artery occlusion, in a patient with giant cell
arteritis (left). Fluorescein angiography shows patchy choroidal nonperfusion and cilioretinal
artery occlusion (right).

ciliary arteries (which supply the choroid of the eye) and the cilioretinal
artery (which variably supplies the papillomacular portion of the retina),
concurrent choroidal or cilioretinal ischemia is highly suggestive of GCA
(see Figure 2-1). Prior to the onset of AAION, a substantial proportion of
patients will have episodes of transient monocular or binocular vision loss,
which are typically precipitated by postural changes. Some patients experi-
ence transient diplopia, which is thought to be due to ischemia of the
extraocular muscles. Many report systemic symptoms, such as temporal
headache, jaw claudication, scalp tenderness, malaise, weight loss, and fever,
which should immediately suggest GCA. However, their absence does not
preclude the diagnosis, because over 20% of patients with biopsy-proven
GCA do not have systemic symptoms.
In our patient with a dull temporal headache and severe monocular
vision loss due to anterior ischemic optic neuropathy, the clinical suspicion
for GCA is high. Therefore, urgent investigations and treatment are required.
An erythrocyte sedimentation rate (ESR), C-reactive protein (CRP), and
platelet count should be obtained immediately. Most patients with GCA
have elevation of all inflammatory markers, reflecting the systemic inflam-
mation, but occasionally only one might be elevated. In patients who do
not have elevated inflammatory markers, further investigations should still
be obtained and empiric treatment initiated if the clinical suspicion for

2. ARTERITIC ANTERIOR ISCHEMIC OPTIC NEUROPATHY 13


GCA is high. Fluorescein angiography can be useful to demonstrate choroi-
dal or cilioretinal ischemia, thereby helping to differentiate AAION from
NAION (see Figure 2-1). However, the diagnosis can be confirmed only by
finding the characteristic histopathologic changes on temporal artery biopsy.
The specimen should be at least 2 cm long and serially sectioned, to avoid a
false-negative result in the event that there are skip lesions. If the clinical
suspicion is low, a negative biopsy is sufficient to exclude the diagnosis. The
initiation of treatment should not be delayed while awaiting a temporal
artery biopsy or its result; the histopathologic changes persist for at least
several weeks after treatment is commenced.
In any patient with suspected AAION, systemic corticosteroids should
be administered immediately to reduce the risk of AAION occurring in
the fellow eye. Although no prospective randomized study has been per-
formed, many clinicians treat with intravenous methylprednisone (1 g/day)
for 3 days followed by high-dose oral prednisone (1 mg/kg per day),
although some will begin with high-dose oral prednisone (1–2 mg/kg per
day). High-dose oral prednisone should be continued for at least a month,
until systemic symptoms have resolved and the inflammatory markers have
normalized. Thereafter, the dose should be slowly tapered over 12–18
months, so long as the inflammatory markers remain within normal limits.
The patient should be carefully monitored, because the disease can flare up
as the steroid dose is tapered. It is essential to involve a rheumatologist or
primary care physician in the patient’s management, because many patients
are at risk of developing significant side effects, such as osteoporosis and
steroid-induced diabetes.

K EY P O I N TS TO R E M E M B E R

■ AAION is characterized by a rapid onset of severe monocular vision


loss with pallid or hyperemic optic disc edema and a relative
afferent pupillary defect.
■ AAION occurs because of occlusion of the posterior ciliary arteries
by vasculitis (e.g., GCA).
■ GCA is a rare granulomatous vasculitis that most commonly occurs
in older white women.

14 WHAT DO I DO NOW? NEURO-OPHTHALMOLOGY


■ Systemic symptoms of GCA include temporal headache, jaw
claudication, scalp tenderness, malaise, weight loss, and fever.
■ About 50% of patients with GCA present with vision loss, in most
cases due to AAION.
■ High-dose corticosteroid treatment should be commenced
immediately whenever GCA is suspected and should not be delayed
while awaiting the results of laboratory studies or temporal artery
biopsy.

Further Reading
González-Gay MA, García-Porrúa C, Llorca J, et al. Visual manifestations of giant cell
arteritis: trends and clinical spectrum in 161 patients. Medicine. 2000;79:283–292.
Hayreh SS, Zimmerman B. Management of giant cell arteritis. Our 27-year clinical study:
new light on old controversies. Ophthalmologica. 2003;217:239–259.
Kawasaki A, Purvin V. Giant cell arteritis: an updated review. Acta Ophthalmol.
2009;87:13–32.
Parikh M, Miller NR, Lee AG, et al. Prevalence of a normal C-reactive protein with an
elevated erythrocyte sedimentation rate in biopsy-proven giant cell arteritis.
Ophthalmology. 2006;113:1842–1845.

2. ARTERITIC ANTERIOR ISCHEMIC OPTIC NEUROPATHY 15


3 Nonarteritic Anterior
Ischemic Optic Neuropathy

An overweight 58-year-old white man presents to the


emergency department after waking with painless vision
loss in his right eye. He has hypertension, diabetes, and
erectile dysfunction. He is taking multiple antihypertensive
medications. Examination reveals visual acuities of
20/200 in the right eye and 20/20 in the left eye. He
cannot identify the control Ishihara color plate with the
right eye, but he correctly identifies all plates with the
left eye. Confrontation visual fields reveal an inferior
altitudinal defect in the right eye. His pupils are equal in
size, but there is a right relative afferent pupillary defect.
Funduscopic examination reveals optic disc edema in the
right eye and a small, structurally congested optic disc in
the left eye.

What do you do now?

16
T he acute onset of painless monocular vision loss with associated optic
disc edema in this middle-aged man with multiple vascular risk factors
suggests nonarteritic anterior ischemic optic neuropathy (NAION).
NAION is the most common cause of acute-onset optic neuropathy in
older adults in the Western world. It typically produces a rapid onset of
painless monocular vision loss that worsens over hours to days, with associ-
ated optic disc edema (see Figure 3-1) and a relative afferent pupillary
defect. An inferior altitudinal visual field defect is often present, although
other visual field defects (e.g., superior altitudinal, arcuate, or central) can
also occur. The optic disc edema is often segmental (e.g., superior greater
than inferior) and gradually resolves over weeks, with the optic disc ulti-
mately becoming pale in a segmental fashion. The visual acuity and field
defects usually stabilize in the days following onset. In contrast with optic
neuritis (see case 1), there is rarely a substantial recovery of vision thereafter,
although there can be minor improvement. There is a small risk of recur-
rence in the affected eye, but there is a significant risk of NAION occurring
in the fellow eye (about 15% over 5 years), which our patient should be
warned about.
NAION occurs due to hypoperfusion and ischemia of the optic nerve
head. In contrast with the arteritic form (AAION; see case 2), where giant
cell arteritis (GCA) leads to inflammatory occlusion of the vessels supplying
the optic nerve head, the pathogenesis of NAION is poorly understood.
Since many affected patients have vascular risk factors, such as diabetes and
hypertension, there may be atherosclerotic stenoses in the vessels supplying
the optic nerve head. Given that many patients awake with the vision loss,
nocturnal hypotension and obstructive sleep apnea might play an impor-
tant role in the pathogenesis of NAION. It is important to ask our patient
about the timing of his antihypertensive medication doses, because the use
of antihypertensive medications at night might exacerbate nocturnal
hypotension. Several other factors (e.g., vasospasm and impaired vascular
autoregulation) might play a role in the pathogenesis of NAION in some
patients. Consistently, however, affected patients have small, structurally
congested optic discs, with a small or absent physiologic cup (see Figure 3-1);
this normal variant is well established as a marker for increased risk of
NAION and has been called a “disc at risk”, although the exact means by
which it influences the pathogenesis is unclear. The “disc at risk” is more

3. NONARTERITIC ANTERIOR ISCHEMIC OPTIC NEUROPATHY 17


FIGURE 3-1 Fundus photographs demonstrating optic disc edema due to nonarteritic
anterior ischemic optic neuropathy in the right eye and a small, structurally congested optic
disc (“disc at risk”) in the left eye.

common in white patients, which might explain why there is a higher inci-
dence of NAION in whites than in other racial groups.
Several medications have been implicated as precipitants for NAION.
Amiodarone has been reported to cause bilateral simultaneous optic neu-
ropathies that are reminiscent of NAION, although a causal link has not yet
been definitively established in larger studies. NAION might also occur in
association with the use of phosphodiesterase type-5 inhibitors (sildenafil,
vardenafil, or tadalafil) for erectile dysfunction. However, the evidence sug-
gesting a relationship between NAION and phosphodiesterase type-5
inhibitor use comes mostly from case reports or small series (Thurtell &
Tomsak, 2008); a causal association has not yet been definitively demon-
strated. Nevertheless, it is important to ask our patient about the use of
such medications, because their continued use could increase the risk of
NAION occurring in the fellow eye.
NAION is a clinical diagnosis that must be differentiated from AAION,
because patients with AAION can develop devastating vision loss in the
fellow eye if corticosteroid therapy is not immediately commenced.
Although AAION is also characterized by acute onset of severe monocular
vision loss with optic disc edema, the patient often has other symptoms to
suggest GCA (e.g., headache, jaw claudication, or scalp tenderness). If our

18 WHAT DO I DO NOW? NEURO-OPHTHALMOLOGY


patient has such symptoms, his inflammatory markers should be checked.
If they are elevated, he should be immediately begun on steroids and a tem-
poral artery biopsy should be obtained (see case 2).
There is no effective treatment for NAION, although a number of poten-
tial therapies have been studied. In a multicenter randomized trial, optic
nerve decompression was found to be ineffective and potentially harmful
(Ischemic Optic Neuropathy Decompression Trial Research Group, 1995).
The findings of one nonrandomized and unmasked study (Hayreh &
Zimmerman, 2008) suggest that oral prednisone treatment might result in
a higher probability of improvement in visual acuity and visual field loss,
compared with no treatment. However, the use of steroids for NAION
remains controversial. Furthermore, there is no proven prophylactic therapy
to prevent NAION from occurring in the fellow eye. The focus is therefore
on reducing the risk of further events by eliminating potential precipitating
factors, such as nocturnal hypotension and obstructive sleep apnea. In addi-
tion, treatment of vascular risk factors and the use of antiplatelet therapy
(e.g., aspirin 81 mg daily) should be considered, although there is no trial
evidence to suggest that these strategies prevent recurrence or fellow eye
involvement.

K EY P O I N TS TO R E M E M B E R

■ NAION is the most common cause of acute-onset optic neuropathy


in older adults.
■ NAION causes a rapid onset of painless monocular vision loss with
associated optic disc edema and a relative afferent pupillary defect.
■ NAION usually occurs in eyes with small, structurally congested
optic discs (“discs at risk”).
■ NAION can occur in association with nocturnal hypotension, which
can be exacerbated by evening dosing of antihypertensive
medications.
■ No treatment has been definitively shown to improve visual
recovery, prevent recurrence, or prevent fellow eye involvement in
NAION, although avoidance of precipitating factors, treatment of
vascular risk factors, and antiplatelet therapy could be considered.

3. NONARTERITIC ANTERIOR ISCHEMIC OPTIC NEUROPATHY 19


Further Reading
Arnold AC. Pathogenesis of nonarteritic anterior ischemic optic neuropathy.
J Neuroophthalmol. 2003;23:157–163.
Hayreh SS, Zimmerman MB. Non-arteritic anterior ischemic optic neuropathy: role of
systemic corticosteroid therapy. Graefes Arch Clin Exp Ophthalmol. 2008;246:
1029–1046.
Hayreh SS, Zimmerman MB, Podhajsky PA, Alward WL. Nocturnal arterial hypotension
and its role in optic nerve head and ocular ischemic disorders. Am J Ophthalmol.
1994;117:603–624.
Ischemic Optic Neuropathy Decompression Trial Research Group. Optic nerve
decompression surgery for nonarteritic anterior ischemic optic neuropathy (NAION)
is not effective and may be harmful. JAMA. 1995;273:625–632.
Thurtell MJ, Tomsak RL. Nonarteritic anterior ischemic optic neuropathy with PDE-5
inhibitors for erectile dysfunction. Int J Impot Res. 2008;20:537–543.

20 WHAT DO I DO NOW? NEURO-OPHTHALMOLOGY


4 Compressive Optic
Neuropathy

A 40-year-old woman presents to your clinic with a


several-month history of progressive painless vision
loss in her right eye. Examination reveals visual acuities
of 20/40 in the right eye and 20/20 in the left eye.
She correctly identifies 4 of 14 Ishihara color plates
with the right eye and 14 of 14 plates with the left eye.
Confrontation visual fields reveal a subtle central
scotoma in the right eye. Her pupils are equal in size,
but there is a right relative afferent pupillary defect.
Funduscopic examination reveals temporal optic disc
pallor in the right eye. MRI of the orbits with contrast
reveals an enhancing lesion surrounding the right optic
nerve.

What do you do now?

21
T he progressive onset of painless monocular vision loss in this patient’s
right eye, with associated dyschromatopsia, temporal optic disc pallor,
and a relative afferent pupillary defect, suggests a compressive right optic
neuropathy. Compressive optic neuropathies can be divided into anterior
and posterior forms: optic disc edema is often present when the compres-
sion is anterior, whereas it is usually absent when the compression is poste-
rior. Both forms are characterized by progressive, and usually painless,
central vision loss. The patient often has dyschromatopsia that is out of
proportion to the decrease in visual acuity. Formal visual field testing should
be obtained to determine the extent of visual field loss; it will often demon-
strate a central visual field defect, which is typically subtle, and blind-spot
enlargement in those patients with optic disc edema due to anterior com-
pressive optic neuropathy. Optociliary collateral vessels may be present on
funduscopic examination in patients with long-standing anterior compres-
sive optic neuropathy (see Figure 4-1), and there may also be retinal folds.
In contrast, patients with long-standing posterior compressive optic neu-
ropathy usually develop progressive optic disc pallor without shunt vessels
or folds. Patients with orbital pathology as the cause for their compressive
optic neuropathy may have orbital signs, such as proptosis, chemosis,

FIGURE 4-1 Optic disc photograph demonstrating mild right optic disc edema, with
optociliary collateral vessels (left). Magnetic resonance imaging (MRI) of the orbit
demonstrates a large enhancing lesion in the right orbit that has an appearance consistent
with an optic nerve sheath meningioma (right).

22 WHAT DO I DO NOW? NEURO-OPHTHALMOLOGY


conjunctival injection, eyelid abnormalities, or reduced ocular ductions.
However, the absence of these signs does not exclude orbital pathology as
the cause for the optic neuropathy.
Common causes for compressive optic neuropathy include orbital
tumors (e.g., optic nerve sheath meningioma, optic glioma, capillary
hemangioma, or lymphoma), intracranial tumors (e.g., pituitary macroad-
enoma, meningioma, or craniopharyngioma; see case 8), aneurysms (e.g.,
internal carotid or ophthalmic artery), orbital infections (e.g., bacterial
infection or fungal infection; see case 34), and orbital inflammation (e.g.,
idiopathic orbital inflammation or “orbital pseudotumor”). Thyroid eye
disease can also cause anterior or posterior compressive optic neuropathy,
sometimes in the absence of orbital signs (see case 36). Most causes of com-
pressive optic neuropathy can be diagnosed with imaging. Magnetic reso-
nance imaging (MRI) of the orbits (with contrast and fat suppression) is the
imaging modality of choice in most cases, although computed tomography
(CT) is preferable for demonstrating calcification or if details of the bony
anatomy are required. Depending on the imaging findings, biopsy of the
causative lesion may be required for definitive diagnosis.
In our patient, MRI of the orbits has revealed an enhancing lesion sur-
rounding the right optic nerve (see Figure 4-1), which is likely to be an
optic nerve sheath meningioma (ONSM). ONSMs occur most commonly
in middle-aged women and are usually unilateral. MRI shows that the
lesion is distinct from the optic nerve and thereby allows differentiation
from intrinsic tumors (e.g., optic glioma). Tumor calcification gives a char-
acteristic “tram-track” sign on CT. Further investigations (e.g., biopsy) are
usually not required, because the imaging findings are often diagnostic.
Treatment of ONSM depends on the severity and rate of vision loss.
Patients with minimal or no vision loss should initially be observed, because
they may remain stable without intervention. Those with more significant
or progressive vision loss, such as ours, are best managed with fractionated
stereotactic radiotherapy, which can improve or stabilize vision with mini-
mal morbidity. Surgical resection is not appropriate unless there is already
severe vision loss and another indication for resection (e.g., intractable
pain), because it invariably results in severe vision loss due to interruption
of the blood supply to the optic nerve.

4. COMPRESSIVE OPTIC NEUROPATHY 23


K EY P O I N TS TO R E M E M B E R

■ Compressive optic neuropathy is characterized by progressive


central vision loss, with dyschromatopsia and a relative afferent
pupillary defect, with or without optic disc edema.
■ Compressive optic neuropathy can be caused by orbital tumors,
intracranial tumors, aneurysms, orbital infections, orbital
inflammation, and thyroid eye disease.
■ MRI or CT of the orbits is usually sufficient to diagnose the
causative lesion, although biopsy and histopathologic examination
may be required in some cases.
■ ONSM occurs most often in middle-aged women and is best
managed conservatively (if there is minimal vision loss) or with
stereotactic radiotherapy (if there is more severe or progressive
vision loss).

Further Reading
Andrews DW, Foroozan R, Yang BP, et al. Fractionated stereotactic radiotherapy for the
treatment of optic nerve sheath meningiomas: preliminary observations of 33 optic
nerves in 30 patients with historical comparison to observation with or without prior
surgery. Neurosurgery. 2002;51:890–902.
Moster ML. Detection and treatment of optic nerve sheath meningioma. Curr Neurol
Neurosci Rep. 2005;5:367–375.
Saeed P, Blank L, Selva D, et al. Primary radiotherapy in progressive optic nerve sheath
meningiomas: a long-term follow-up study. Br J Ophthalmol. 2010;94:564–568.
Soares-Welch CV, Fatourechi V, Bartley GB, et al. Optic neuropathy of Graves disease:
results of transantral orbital decompression and long-term follow-up in 215 patients.
Am J Ophthalmol. 2003;136:433–441.

24 WHAT DO I DO NOW? NEURO-OPHTHALMOLOGY


5 Hereditary Optic Neuropathy

You are called to the emergency department to see a


14-year-old boy who has suddenly developed painless
vision loss in his right eye. There is a history of vision loss
on the maternal side of his family. Examination reveals
visual acuities of 20/200 in the right eye and 20/15 in the
left eye. He is unable to identify the control Ishihara color
plate with the right eye, but he correctly identifies all
plates with the left eye. Confrontation visual fields reveal
a dense central scotoma in the right eye. His pupils are
equal in size, but there is a right relative afferent
pupillary defect. Funduscopic examination reveals trace
optic disc edema in the right eye and a normal optic disc
in the left eye.

What do you do now?

25
W hen a young patient has an acute onset of monocular vision loss and
has clinical signs consistent with a unilateral optic neuropathy, the
most likely diagnosis is idiopathic optic neuritis (see case 1). However, the
absence of pain is atypical and, consequently, other diagnoses should also be
considered in this patient. Nonarteritic anterior ischemic optic neuropathy
(NAION) can occasionally cause an acute painless optic neuropathy in a
younger patient who does not have vascular risk factors, but the absence of
significant optic disc edema in this patient argues against that diagnosis. An
acute onset of painless optic neuropathy can also be caused by sudden optic
nerve compression (see case 4); magnetic resonance imaging (MRI) of the
orbits should be obtained urgently to exclude this possibility, because a sub-
stantial recovery of vision is possible with timely decompression.
In this patient, the presence of a family history of vision loss should
immediately suggest hereditary optic neuropathy. While a progressive onset
of painless binocular central vision loss is more suggestive of dominant
optic atrophy (DOA), acute onset of painless monocular vision loss is highly
suggestive of Leber’s hereditary optic neuropathy (LHON). LHON is a rare
disease that is caused by point mutations in the mitochondrial DNA
(mtDNA). The most common causative point mutations are at positions
11778 (in about 69% of cases), 14484 (in about 14% of cases), and 3460
(in about 13% of cases) in the mtDNA; these mutations involve genes that
encode subunits of complex I of the mitochondrial respiratory chain. Many
rarer mtDNA point mutations have been reported to cause LHON. Because
LHON has a maternal inheritance, the mutation can only be passed to
offspring by females. Most affected patients are male, with onset of symp-
toms typically occurring between the ages of 15 and 35 years. The usual
presentation is with acute painless central vision loss in one eye, with clini-
cal signs of a unilateral optic neuropathy. Funduscopic examination at the
time of onset will often reveal what appears to be trace optic disc edema,
due to swelling of the retinal nerve fiber layer around the optic disc, with
optic disc hyperemia and peripapillary telangiectatic vessels. These fundu-
scopic changes gradually resolve, resulting in temporal optic disc atrophy.
Unfortunately, the vision loss is usually permanent, although some patients
with the 14484 mutation report subsequent improvement in vision.
However, almost all patients will develop fellow eye involvement within
1 year, often within 6–8 weeks of their initial presentation.

26 WHAT DO I DO NOW? NEURO-OPHTHALMOLOGY


LHON can be diagnosed by demonstrating the presence of a causative
mtDNA mutation. When testing for such mutations is unrevealing but
clinical suspicion for LHON is high, whole mtDNA genome sequencing
might reveal a rare or novel mutation. Once the diagnosis is confirmed,
many patients ask if there is a way to prevent fellow eye involvement.
However, factors that influence the expression of the disease remain uncer-
tain. There is limited evidence that certain environmental factors, such as
tobacco and alcohol, might play a role in triggering LHON. Thus, patients
should be advised to avoid tobacco and excessive alcohol consumption.
Treatment options for LHON remain limited. A variety of treatments
have been proposed, including coenzyme Q10, succinate, and various vita-
mins, but there are only anecdotal reports of a beneficial effect. Preliminary
findings from a prospective randomized controlled study of idebenone (an
analogue of coenzyme Q10) suggest that it may improve vision in patients
with LHON. Gene therapy might also be effective for treating the acute
event and preventing fellow eye involvement; results of clinical trials are
awaited (Lam et al., 2010). From a practical point of view, genetic counsel-
ing and evaluation by a low-vision specialist are often very helpful.

K EY P O I N TS TO R E M E M B E R

■ LHON demonstrates maternal inheritance and most commonly


occurs in young men.
■ LHON causes an acute onset of severe (and usually irreversible)
painless monocular vision loss, with subsequent fellow eye
involvement within 6–12 months.
■ LHON can be definitively diagnosed if a pathogenic mtDNA point
mutation is detected.
■ Treatment options for LHON are limited, although idebenone
treatment and gene therapies hold promise.

Further Reading
Fraser JA, Biousse V, Newman NJ. The neuro-ophthalmology of mitochondrial disease.
Surv Ophthalmol. 2010;55:299–334.
Kirkman MA, Yu-Wai-Man P, Korsten A, et al. Gene–environment interactions in Leber
hereditary optic neuropathy. Brain. 2009;132:2317–2326.

5. HEREDITARY OPTIC NEUROPATHY 27


Lam BL, Feuer WJ, Abukhalil F, Porciatti V, Hauswirth WW, Guy J. Leber hereditary optic
neuropathy gene therapy clinical trial recruitment: year 1. Arch Ophthalmol.
2010;128:1129–1135.
Preechawat P, Bruce BB, Newman NJ, Biousse V. Anterior ischemic optic neuropathy in
patients younger than 50 years. Am J Ophthalmol. 2007;144:953–960.

28 WHAT DO I DO NOW? NEURO-OPHTHALMOLOGY


6 Idiopathic Intracranial
Hypertension

A 22-year-old overweight woman presents to your clinic


with a several-month history of increasing headaches.
She also reports having multiple brief episodes of
complete vision loss in both eyes, precipitated by
postural changes. She is otherwise well, does not take
any medications, but has gained 60 pounds in weight
over the past 6 months. Examination reveals visual
acuities of 20/20 in both eyes. She correctly identifies all
Ishihara color plates with both eyes. Confrontation visual
fields are full in both eyes. Her pupils are equal in size
and there is no relative afferent pupillary defect. Eye
movements are normal. Funduscopic examination reveals
bilateral optic disc edema.

What do you do now?

29
T he patient in this scenario presents with symptoms and signs of raised
intracranial pressure (ICP). Headache is the most frequent symptom of
raised ICP and is typically a daily holocranial headache, which awakens the
patient and is exacerbated by maneuvers that increase ICP (e.g., coughing
or straining). However, headache is a nonspecific symptom that can be
caused by many other disorders. Consequently, it is important to inquire
about other symptoms of raised ICP, such as pulsatile tinnitus, a common
symptom that is often not volunteered by the patient and seldom present in
patients with other causes of headache.
Visual symptoms are common in patients with raised ICP and are often
related to papilledema, which is the term used to describe optic disc edema
that is secondary to raised ICP. Papilledema is the most common examina-
tion finding in patients with raised ICP and is usually symmetric (see
Figure 6-1), but is unilateral or asymmetric in about 15% of patients.
Patients with papilledema often report brief episodes of vision loss that are
precipitated by postural changes and Valsalva-like maneuvers, with rapid
recovery (within seconds) back to baseline between episodes. These epi-
sodes, called transient visual obscurations, are thought to occur because of
transient ischemia of the edematous optic nerve head. While transient visual
obscurations are dramatic, they are not correlated with a poor visual out-
come. Papilledema can also cause progressive irreversible visual field loss
and secondary optic atrophy. Enlargement of the physiologic blind spot
is the earliest visual field change to occur (see Figure 6-1). If papilledema
is persistent or severe, nasal (often inferonasal) defects, arcuate defects,
and severe visual field constriction can develop. However, the vision loss
often goes unnoticed by the patient until severe, because visual acuity is
usually not affected until the visual field loss is advanced. Visual acuity can
be affected early if there is macular pathology (e.g., edema or exudates) or a
change in refractive error due to chorioretinal folds or posterior globe
flattening. Other common visual symptoms of raised ICP include binocu-
lar horizontal diplopia, which is due to unilateral or bilateral sixth nerve
palsies.
Raised ICP can have a number of sinister causes (see Box 6-1). Our
patient therefore requires urgent evaluation. Many causes of raised ICP,
such as mass lesions and hydrocephalus, can be detected on imaging. Unless
contraindicated, the imaging study of choice is magnetic resonance imaging

30 WHAT DO I DO NOW? NEURO-OPHTHALMOLOGY


FIGURE 6-1 Fundus photographs demonstrating bilateral papilledema in a patient with
idiopathic intracranial hypertension (top row). Automated perimetry reveals enlarged blind
spots and mild peripheral visual field defects in both eyes (bottom row).

(MRI) of the brain with and without contrast. Even if no cause for raised
ICP is identified on imaging, there may be signs that are consistent with
raised ICP (e.g., empty sella, posterior globe flattening, and distension or
tortuosity of the optic nerve sheaths). Although routine MRI is usually suf-
ficient to exclude cerebral venous sinus thrombosis (CVST), magnetic reso-
nance venography (MRV) should be obtained if there is a high clinical
suspicion for CVST. If no cause for raised ICP is identified on MRI or
MRV, the next step in evaluation is to perform a lumbar puncture in the
lateral decubitus position, in order to measure the cerebrospinal fluid (CSF)
opening pressure (normal in adults is <20 cm H2O) and evaluate the CSF
constituents.

6. IDIOPATHIC INTRACRANIAL HYPERTENSION 31


BOX 6 -1 CAUS ES O F RA I S E D I N T RAC RA N I A L P RESS U R E

Mass lesions (e.g., tumor, hemorrhage, stroke)


Obstruction of ventricular system (e.g., aqueduct stenosis,
third-ventricular tumor)
Obstruction of venous outflow (e.g., cerebral venous sinus
thrombosis)
Decreased cerebrospinal fluid (CSF) absorption (e.g., meningitis,
subarachnoid hemorrhage)
Increased CSF secretion (e.g., choroid plexus tumor)
Diffuse cerebral edema (e.g., head injury)
Medications (e.g., vitamin A, tetracyclines)
Idiopathic intracranial hypertension

Given the scenario, the most likely cause for our patient’s raised ICP is
idiopathic intracranial hypertension (IIH). Formerly known as pseudotu-
mor cerebri, IIH is a syndrome of unknown cause that occurs most fre-
quently in obese women of childbearing age, although it can occur in
children, men, and older adults. There is often a history of weight gain in
the months prior to symptom onset. Other possibilities need to be excluded
with imaging and lumbar puncture before a diagnosis of IIH can be made
(see Box 6-2 for diagnostic criteria). Use of certain medications (e.g., vita-
min A derivatives or tetracyclines) can cause an indistinguishable clinical
picture. Formal visual field testing (e.g., automated or Goldmann perime-
try) must be obtained, because visual field loss from papilledema can go
unnoticed until severe and irreversible. Consultation with an ophthalmolo-
gist or neuro-ophthalmologist is therefore essential.
The main goals of IIH treatment are to alleviate symptoms and preserve
vision. Thus, the approach depends on the severity of symptoms and vision
loss (see Figure 6-2). Treatment begins with the diagnostic lumbar punc-
ture, which may transiently improve symptoms and signs. The patient
should be counseled about the importance of modest weight loss (with a
diet and exercise program), which may be all that is required to improve
symptoms and signs; a 5%–10% loss of body weight is usually sufficient.

32 WHAT DO I DO NOW? NEURO-OPHTHALMOLOGY


BOX 6 -2 DIAGN OST I C C RI T E RI A FO R I D I O PAT H I C I NT RAC RA NI A L
H Y PERT EN S I O N

1. Symptoms and signs of raised intracranial pressure (ICP)


2. No localizing neurologic signs, except for unilateral or bilateral
sixth nerve palsies.
3. Raised cerebrospinal fluid (CSF) opening pressure, but normal CSF
composition.
4. No evidence of hydrocephalus, mass, structural, or vascular lesion
on imaging.
5. No other cause of raised ICP identified.

Evaluate severity:
• Severity/tolerance of headache
• Diplopia (VIth nerve palsy)
• Visual function: visual acuity, visual field testing

Evaluate and correct predisposing factors


Careful follow-up (visual acuity, formal visual fields)

No headache or diplopia Headaches ± diplopia ± tinnitus Headaches ± diplopia ± tinnitus


No visual loss No visual loss Visual loss
Normal visual fields Normal visual fields Visual field defects

Mild Severe
Conservative management: Medical management:
• No treatment • ± repeat lumbar puncture
• Careful follow-up • Acetazolamide ± Repeat lumbar puncture
(immediate decrease in ICP)

Consider surgical treatment:


Surgical treatment
CSF shunting procedure

No or mild
Headaches and diplopia headaches

Optic nerve sheath


CSF shunting procedure >> fenestration >> CSF
Optic nerve sheath fenestration shunting procedure

FIGURE 6-2 Algorithm for treatment of idiopathic intracranial hypertension. CSF,


cerebrospinal fluid; ICP, intracranial pressure. (Adapted from Thurtell MJ, Bruce BB,
Newman NJ, Biousse V. An update on idiopathic intracranial hypertension. Rev Neurol Dis.
2010;7:e64.).

6. IDIOPATHIC INTRACRANIAL HYPERTENSION 33


Potential contributing factors (e.g., obstructive sleep apnea) should also be
diagnosed and treated.
A variety of medical and surgical treatments can be offered to patients
with IIH, although no large randomized prospective trials have evaluated
the efficacy of these treatments. When there is minimal or mild visual field
loss, as in our patient, medical treatment with acetazolamide should be
initiated. Acetazolamide is a carbonic anhydrase inhibitor that is thought to
decrease CSF production. Doses of 1–2 g per day are often required for a
clinical effect but can produce side effects (e.g., paresthesias, altered taste
sensation, lethargy, and, rarely, renal calculi). When visual field loss is severe
or rapidly progressive, surgical treatment should be considered. The two
main options are CSF shunting (e.g., ventriculoperitoneal or lumboperito-
neal shunting) and optic nerve sheath fenestration (ONSF). Although CSF
shunting often produces a rapid reduction in ICP, it is associated with a
high complication rate (e.g., shunt infections and obstruction), such that
shunt revisions are often required. ONSF results in a rapid reduction in the
pressure on the optic nerve, leading to reduced papilledema and improve-
ment in vision, but it is often not effective in treating other symptoms and
signs of raised ICP. Regardless of the treatment strategy used, all patients
require careful follow-up, because IIH is a chronic disease.
Stenoses of the transverse cerebral venous sinuses are often present on
MRV in patients with IIH. Although their origin remains debated, the
stenoses might cause cerebral venous hypertension and thereby exacerbate
intracranial hypertension. Endovascular stenting of these stenoses has been
proposed as a treatment for IIH patients who have failed conventional ther-
apy but can result in serious complications (e.g., subdural hematoma). At
present, the role of endovascular stenting remains unclear and therefore it
should not be routinely recommended as a treatment for IIH.

K EY P O I N TS TO R E M E M B E R

■ Papilledema specifically refers to optic disc edema that is


secondary to raised ICP.
■ Papilledema can produce transient visual obscurations and
progressive insidious visual field loss, with sparing of central vision
until late.

34 WHAT DO I DO NOW? NEURO-OPHTHALMOLOGY


■ Raised ICP can be caused by mass lesions, obstruction of the
ventricular system, obstruction of cerebral venous outflow,
decreased CSF absorption, increased CSF secretion, diffuse
cerebral edema, medications, and IIH.
■ IIH is a syndrome of unknown cause that often occurs in obese
women of childbearing age.
■ Treatment options for IIH include weight loss, acetazolamide, and
surgical procedures (e.g., CSF shunting procedures and ONSF).

Further Reading
Sinclair AJ, Burdon MA, Nightingale PG, et al. Low energy diet and intracranial pressure
in women with idiopathic intracranial hypertension: prospective cohort study. BMJ.
2010;341:c2701.
Thurtell MJ, Bruce BB, Newman NJ, Biousse V. An update on idiopathic intracranial
hypertension. Rev Neurol Dis. 2010;7:e56–e68.
Wall M. The headache profile of idiopathic intracranial hypertension. Cephalalgia.
1990;10:331–335.
Wall M, George D. Idiopathic intracranial hypertension: a prospective study of 50 patients.
Brain. 1991;114:155–180.

6. IDIOPATHIC INTRACRANIAL HYPERTENSION 35


7 Pseudopapilledema

A thin 14-year-old white girl presents to your clinic with a


several-month history of headache. She has not noticed
any changes in her vision, but her neurologist noted that
her optic nerve heads were elevated. Examination reveals
visual acuities of 20/20 in both eyes. She correctly
identifies all Ishihara color plates with both eyes.
Confrontation visual fields are full in both eyes. Her
pupils are equal in size and there is no relative afferent
pupillary defect. Eye movements are normal. Funduscopic
examination reveals elevated optic discs. She has already
undergone a thorough workup, including magnetic
resonance imaging (MRI) of the brain with contrast,
magnetic resonance venography of the head, and lumbar
puncture, but no cause for her optic nerve head elevation
has been established.

What do you do now?

36
W hen a patient with headaches is found to have elevated optic nerve
heads, the initial concern should be for papilledema (i.e., optic disc
edema due to raised intracranial pressure; see case 6). However, optic nerve
head elevation can be due to optic disc edema from another cause (e.g.,
anterior ischemic optic neuropathy or papillitis). It can also be caused by
optic nerve head infiltration (e.g., due to sarcoid) or might simply reflect a
benign anomaly of the optic nerve head (see Box 7-1). Since anomalous
optic nerve head elevation can mimic optic disc edema, it is often termed
pseudopapilledema.
In many cases, pseudopapilledema can be distinguished from papille-
dema on the basis of the clinical history and examination findings.
Papilledema is characterized by retinal nerve fiber layer (RNFL) edema,
which gives the RNFL an opaque appearance that obscures the underlying
retinal vessels and produces a halo around the optic disc (see Figure 7-1). In
pseudopapilledema, the optic disc margins are sharp and there is no obscu-
ration of the vessels, because there is no RNFL edema (see Figure 7-1).
Several other clinical features can help to differentiate pseudopapilledema
from papilledema (see Table 7-1).
Many patients with optic nerve head elevation undergo extensive inves-
tigations before the possibility of pseudopapilledema is considered.
The patient in this scenario has had several investigations looking for causes
of raised intracranial pressure, but these have been unrevealing. It is there-
fore possible that her optic nerve head elevation is an incidental finding

BOX 7-1 DIFF ER EN T I A L D I AG N OS I S O F PA P I L L E D E M A

Optic disc edema of another cause (e.g., anterior ischemic optic


neuropathy, papillitis)
Optic nerve head infiltration (e.g., sarcoid granuloma, optic nerve
head melanoma)
Congenitally small, structurally congested optic disc (“little red disc”)
Congenitally tilted optic disc
Myelinated nerve fiber layer
Vitreopapillary traction
Optic nerve head drusen

7. PSEUDOPAPILLEDEMA 37
FIGURE 7-1 Optic disc photographs demonstrating papilledema (left), with swollen retinal
nerve fiber layer causing obscuration of the vessels, compared with pseudopapilledema
(right), in which there is no vessel obscuration.

TABLE 7-1 Clinical Features of Papilledema Compared with


Pseudopapilledema

Clinical feature Papilledema Pseudopapilledema

Transient visual obscurations Yes Rarely

Enlarged blind spot Yes Sometimes

Spontaneous venous pulsations No Yes

Obscuration of vessels Yes No

Venous congestion Yes No

Early branching of vessels No Sometimes

Hemorrhages Yes (retinal nerve Rarely (subretinal)


fiber layer)

Preserved physiologic cup Yes (until late) No

Retinal folds Sometimes No

Fluorescein leakage Yes No

Other neurologic signs Sometimes Rarely

38 WHAT DO I DO NOW? NEURO-OPHTHALMOLOGY


and unrelated to her headaches. The next step in her evaluation would
therefore be an examination of the optic discs by an ophthalmologist or
neuro-ophthalmologist looking for features that might suggest a cause for
her optic nerve head elevation. The history of her headaches should also be
reevaluated, as there may be a treatable cause (e.g., migraine or chronic
daily headache).
Optic nerve head elevation is often present in patients who have congeni-
tally small, structurally congested optic discs (see case 3 and Figure 3-1).
Since these optic discs often have a red appearance, they are sometimes
called “little red discs.” Optic nerve head elevation can also be present in
patients with congenitally tilted optic discs (e.g., in those with high myopia).
Such patients often have superior bitemporal visual field defects that do not
respect the vertical meridian (see case 8).
A common cause of optic nerve head elevation that may be evident on
funduscopic examination is optic nerve head drusen (ONHD). ONHD are
laminated acellular concretions that form within the substance of the optic
nerve head. Although their etiology remains poorly understood, they usu-
ally occur in patients with small, structurally congested optic nerve heads
and are inherited in an autosomal dominant fashion. In younger patients,
ONHD are often not visible on funduscopic examination (“buried”
ONHD). However, the optic disc is often elevated and can have a “lumpy
bumpy” appearance (see Figure 7-2). As the patient ages, ONHD become
visible as rounded, whitish-yellow, crystal-like excrescences on the optic
disc (“exposed” ONHD; see Figure 7-2). They are often most conspicuous
at the nasal aspect of the disc and can coalesce to form large “rock candy”
conglomerates (see Figure 7-2). Despite a dramatic funduscopic appear-
ance, most patients with ONHD remain asymptomatic, but some patients
can develop peripheral visual field defects, presumably due to RNFL attri-
tion, which can sometimes progress to severe visual field constriction. Some
patients report transient visual obscurations. Rarely, ONHD can be com-
plicated by sudden, painless monocular vision loss due to nonarteritic ante-
rior ischemic optic neuropathy or central retinal artery occlusion.
Unfortunately, no intervention has been proven to prevent the progressive
visual field loss or vascular complications associated with ONHD, although
some authorities suggest treatment with ocular hypotensive agents.

7. PSEUDOPAPILLEDEMA 39
FIGURE 7-2 Optic disc photographs (left), autofluorescence (middle), and B-scan ultrasonography (right) in patients with exposed (top row) and buried
(bottom row) optic nerve head drusen.
While ONHD are easily diagnosed if drusen are visible on funduscopic
examination, optic nerve head elevation from buried ONHD can be diffi-
cult to distinguish from mild papilledema. Several forms of ophthalmic
imaging are often helpful for making the distinction. Since ONHD exhibit
autofluorescence, they may be visible on autofluorescence photography,
even when they are not evident on funduscopic examination (see Figure 7-2).
When calcified, ONHD are usually obvious on B-scan ultrasonography as
foci of increased reflectivity within an elevated optic nerve head, with a
characteristic posterior reduplication artifact (see Figure 7-2). Buried
ONHD may also be evident as optic nerve head calcification on computed
tomography (CT). Our patient should therefore have further ophthalmic
imaging looking for ONHD if another cause for pseudopapilledema is not
evident on funduscopic examination. If there continues to be a concern for
papilledema, several other investigations can be helpful. Fluorescein angiog-
raphy typically demonstrates late leakage of dye beyond the optic disc
margin in papilledema, whereas there is no leakage in pseudopapilledema.
Optical coherence tomography (OCT) of the peripapillary RNFL will
demonstrate increased RNFL thickness in mild papilledema, especially
nasally, and normal thickness in pseudopapilledema. Some patients with
ONHD show RNFL thinning on OCT and the ONHD may be evident as
foci of hyperreflectivity. OCT may reveal another cause for the pseudop-
apilledema, such as vitreopapillary traction, in which there is optic nerve
head elevation due to vitreous traction. However, if doubt remains, the
patient should be followed clinically; a stable optic nerve head appearance
suggests pseudopapilledema, whereas a changing appearance is concerning
for papilledema. Regardless of the outcome of any further evaluation, ongo-
ing consultation with her neurologist is essential to optimize the treatment
of her headaches.

K EY P O I N TS TO R E M E M B E R

■ Optic nerve head elevation can be due to true optic disc edema
(e.g., papilledema or papillitis), optic nerve head infiltration
(e.g., sarcoid), or a benign optic nerve head anomaly.
■ Pseudopapilledema occurs when benign optic nerve head anomalies
mimic papilledema (e.g., when there is optic nerve head elevation).

7. PSEUDOPAPILLEDEMA 41
■ ONHD are a common cause for pseudopapilledema, but are difficult
to diagnose clinically if the ONHD are not visible on funduscopic
examination.
■ B-scan ultrasonography, optic disc autofluorescence photography,
fluorescein angiography, and OCT can help to distinguish
pseudopapilledema from papilledema.

Further Reading
Auw-Haedrich C, Staubach F, Witschel H. Optic disk drusen. Surv Ophthalmol. 2002;47:
515–532.
Brodsky MC. Congenital anomalies of the optic disc. In: Miller NR, Newman NJ, Biousse V,
Kerrison JB, eds. Walsh & Hoyt’s clinical neuro-ophthalmology. 6th ed. Philadelphia,
PA: Lippincott Williams & Wilkins; 2005:151–195.
Lee KM, Woo SJ, Hwang JM. Differentiation of optic nerve head drusen and optic disc
edema with spectral-domain optical coherence tomography. Ophthalmology.
2011;118:971–977.
Rosenberg MA, Savino PJ, Glaser JS. A clinical analysis of pseudopapilledema. I:
population, laterality, acuity, refractive error, ophthalmoscopic characteristics, and
coincident disease. Arch Ophthalmol. 1979;97:65–70.

42 WHAT DO I DO NOW? NEURO-OPHTHALMOLOGY


8 Chiasmal Syndromes

A 42-year-old man presents to the emergency


department following the sudden onset of a severe global
headache with vomiting. He has a 10-month history of
galactorrhea. Examination reveals visual acuities of
20/20 in both eyes. He correctly identifies all Ishihara
color plates with both eyes. Confrontation visual fields
reveal a dense bitemporal hemianopia. His pupils are
equal in size but mildly sluggish, and there is no relative
afferent pupillary defect. Eye movements are normal.
Funduscopic examination is normal and the remainder of
his cranial nerve examination is unrevealing.

What do you do now?

43
B itemporal hemianopia is highly localizing to the optic chiasm, because
this is the only location in the nervous system where the nerve fibers
subserving vision from both temporal fields are in close proximity.
Depending on the location of the lesion, several variations on the bitempo-
ral hemianopic field defect can be recognized (see Figure 8-1). For instance,
if the intracranial segment of one optic nerve is involved, there will usually
be a central visual field defect in that eye; the combination of a central
visual field defect in one eye and a temporal defect in the other is known as
a junctional scotoma, because it is caused by a lesion at the junction of the
optic nerve and chiasm (see Figure 8-1). Similar bitemporal visual field
defects can occur in patients with tilted optic discs (see case 7), but the
defects do not respect the vertical meridian, unlike those of a true bitempo-
ral hemianopia. True bitemporal hemianopias usually occur in patients with
optic chiasm dysfunction due to compression by pituitary macroadenomas,
suprasellar meningiomas, craniopharyngiomas, gliomas, or aneurysms of
the internal carotid artery. In cases where the compression occurs gradually
over months or years, the visual field defect develops insidiously and may
go unnoticed by the patient. It is not uncommon for such visual field
defects to be identified on a routine eye examination or screening visual
field testing.
Although most causes of optic chiasm compression can be identified on
computed tomography (CT) or magnetic resonance imaging (MRI) of the
brain, the clinical presentation occasionally suggests a specific diagnosis.
The acute onset of severe headache with vomiting in this man with galactor-
rhea and bitemporal hemianopia should suggest a macroprolactinoma with
pituitary apoplexy. Pituitary apoplexy is a rare, life-threatening clinical syn-
drome that results from infarction of (or hemorrhage into) a pituitary mac-
roadenoma. In many cases, the macroadenoma is nonfunctioning and was
previously undiagnosed. When the infarction or hemorrhage occurs, there
is a rapid increase in the size of the tumor, leading to compression of adja-
cent structures in the suprasellar cistern and cavernous sinuses. Subsequently,
there is a dramatic onset of symptoms and signs, which can include head-
ache, meningism, vomiting, vision loss, ophthalmoplegia, stupor, and vas-
cular collapse. Factors implicated as precipitants for pituitary apoplexy
include major surgery (e.g., cardiac bypass surgery) and anticoagulant use.

44 WHAT DO I DO NOW? NEURO-OPHTHALMOLOGY


FIGURE 8-1 Visual field defects occurring with chiasmal lesions.

8. CHIASMAL SYNDROMES 45
FIGURE 8-2 Coronal (left) and sagittal (right) magnetic resonance imaging (MRI) of the
brain showing a large pituitary macroadenoma extending into the suprasellar cistern.
The patient had a dense bitemporal hemianopia.

MRI is the imaging modality of choice for diagnosing pituitary apo-


plexy, because it clearly demonstrates both pituitary infarction and hemor-
rhage and shows the degree of compression of neighboring structures (see
Figure 8-2). Alternatively, a brain CT with contrast can be obtained,
although the lesion might not be identified unless specifically sought. Our
patient therefore requires emergent imaging with MRI, formal visual field
testing (if possible), and admission to the hospital for treatment. Intravenous
corticosteroids (e.g., hydrocortisone) should be administered immediately,
to treat acute hypoadrenalism, and an endocrinologist should be consulted,
because the patient may have or subsequently develop panhypopituitarism.
Although improvements in vision can occur with nonsurgical treatment
alone, decompression of the lesion is generally advised unless the patient is
too unwell to have an anesthetic. The appropriate timing of surgery remains
controversial, as there are reports of a good visual outcome in patients with
severe vision loss who did not undergo decompression until several days
following onset of symptoms. However, the degree of visual recovery can be
poor, even when decompression is performed within 24 hours. Although
the apparent poor recovery might be a consequence of long-standing chias-
mal compression producing irreversible nerve fiber layer loss, urgent decom-
pression should be considered in patients with pituitary apoplexy, to
maximize the chance of some visual recovery.

46 WHAT DO I DO NOW? NEURO-OPHTHALMOLOGY


K EY P O I N TS TO R E M E M B E R

■ A bitemporal hemianopic visual field defect localizes the lesion to


the optic chiasm.
■ Chiasmal dysfunction is most commonly caused by pituitary
macroadenomas, suprasellar meningiomas, craniopharyngiomas,
optic gliomas, and aneurysms of the internal carotid artery.
■ Pituitary apoplexy is a rare, life-threatening clinical syndrome that
results from infarction of (or hemorrhage into) a pituitary
macroadenoma.
■ Pituitary apoplexy requires immediate treatment with intravenous
steroids and, in many cases, emergent decompression of the optic
apparatus.

Further Reading
Agrawal D, Mahapatra AK. Visual outcome of blind eyes in pituitary apoplexy after
transsphenoidal surgery: a series of 14 eyes. Surg Neurol. 2005;63:42–46.
Biousse V, Newman NJ, Oyesiku NM. Precipitating factors in pituitary apoplexy. J Neurol
Neurosurg Psychiatry. 2001;71:542–545.
Piotin M, Tampieri D, Rufenacht DA, et al. The various MRI patterns of pituitary apoplexy.
Eur Radiol. 1999;9:918–923.
Semple PL, Webb MK, de Villiers JC, Laws ER Jr. Pituitary apoplexy. Neurosurgery.
2005;56:65–73.

8. CHIASMAL SYNDROMES 47
9 Homonymous Hemianopia

You are called to see a 72-year-old woman who was


brought to the emergency department following a motor
vehicle accident. She was the driver of the vehicle at
fault and claims to have not seen the other vehicle, but
she denies vision loss. Examination reveals visual acuities
of 20/25 in both eyes. She correctly identifies most
Ishihara color plates with both eyes, but she occasionally
misses the numbers on the left side of the plates.
Confrontation visual fields reveal a left homonymous
hemianopia. Pupils, eye movements, and funduscopic
examination are all normal. Magnetic resonance
imaging (MRI) of the brain reveals a lesion in the right
occipital lobe.

What do you do now?

48
U nilateral lesions affecting the retrochiasmal visual pathways or primary
visual cortex produce homonymous visual field defects in the contral-
ateral hemifield of both eyes. The pattern of the visual field defect and pres-
ence of other neurologic symptoms or signs can help in the localization of
the causative lesion. For instance, an incongruent visual field defect (i.e.,
one that is dissimilar in the two eyes) with a relative afferent pupillary defect
in the eye with the temporal defect suggests a lesion affecting the optic tract.
It is also useful to know the tempo of onset of visual field loss, because this
can also suggest the etiology (e.g., sudden onset of homonymous visual field
loss implies a stroke, whereas gradual onset implies a tumor). The initial
evaluation of a patient with homonymous visual field loss should therefore
include a careful history, to determine the temporal profile of onset and to
inquire about other neurologic symptoms, and a neurologic examination
looking for signs that aid in localization of the lesion. Formal visual field
testing should also be obtained, if possible, to determine the pattern of the
visual field defect.
When a complete homonymous visual field defect (homonymous hemi-
anopia) is present without other neurologic symptoms or signs, the lesion is
likely to be in the occipital lobe. In some such cases, the patient may not
become aware of the visual field defect until it is called to their attention
(e.g., with formal visual field testing or following a motor vehicle accident).
The nature of the causative lesion is often evident on MRI of the brain (see
Figure 9-1), which should be obtained with and without contrast. The sub-
sequent evaluation and treatment of the patient depends on the nature of
the lesion. For instance, a patient with an occipital infarct requires a stroke
workup, whereas a patient with an occipital tumor might require a biopsy
or workup for metastatic disease.
The prognosis for recovery of homonymous hemianopia varies depend-
ing on the cause. Unfortunately, most patients are left with a permanent
visual field defect, although many report an improvement in vision due to
the development of adaptive strategies (e.g., increased eye and head move-
ments into the blind hemifield) to compensate for their visual field loss.
Several recent studies assessing on-road driving performance have reported
that some such patients might be able to drive safely. However, patients
with homonymous hemianopia are not permitted to drive by law in many
jurisdictions. It is therefore crucial to obtain formal visual field testing to

9. HOMONYMOUS HEMIANOPIA 49
FIGURE 9-1 Magnetic resonance imaging (MRI) showing infarction in the territory of the
right posterior cerebral artery in a patient with a left homonymous hemianopia.

determine the extent of the visual field defect before the patient is allowed
to drive, even if there has been an apparent recovery.
A number of therapies have been proposed for the rehabilitation of hom-
onymous hemianopia, but these remain controversial. Some commercially
available therapies claim to be able to reduce the size of the visual field
defect, but there is inadequate scientific evidence to support such claims.
Rather, these therapies are thought to bring about an apparent improve-
ment due to the development of adaptive strategies. The optimum means
by which to entrain such adaptive strategies remains under investigation.
Other approaches have been developed to functionally expand the visual
field in patients with hemianopia. One such approach involves placing a
temporary (Fresnel) prism onto the patient’s spectacle lenses above or below
the visual axis, so that part of the blind hemifield is visible to the patient
when they are looking straight ahead through the central prism-free area of
the lens. Because these approaches can help some patients with obstacle
avoidance, referral to a low-vision specialist should be considered in patients
with functional impairment due to homonymous hemianopia.

50 WHAT DO I DO NOW? NEURO-OPHTHALMOLOGY


K EY P O I N TS TO R E M E M B E R

■ Homonymous visual field defects result from unilateral lesions


affecting the retrochiasmal visual pathways or primary visual
cortex.
■ The pattern of the visual field defect and presence of other
neurologic symptoms or signs might help to localize the causative
lesion.
■ Formal visual field testing must be obtained to determine the extent
of the hemianopia, because patients with homonymous hemianopia
are not permitted to drive in many jurisdictions.
■ Many patients with persistent homonymous hemianopia report
improvement due to the development of adaptive strategies, but
rehabilitation interventions remain controversial.

Further Reading
Bowers AR, Keeney K, Peli E. Community-based trial of a peripheral prism visual field
expansion device for hemianopia. Arch Ophthalmol. 2008;126:657–664.
Reinhard J, Schreiber A, Schiefer U, et al. Does visual restitution training change absolute
homonymous visual field defects? A fundus controlled study. Br J Ophthalmol.
2005;89:30–35.
Wood JM, McGwin G Jr, Elgin J, et al. Hemianopic and quadrantanopic field loss, eye and
head movements, and driving. Invest Ophthalmol Vis Sci. 2011;52:1220–1225.
Zhang X, Kedar S, Lynn MJ, Newman NJ, Biousse V. Homonymous hemianopias:
clinical–anatomic correlations in 904 cases. Neurology. 2006;66:906–910.

9. HOMONYMOUS HEMIANOPIA 51
1
10 Disorders
Carotid Dissection
of Higher
Visual Function

you
A 78are
year-old
call you
man
arepresents
call you to
areyour
call clinic
you arecomplaining
call you are
of
call you
“diffi culty
areseeing.”
call youHe
are
has
call
hadyoubilateral
are callcataract
you are call you
extractions
are call you and wears
are call youreading
are callglasses
you arebut
callotherwise has
you are call
no
yousignificant
are call ocular
you history.
are call He has
you are seenare
call you multiple
call you are
ophthalmologists, but
call you are call you eye
are callexaminations haveare
you are call you notcall you
revealed a cause
are call you foryou
are call his are
complaints. Magnetic
call you are resonance
call you are call
imaging (MRI)
you are call ofare
you thecall
brain
youwas
arereported
call you to
arebe normal.
call you are
Examination reveals
call you are call visual
you are callacuities
you areof 20/25
call in both
you are eyes.
call you
He
areiscall
unable to identify
you are call you the
are control
call youIshihara
are call color plate
you are call
with either eye. Confrontation visual fields are full. Pupil,
What
eye do you do
movement, now?
and funduscopic examinations are
normal. His wife enters the room at the end of the
examination, and you notice that he does not recognize
her until she speaks.

What do you do now?

52
W hen a patient reports visual difficulties that are out of proportion to
the findings on ophthalmic examination, a disorder of higher visual
function should be considered. These disorders are caused by lesions affect-
ing visual association areas or their interconnections and often go undiag-
nosed for long periods, because the routine ophthalmic examination does
not incorporate screening tests to allow their easy detection. Such patients
can have difficulty describing their visual complaint, but the nature of the
problem often becomes apparent if a more detailed history is obtained.
There might be subtle examination findings that suggest a disorder of higher
visual function, such as difficulty reading despite normal or near-normal
visual acuity.
A variety of disorders of higher visual function have been described
(summarized in Table 10-1). Visual agnosia, a common disorder of higher
visual function, is characterized by an inability to recognize familiar objects
despite intact visual perception, attention, intellect, and language function.
Observation of the patient described in the case scenario above has revealed
that he might have a subtype of visual agnosia called prosopagnosia, in
which the ability to recognize familiar faces is impaired. Prosopagnosia can
be detected clinically by asking the patient to identify relatives or famous
public figures (e.g., politicians, actors, or sporting stars) from portrait pho-
tographs. Affected patients can have coexisting homonymous visual field
defects (see case 9), cerebral achromatopsia (inability to perceive colors),
and impairment of visual memory. Prosopagnosia usually arises because of
bilateral lesions involving the inferior temporo-occipital junction, most
commonly due to infarction in the posterior cerebral artery territory.
However, it can also occur with a more diffuse process, such as viral enceph-
alitis, or in neurodegenerative diseases, such as Alzheimer’s disease.
Simultanagnosia, another common disorder of higher visual function, is
characterized by difficulty interpreting an entire visual scene despite the
retained ability to interpret components of the scene. It should be suspected
when a patient has difficulty identifying the control Ishihara color plate
despite having normal visual acuity. Simultanagnosia can be more formally
assessed by asking the patient to describe a complex scene (e.g., the “cookie
theft” picture). It most commonly occurs in patients with neurodegenera-
tive diseases, such as Alzheimer’s disease.

10. DISORDERS OF HIGHER VISUAL FUNCTION 53


TABLE 10-1 Disorders of Higher Visual Function

Deficit Description Lesion localization

Alexia Acquired inability to read Dominant medial


temporo-occipital

Cerebral Acquired inability to perceive Ventral-medial


achromatopsia colors occipital (V4)

Cerebral akinetopsia Acquired inability to perceive Bilateral lateral


motion temporo-occipital

Cortical blindness Bilateral homonymous Bilateral primary


visual loss visual cortex (V1)

Ocular motor apraxia Inability to move the eyes Bilateral parieto-


(can be a component toward an object of interest occipital
of Balint’s syndrome) despite normal ductions Bilateral frontal

Optic ataxia Impaired visually guided limb Bilateral parieto-


(can be a component movements despite intact occipital
of Balint’s syndrome) motor function

Palinopsia Abnormally persistent visual Nondominant


afterimages parieto-occipital

Prosopagnosia Inability to recognize familiar Bilateral inferior


faces temporo-occipital

Simultanagnosia Inability to interpret an entire Bilateral parieto-


(can be a component visual scene but retained ability occipital
of Balint’s syndrome) to interpret components of the
scene

Visual hallucinations Visual perceptions in the Primary visual cortex


absence of external sensory (V1) for simple
stimulation (elementary) visual
hallucinations

Patients with disorders of higher visual function due to focal lesions (e.g.,
stroke or tumor) can have other neurologic symptoms or signs that help to
localize the lesion; MRI of the brain will usually demonstrate the causative
lesion. Those patients with disorders of higher visual function due to neu-
rodegenerative disease will often have other cognitive deficits (e.g., deficits
in short-term memory) that can be detected on screening tests to evaluate

54 WHAT DO I DO NOW? NEURO-OPHTHALMOLOGY


higher cortical function (e.g., Mini-Mental State Examination). However,
in the visual (or posterior) variant of Alzheimer’s disease (VVAD), in which
the visual association areas are preferentially affected, patients have only
visual complaints initially. Visual–spatial difficulties (e.g., getting lost in
familiar places) are particularly characteristic of VVAD. Since other cogni-
tive domains are intact, patients with VVAD often present to ophthalmolo-
gists rather than neurologists and pose a diagnostic challenge because the
routine ophthalmic examination is unrevealing. Brain MRI typically shows
focal parieto-occipital or temporo-occipital atrophy (see Figure 10-1), which
might not be appreciated unless specifically sought. Thus, the first step in the
evaluation of our patient is to review his previous imaging. Positron emission
tomography (PET) often reveals parieto-occipital hypometabolism, which
supports a diagnosis of VVAD. However, other treatable causes of dementia
should be excluded with laboratory testing (e.g., B12 level, thyroid function
tests, and syphilis serology).
Treatment for disorders of higher visual function should be directed
toward the underlying cause. Neurology evaluation should be arranged for
patients with VVAD, because they might experience some improvement
with standard treatments for Alzheimer’s disease (e.g., donepezil).

FIGURE 10-1 Magnetic resonance imaging (MRI) demonstrating parieto-occipital atrophy in


a patient with the visual (posterior) variant of Alzheimer’s disease.

10. DISORDERS OF HIGHER VISUAL FUNCTION 55


K EY P O I N TS TO R E M E M B E R

■ A disorder of higher visual function should be considered in


patients who have visual complaints that are out of proportion to
the findings on ophthalmic examination.
■ VVAD often presents with isolated visual symptoms, because
other cognitive functions are intact.
■ Patients with VVAD often have focal parieto-occipital or
temporo-occipital atrophy on MRI.
■ Some patients with VVAD may report temporary improvement in
symptoms with standard treatments for Alzheimer’s disease.

Further Reading
Barton JJ. Disorders of face perception and recognition. Neurol Clin. 2003;21:521–548.
Brazis PW, Graff-Radford NR, Newman NJ, Lee AG. Ishihara color plates as a test for
simultanagnosia. Am J Ophthalmol. 1998;126:850–851.
Lee AG, Martin CO. Neuro-ophthalmic findings in the visual variant of Alzheimer’s disease.
Ophthalmology. 2004;111:376–380.
Rizzo M, Anderson SW, Dawson J, Nawrot M. Vision and cognition in Alzheimer’s disease.
Neuropsychologia. 2000;38:1157–1169.

56 WHAT DO I DO NOW? NEURO-OPHTHALMOLOGY


11 Transient Visual Loss

A 60-year-old man presents to your clinic following two


episodes of vision loss in his right eye. Both episodes
began suddenly, lasted for about 5 minutes, and then
resolved spontaneously. He had no other symptoms
during the episodes. Examination reveals visual acuities
of 20/20 in both eyes. Color vision, confrontation visual
fields, pupils, and eye movements are normal. Undilated
funduscopic examination is unremarkable.

What do you do now?

57
T ransient visual loss (TVL) is an abrupt temporary monocular or bin-
ocular loss of vision that often results from reduced blood supply to the
afferent visual system. Common causes of TVL include primary arterial
occlusion or stenosis (e.g., atheroma), secondary arterial occlusion due to
embolism from a distant site (e.g., carotid artery, aortic arch, or heart),
vasospasm (e.g., migraine), or systemic hypoperfusion (e.g., cardiac arrhyth-
mia, hypotension, or hyperviscosity) (see Table 11-1). Many less common
causes for TVL are recognized (see Table 11-1).
The first step in the evaluation of this patient with TVL is to obtain a
careful history. The history is often crucial for making the diagnosis, because
physical examination and investigations may be unrevealing. It is important
to establish that there was TVL rather than visual blurring, which is often
due to a benign ophthalmic problem (e.g., tear film dysfunction). It is also
important to determine if the TVL was monocular or binocular; monocular
TVL suggests a prechiasmal lesion, whereas binocular TVL suggests a chi-
asmal lesion, retrochiasmal lesion, or bilateral prechiasmal lesions. It is
essential to ask patients if they checked for vision loss in both eyes by cover-
ing each eye in turn during the episode, because homonymous visual field
loss is often mistaken for monocular vision loss on the side with the tempo-
ral field defect. Although TVL often occurs spontaneously, the presence of
a precipitating factor can help to determine the cause. For example, TVL
can be precipitated by postural changes in systemic hypotension, papille-
dema, and giant cell arteritis (GCA). A description of the pattern of visual
loss might also suggest the etiology; an altitudinal onset (“like a curtain
descending over my vision”) suggests embolic arterial occlusion, whereas a
concentric onset might indicate vasospasm or a neurologic cause. The dura-
tion of TVL sometimes suggests a specific cause: TVL from papilledema
and optic nerve head drusen usually lasts for seconds, TVL from retinal
emboli or transient ischemic attacks usually lasts for less than 15 minutes,
and TVL from migraine usually lasts for more than 15 minutes. Many
causes of TVL produce other symptoms during the attack, such as head-
ache, positive visual phenomena (e.g., flashes), and focal neurologic symp-
toms (e.g., sensory or speech disturbance), or the patient may have noticed
other symptoms before or after the attack that suggest a certain etiology.
Patients should also be asked about other symptoms that they may not

58 WHAT DO I DO NOW? NEURO-OPHTHALMOLOGY


TABLE 11-1 Causes of Transient Visual Lossa

Monocular visual loss Binocular visual loss

Vascular Carotid artery stenosis, occlusion, or Transient ischemic


dissection attack
Ophthalmic artery stenosis, occlusion, or Bilateral carotid
dissection artery stenosis or
Aortic arch atheroma occlusion
Cardioembolic source (arrhythmia, Systemic
structural defect) hypoperfusion
Venous source (paradoxical embolism)
Vasculitis (giant cell arteritis)
Arterial vasospasm
Hypercoagulable state
Systemic hypoperfusion

Neurologic Retinal migraine Migraine aura


Elicited repetitive daily blindness Occipital seizure
Posterior reversible
encephalopathy
syndrome
Exposure to
angiographic
contrast media
Head trauma

Ophthalmic Papilledema and optic disc edema Papilledema and


Optic nerve head drusen optic disc edema
Optic neuritis (Uhthoff phenomenon) Optic nerve head
Orbital masses and foreign bodies drusen
Age-related macular degeneration Age-related macular
Intermittent angle-closure glaucoma degeneration
Corneal basement membrane dystrophy
Tear film dysfunction and dry eye

a
Adapted from Thurtell MJ, Rucker JC. Transient visual loss. Int Ophthalmol Clin. 2009;49:148.

volunteer or think relevant (e.g., symptoms suggestive of GCA; see case 2).
A history of vascular risk factors, cardiovascular disease, and migraine should
also be sought. A history of ophthalmic disease, polymyalgia rheumatica,
connective tissue disease, or hematologic disease (e.g., hypercoagulable state)
might be relevant in some patients.

11. TRANSIENT VISUAL LOSS 59


Physical examination is often unrevealing in patients with TVL.
Nevertheless, a thorough examination is required to assess the state of the
afferent visual system (e.g., the patient may have a persistent asymptomatic
visual field defect; see Figure 11-1) and to detect signs that suggest a specific
etiology for the TVL. A thorough ophthalmic examination, including a
dilated funduscopic examination, is essential, because many ophthalmic
conditions can cause TVL (see Table 11-1) or there may be signs indicating
the likely etiology (e.g., retinal emboli; see Figure 11-1). The pulse and
blood pressure should be assessed, and the chest and neck should be auscul-
tated for heart murmurs and carotid bruits, respectively.
Investigations are often required when a vascular cause of TVL is sus-
pected. In our patient with several episodes of short-lived painless monocu-
lar TVL, secondary arterial occlusion due to embolism must be excluded.
Possible sources of embolism include the carotid arteries, aortic arch, and
heart. Because both episodes occurred in the right eye, carotid Doppler
ultrasound, magnetic resonance angiography (MRA) of the head and neck
vessels, or computed tomographic angiography (CTA) of the head and neck
vessels should be obtained to assess for a stenosis in the vascular supply of
the right eye. Echocardiography should be obtained to assess for structural

FIGURE 11-1 Calcific embolus (left) causing a branch retinal artery occlusion with a superior
altitudinal visual field defect (right) in a patient who complained of transient monocular
visual loss.

60 WHAT DO I DO NOW? NEURO-OPHTHALMOLOGY


cardiac abnormalities and aortic arch atheroma. Electrocardiographic mon-
itoring should be considered to screen for paroxysmal arrhythmias (e.g.,
atrial fibrillation).
The management of TVL depends on its etiology. In patients with a
cardiac source of embolism, anticoagulation with warfarin and treatment of
the underlying cardiac disease is indicated. In those with a vascular stenosis,
antiplatelet therapy with aspirin should be initiated. The management of
high-grade internal carotid artery stenosis (70%–99%) in patients with iso-
lated TVL remains controversial, because the risk for stroke is lower in
patients with isolated TVL than in those with hemispheric transient isch-
emic attacks. Findings from the North American Symptomatic Carotid
Endarterectomy Trial (Benavente et al., 2001) suggest that patients need
three or more of the following to benefit from carotid endarterectomy: male
sex; age of 75 years or more; history of hemispheric transient ischemic
attack or stroke; history of intermittent leg claudication; stenosis of 80%–
94%; or absence of collaterals on angiography. Carotid artery stenting has
been proposed as an alternative to endarterectomy but might result in a
higher risk of periprocedural stroke.

K EY P O I N TS TO R E M E M B E R

■ TVL is an abrupt temporary monocular or binocular loss of vision.


■ TVL is often caused by primary arterial occlusion or stenosis,
secondary arterial occlusion due to embolism from a distant site,
vasospasm, or systemic hypoperfusion.
■ The history of the event is often crucial for determining the etiology
of TVL, because physical examination and investigations can be
unrevealing.
■ Patients with TVL that might be due to embolism require urgent
investigations and treatment that is directed toward the underlying
etiology.

Further Reading
Benavente O, Eliasziw M, Streifler JY, et al. Prognosis after transient monocular blindness
associated with carotid-artery stenosis. N Engl J Med. 2001;345:1084–1090.

11. TRANSIENT VISUAL LOSS 61


Donders RC. Clinical features of transient monocular blindness and the likelihood of
atherosclerotic lesions of the internal carotid artery. J Neurol Neurosurg Psychiatry.
2001;71:247–249.
Silver FL, Mackey A, Clark WM, et al. Safety of stenting and endarterectomy by
symptomatic status in the Carotid Revascularization Endarterectomy Versus
Stenting Trial (CREST). Stroke. 2011;42:675–680.
Thurtell MJ, Rucker JC. Transient visual loss. Int Ophthalmol Clin. 2009;49:147–166.

62 WHAT DO I DO NOW? NEURO-OPHTHALMOLOGY


12 Migraine Aura

A 60-year-old woman presents to your clinic following


several recent episodes during which she has seen
jagged silver lines in the left half of her visual field. The
episodes last for about 20 minutes. She denies having
any other symptoms before, during, or after these
episodes. There is no personal or family history of
migraine. Examination reveals visual acuities of 20/20 in
both eyes. She correctly identifies all Ishihara color
plates with both eyes. Her confrontation visual fields are
full. Pupil, eye movement, and funduscopic examinations
are normal.

What do you do now?

63
M igraine aura is a common cause of transient visual disturbance (see
case 11). The classic visual aura is the fortification spectrum, in which
an achromatic (e.g., silver) figure with a scintillating zigzag edge appears
near the center of the visual field of both eyes and gradually expands toward
the periphery over minutes, leaving a bean-shaped scotoma in its wake. The
aura usually lasts for about 15 minutes, but rarely for more than an hour. In
classic migraine, the aura is followed by a severe unilateral throbbing head-
ache, often with associated photophobia, phonophobia, nausea, or vomit-
ing. The diagnosis is clinical, in accordance with International Headache
Society (IHS) diagnostic criteria, and further investigations are usually not
necessary.
Occasionally, a patient will report having a typical visual aura without a
subsequent migraine headache. Such patients usually have a history of
migraine with visual aura, but the headaches have either lost their migraine
characteristics or have completely remitted with time. A dilemma arises
when an older patient without a history of migraine has a visual aura with-
out headache, as in this case scenario. Even if the patient describes a typical
visual aura and has an unremarkable examination, including formal visual
field testing, investigations should be obtained to exclude an occipital lesion
(e.g., arteriovenous malformation; see Figure 12-1) or vertebrobasilar isch-
emia. Likewise, in cases where the visual aura is always lateralized to the
same side, as in this patient, imaging is essential to exclude an occipital
lesion, even if there is no visual field defect on formal visual field testing.
Migraine aura should be distinguished from retinal migraine. The IHS
diagnostic criteria for retinal migraine require at least two attacks of fully
reversible monocular positive and/or negative visual phenomena associated
with a headache that meets diagnostic criteria for migraine without aura.
Migraine aura should also be distinguished from the positive visual phe-
nomena occurring with occipital seizures (see Table 12-1). Occipital sei-
zures usually produce a sudden onset of binocular positive visual phenomena
that, in contrast with migraine, consist of multiple, brightly colored,
small circular spots or balls that are usually located in the contralateral visual
field. They can increase in size and multiply in number over the course of
the episode. They can also flash, move horizontally across the visual field,
spin, or rotate. Vision is obscured in the area occupied by the visual phe-
nomena from the onset. The positive visual phenomena usually last for less

64 WHAT DO I DO NOW? NEURO-OPHTHALMOLOGY


FIGURE 12-1 T2-weighted magnetic resonance imaging (left) and magnetic resonance
angiography (right) demonstrating a left occipital arteriovenous malformation that caused
episodic right-sided positive visual phenomena reminiscent of migraine visual aura.

TABLE 12-1 Positive Visual Phenomena in Migraine Aura Compared with


Occipital Seizures

Characteristic Migraine visual aura Occipital seizure

Onset Sudden Sudden

Progression Slow over minutes Rapid over seconds

Duration <60 minutes, typically <1 minute, typically seconds


15–20 minutes

Shape Angular, zigzag Spots, circles, balls

Color Black and white or silver Bright (often primary) colors

Vision loss Bean-like scotoma following Hemianopia during episode


scintillating edge and (variably) after episode

Associated Headache Eye–head deviation


symptoms Photophobia Eyelid fluttering
Phonophobia Altered level of consciousness
Nausea and vomiting Postictal headache

12. MIGRAINE AURA 65


BOX 1 2-1 CAUS ES O F O CC I P I TA L S E I Z U RES a

Posterior reversible encephalopathy syndrome (e.g., malignant


hypertension)
Metabolic encephalopathy (e.g., hypercalcemia)
Malformations of cortical development (e.g., cortical dysplasia)
Tumors (e.g., neuroepithelial tumors, astrocytoma)
Vascular lesions (e.g., arteriovenous malformations)
Prior head trauma (e.g., posttraumatic occipital gliosis)
Metabolic diseases (e.g., mitochondrial disorders)
Localized infections (e.g., bacterial abscesses)
Idiopathic
a
Adapted from Taylor I, Scheffer IE, Berkovic SF. Occipital epilepsies: identification
of specific and newly recognized syndromes. Brain. 2003;126:756.

than a minute. However, other ictal phenomena can develop with seizure
propagation (e.g., eye–head deviation, eyelid fluttering, or altered level of
consciousness), and a headache indistinguishable from migraine commonly
occurs following the event. Occipital seizures can occur in a variety of con-
ditions (see Box 12-1).
The treatment for migraine involves avoidance of precipitating factors
and the use of abortive treatments, such as triptan medications, at the onset
of attacks. Patients with frequent or disabling attacks should be offered pro-
phylactic therapies, such as propranolol, amitriptyline, or topiramate.
However, if the patient is reporting only occasional, short-lived visual aura
without subsequent headache, medical treatment might not be required or
requested by the patient.

K EY P O I N TS TO R E M E M B E R

■ Migraine visual aura is characterized by an achromatic scintillating


zigzag figure that gradually progresses across half of the visual
field, leaving a bean-like scotoma in its wake.
■ Migraine visual aura is usually followed by a severe unilateral
throbbing headache.

66 WHAT DO I DO NOW? NEURO-OPHTHALMOLOGY


■ Migraine visual aura is a clinical diagnosis and does not usually
require further investigation other than formal visual field testing.
■ Patients should be investigated with imaging if the aura is always
lateralized to the same side or if there is no associated headache in
an older patient who has no history of migraine.
■ Migraine visual aura should be differentiated from retinal migraine
and occipital seizures.

Further Reading
Hill DL, Daroff RB, Ducros A, Newman NJ, Biousse V. Most cases labeled as “retinal
migraine” are not migraine. J Neuroophthalmol. 2007;27:3–8.
Panayiotopoulos CP. Elementary visual hallucinations, blindness, and headache in
idiopathic occipital epilepsy: differentiation from migraine. J Neurol Neurosurg
Psychiatry. 1999;66:536–540.
Russell MB, Olesen J. A nosographic analysis of the migraine aura in a general population.
Brain. 1996;119:355–361.
Shams PN, Plant GT. Migraine-like visual aura due to focal cerebral lesions: case series
and review. Surv Ophthalmol. 2011;56:35–161.
Taylor I, Scheffer IE, Berkovic SF. Occipital epilepsies: identification of specific and newly
recognized syndromes. Brain. 2003;126:753–769.

12. MIGRAINE AURA 67


13 Nonorganic Vision Loss

A 40-year-old man presents to your clinic complaining


of decreased vision in his left eye following a minor
work injury 1 year ago. He is now seeking workers’
compensation. Examination reveals visual acuities of
20/20 in the right eye and 20/400 in the left eye. He
correctly identifies all Ishihara color plates with his right
eye but cannot see the control plate with his left eye.
Confrontation visual fields are full in the right eye and
constricted in the left eye. His pupils are equal and
briskly reactive to light. There is no relative afferent
pupillary defect. Funduscopic examination is normal.
Magnetic resonance imaging (MRI) of the orbits with
contrast is normal.

What do you do now?

68
N onorganic vision loss is very common but is often challenging to diag-
nose and treat. When one is confronted with a patient in whom non-
organic vision loss is suspected, the main goal of evaluation is to exclude an
organic disorder and to demonstrate that visual function is intact or better
than reported. The evaluation begins with a comprehensive history; there
may be aspects of the history that suggest nonorganic disease (e.g., highly
positive review of systems, disability or workers’ compensation claim, or
impending litigation). A careful ophthalmic examination is the next step,
because patients with nonorganic vision loss can have coexisting organic
disease. In patients reporting monocular vision loss, there should be a care-
ful inspection of the cornea, ocular media, and retina on the affected side,
keeping in mind that the causative lesion may not be obvious (e.g., in kera-
toconus). There should also be careful evaluation for signs suggesting a uni-
lateral optic neuropathy (e.g., relative afferent pupillary defect or optic disc
pallor). If any signs of organic disease are detected, further investigations
should be pursued as appropriate.
The maneuvers that can be used to demonstrate the presence of intact
visual function vary depending on the claimed deficit and its severity.
When moderate monocular vision loss is reported, as in our patient, the
first step is to demonstrate that visual acuity is better than reported. In a
technique known as fogging, the patient undergoes a “refraction” such that
the unaffected eye is fogged with a high-plus spectacle lens while the
patient’s usual correction is placed in front of the affected eye. If the patient
can see optotypes on the 20/20 line with both eyes opened and the unaf-
fected eye fogged, this proves normal visual acuity in the affected eye.
Evaluation of stereopsis (depth perception) can also be helpful when mon-
ocular nonorganic vision loss is suspected, because stereopsis correlates with
the visual acuity of the two eyes. If our patient’s stereopsis is 40 seconds
of arc, this proves normal visual acuity (20/20) in both eyes. When
severe monocular vision loss is reported (e.g., no light perception), the
presence of optokinetic nystagmus while the patient is viewing a rotating
optokinetic drum with only the affected eye indicates a visual acuity of at
least 20/400 in that eye. In patients who do not show optokinetic nystag-
mus, a large mirror can be oscillated in front of the eyes instead; some
degree of visual function can be assumed if smooth pursuit movements are

13. NONORGANIC VISION LOSS 69


elicited when the patient is viewing the oscillating mirror with only the
affected eye.
Several maneuvers can be helpful in the assessment of suspected nonor-
ganic visual field loss. When the visual field is constricted, it is useful to use
a tangent screen (or the examination room wall) to assess for expansion of
the visual field with increasing distance from the screen. Individuals with
normal vision and patients with organic visual field constriction will show
expansion in the area of the visual field when the distance to the screen is
increased. In contrast, those with nonorganic visual field constriction will
often not show expansion of the visual field. Goldmann perimetry can also
be helpful in the evaluation of patients with suspected nonorganic visual
field constriction and may demonstrate crossing or spiraling isopters. A
nonorganic monocular visual field defect (e.g., a monocular hemianopia)
often persists on perimetry that is performed with both eyes opened.
The management of nonorganic vision loss is often difficult, especially if
there is a significant secondary gain for the patient (e.g., compensation
payout). Confronting the patient is rarely helpful. Rather, it is usually pref-
erable to reassure the patient that there is no evidence of permanent damage
to the visual system and that there is a good prognosis for visual recovery.
One should state the objective examination findings if requested to com-
plete workers’ compensation or disability forms. Attempts to address under-
lying motivating factors and reduce secondary gain are variably successful
and are best handled by a psychiatrist or psychologist.

K EY P O I N TS TO R E M E M B E R

■ Nonorganic vision loss is very common and can sometimes coexist


with organic disease.
■ The goal of evaluation is to exclude organic disease and
demonstrate that visual function is intact or better than reported.
■ Specific maneuvers can prove normal visual acuity and show
nonorganic field constriction.
■ The patient with nonorganic vision loss should be reassured that
there is no evidence of permanent damage to the visual system and
that there is a good prognosis for spontaneous visual recovery.

70 WHAT DO I DO NOW? NEURO-OPHTHALMOLOGY


Further Reading
Bengtzen R, Woodward M, Lynn MJ, Newman NJ, Biousse V. The “sunglasses sign”
predicts nonorganic visual loss in neuro-ophthalmologic practice. Neurology.
2008;70:218–221.
Levy NS, Glick EB. Stereoscopic perception and Snellen visual acuity. Am J Ophthalmol.
1974;78:722–724.
Miller NR. Neuro-ophthalmologic manifestations of nonorganic disease. In: Miller NR,
Newman NJ, Biousse V, Kerrison JB, eds. Walsh & Hoyt’s clinical neuro-
ophthalmology. 6th ed. Philadelphia, PA: Lippincott Williams & Wilkins; 2005:
1315–1334.
Scott JA, Egan RA. Prevalence of organic neuro-ophthalmologic disease in patients with
functional visual loss. Am J Ophthalmol. 2003;135:670–675.

13. NONORGANIC VISION LOSS 71


This page intentionally left blank
SECTION II

EFFERENT DISORDERS
This page intentionally left blank
14 Third Nerve Palsy

You are called to see a 46-year-old man who has


presented to the emergency department following the
acute onset of headache, left-sided ptosis, and binocular
diagonal diplopia. He has poorly controlled type 2
diabetes. Examination reveals normal distance visual
acuities, color vision, and confrontation visual fields.
There is a mid-dilated unreactive left pupil and partial
left-sided ptosis. The left eye is “down and out.” There is
limited elevation, depression, and adduction of the left
eye. Funduscopic examination is normal. Computed
tomography (CT) of the brain is normal.

What do you do now?

75
T he third cranial nerve innervates the iris sphincter, the levator palpe-
brae superioris muscle, and all of the extraocular muscles apart from
the lateral rectus and superior oblique muscles. A complete third cranial
nerve palsy therefore produces a dilated and unreactive pupil, complete
ptosis, and an eye that is exotropic and hypotropic (“down and out”) with
limited elevation, depression, and adduction. A third cranial nerve palsy
can be incomplete, where some muscles are spared (e.g., pupil-sparing third
nerve palsy), or partial, where all muscles are involved but are not totally
paretic. The palsy can be isolated, with or without associated pain, or it can
be associated with other neurologic symptoms and signs if there is involve-
ment of adjacent neural structures. A careful history and examination, to
determine if the third nerve palsy is isolated or associated with other neuro-
logic deficits, is the first step in the evaluation of our patient with a suspected
third nerve palsy. The presence of other neurologic symptoms and signs helps
to localize the causative lesion. For example, the presence of contralateral
hemiparesis, ataxia, or tremor suggests a midbrain lesion, whereas the pres-
ence of other ipsilateral cranial nerve palsies suggests a lesion involving the
meninges, cavernous sinus, superior orbital fissure, or orbital apex.
Common causes of isolated third nerve palsy include trauma, micro-
vascular ischemia, compression by neoplasm, and compression by aneu-
rysm (see Figure 14-1). The tempo of onset may suggest a particular etiology:
acute onset suggests microvascular ischemia; subacute onset suggests com-
pression by a rapidly enlarging aneurysm; and slowly progressive onset sug-
gests compression by a slowly enlarging neoplasm or aneurysm. Pain is
common in patients with third nerve palsy caused by aneurysmal compres-
sion or microvascular ischemia. Signs of aberrant regeneration (e.g., inap-
propriate elevation of the upper eyelid with adduction or depression of the
eye) should be specifically sought, because these suggest slowly progressive
third nerve compression with misdirected regrowth of nerve fibers. The pupil
should also be carefully assessed. While the absence of pupil involvement
suggests that a third nerve palsy is not due to compression, it is not a guar-
antee. However, other conditions that can mimic third nerve palsy (e.g.,
ocular myasthenia) should be considered when the pupil is not involved.
In this case, there has been an acute onset of isolated, painful, partial pupil-
involving third nerve palsy in a patient with poorly controlled diabetes.
While the cause could be microvascular ischemia, this presentation is a

76 WHAT DO I DO NOW? NEURO-OPHTHALMOLOGY


FIGURE 14-1 Aneurysm of the left posterior communicating artery (arrows) on magnetic
resonance angiography (MRA) source image (left) and MRA reconstructions (middle and
right) in a patient with a painful pupil-involving left third nerve palsy.

medical emergency with a potentially catastrophic outcome, because a rap-


idly enlarging aneurysm has not yet been excluded.
The investigation of acute pupil-involving third nerve palsy has evolved
in recent years. In the past, catheter angiography was necessary for identify-
ing and excluding aneurysm. Recent improvements in the resolution of CT
angiography (CTA) and magnetic resonance angiography (MRA) have
made noninvasive detection of aneurysms possible with a sensitivity
approaching that of catheter angiography. CTA is faster, more widely avail-
able, and more reliable than MRA for detecting aneurysms that measure at
least 4 mm in diameter. Thus, CTA is the initial investigation of choice in
patients with acute-onset isolated nontraumatic third nerve palsy, except if
x-ray or iodine dye exposure is contraindicated (e.g., in pregnant women,
children, and patients with renal or cardiac failure). CT of the brain should
be obtained at the same time, because it might demonstrate subarachnoid
hemorrhage. If CTA does not reveal an aneurysm, magnetic resonance
imaging (MRI) of the brain should be performed looking for another cause
for the third nerve palsy. If the suspicion for aneurysm remains high, MRA
should be performed at the same time. MRA is the investigation of choice
in patients with isolated third nerve palsy who have a contraindication to
CTA (see Figure 14-1). Catheter angiography can be considered to defini-
tively exclude aneurysm but is rarely necessary if there is no evidence of
aneurysm on CTA or MRA.
The management of our patient’s isolated third nerve palsy depends on
the cause. If an aneurysm is detected, urgent referral to neurosurgery or
interventional neuroradiology is required, because there is a high risk of

14. THIRD NERVE PALSY 77


imminent rupture. Definitive treatment of the aneurysm by clipping or
coiling is required as soon as possible, given the high mortality and morbid-
ity of subarachnoid hemorrhage. If there is no evidence of aneurysm or
another cause for third nerve palsy on imaging, microvascular ischemia is
the likely etiology, and conservative management is appropriate. Although
most patients with a microvascular third nerve palsy will completely recover
within 3–6 months, vascular risk factors should be addressed. Long-term
antiplatelet therapy should also be considered to reduce the risk of future
vascular events.

K EY P O I N TS TO R E M E M B E R

■ Complete third nerve palsy produces a dilated and unreactive pupil,


complete ptosis, and a “down and out” eye with limited elevation,
depression, and adduction.
■ Third nerve palsy is often caused by trauma, microvascular
ischemia, compression by neoplasm, or compression by aneurysm.
■ Patients with acute-onset isolated non-traumatic third nerve palsy
should undergo urgent CTA or MRA to exclude aneurysm.
■ Patients with acute-onset third nerve palsy due to compression by
aneurysm require urgent referral to neurosurgery or neuroradiology,
because there is a high risk of imminent rupture.
■ Microvascular third nerve palsy typically recovers completely within
3–6 months.

Further Reading
Chaudhary N, Davagnanam I, Ansari SA, Pandey A, Thompson BG, Gemmete JJ.
Imaging of intracranial aneurysms causing isolated third cranial nerve palsy.
J Neuroophthalmol. 2009;29:238–244.
Elmalem VI, Hudgins PA, Bruce BB, Newman NJ, Biousse V. Underdiagnosis of posterior
communicating artery aneurysm in noninvasive brain vascular studies.
J Neuroophthalmol. 2011;31:103–109.
Jacobson DM. Pupil involvement in patients with diabetes-associated oculomotor nerve
palsy. Arch Ophthalmol. 1998;116:723–727.
Keane JR. Third nerve palsy: analysis of 1400 personally-examined inpatients.
Can J Neurol Sci. 2010;37:662–670.
Trobe JD. Searching for brain aneurysm in third cranial nerve palsy. J Neuroophthalmol.
2009;29:171–173.

78 WHAT DO I DO NOW? NEURO-OPHTHALMOLOGY


15 Fourth Nerve Palsy

A 48-year-old man presents to your clinic with a several-


month history of progressively increasing binocular
vertical diplopia that is exacerbated by reading. He is
otherwise well. Examination reveals normal visual
acuities, color vision, confrontation visual fields, pupils,
and eyelids. His ductions appear full, but there is a right
hypertropia that increases with leftward gaze and
rightward head tilt. He has a spontaneous leftward head
tilt. Funduscopic examination reveals extorsion of the
right fundus.

What do you do now?

79
T he history and clinical findings in this case scenario suggest a fourth
nerve palsy. Fourth nerve palsy results in weakness of the superior
oblique muscle, which normally depresses and intorts the eye. Superior
oblique weakness produces binocular vertical diplopia that is often worse
with reading, although some patients also complain of torsional diplopia.
A detailed history often helps to pinpoint the cause. In particular, the tempo
of onset can suggest a particular etiology: sudden onset with or without
pain suggests a microvascular etiology, whereas gradual onset can suggest
decompensation of a long-standing fourth nerve palsy. It is important to ask
about head trauma, because this is a frequent cause of isolated fourth nerve
palsy. Vascular risk factors, such as hypertension and diabetes, might sug-
gest a microvascular etiology in acute onset of isolated fourth nerve palsy. It
is also worthwhile to ask about other neurologic symptoms, which can sug-
gest a location for the lesion, and about symptoms of giant cell arteritis in
elderly patients.
It is often difficult to diagnose a fourth nerve palsy on physical examina-
tion, because the clinical signs can be very subtle. A head tilt to one side is
a valuable clue to the presence of a fourth nerve palsy on the other side. If a
head tilt is present, it is important to ask if it is long-standing and to inspect
old photographs (e.g., on a driver’s license or Facebook profile). If a fourth
nerve palsy is diagnosed, a long-standing head tilt suggests that it is either
congenital or long-standing from another cause. Ocular motor examination
will reveal vertical misalignment of the eyes, with one eye being hypertropic
(elevated) relative to the other. When a hypertropia is detected, the Parks-
Bielschowsky three-step test should be used to identify the paretic muscle
(see Table 15-1). The first step of the test is to determine the side of the
hypertropia; the second step is to determine if the hypertropia increases
with adduction or abduction; and the final step is to determine if the hyper-
tropia increases with head tilt to the ipsilateral or contralateral side. In a
patient with right hypertropia due to a right fourth nerve palsy, the vertical
misalignment would be greatest with leftward gaze (adduction) and right-
ward head tilt (see Table 15-1). When the deviation is subtle, it can be help-
ful to quantify it at each stage of the test using prisms with a Maddox rod
or red glass. Prisms can also be used to determine the vertical fusional
amplitude; when it is increased by more than 10 prism diopters, long-
standing fourth nerve palsy is likely. Extorsion of the affected eye may be

80 WHAT DO I DO NOW? NEURO-OPHTHALMOLOGY


TABLE 15-1 Results of the Parks-Bielschowsky Three-Step Test for Individual
Muscle Palsies Causing a Right Hypertropia (RHT)

Step 1 Step 2 Step 3 Affected muscle

RHT RHT ↑ with adduction RHT ↑ with right head tilt Right superior
oblique

RHT RHT ↑ with abduction RHT ↑ with right head tilt Left inferior
oblique

RHT RHT ↑ with adduction RHT ↑ with left head tilt Left superior
rectus

RHT RHT ↑ with abduction RHT ↑ with left head tilt Right inferior
rectus

seen on funduscopic examination, as in our patient, or demonstrated and


quantified with double Maddox rods.
Isolated fourth nerve palsies are often congenital, posttraumatic, or
microvascular in etiology. However, with improved magnetic resonance
imaging (MRI) resolution, fourth nerve schwannomas are becoming an
increasingly recognized cause for long-standing and progressive fourth
nerve palsies. These benign lesions can be identified on MRI as enhancing
areas along the course of the fourth nerve, typically in the ambient or per-
imesencephalic cistern (see Figure 15-1), and usually do not cause any other
deficit. Rarely, fourth nerve palsy can be caused by an intrinsic brainstem
lesion, in which case there may be an associated contralateral internuclear
ophthalmoplegia, Horner’s syndrome, or upbeat nystagmus due to involve-
ment of adjacent neural pathways. The presence of other cranial nerve pal-
sies ipsilateral to a fourth nerve palsy suggests a lesion involving the
meninges, cavernous sinus, superior orbital fissure, or orbital apex.
Alternative causes of vertical misalignment should also be considered,
especially when the findings of a Parks-Bielschowsky three-step test are
not consistent with superior oblique weakness. The presence of areflexia or
ataxia suggests Miller Fisher syndrome. Fluctuating signs (e.g., ptosis) sug-
gest ocular myasthenia. Proptosis and lid retraction suggest thyroid eye
disease. Brainstem signs or intorsion of the hypertropic eye suggest a skew
deviation rather than a fourth nerve palsy. However, a skew deviation can

15. FOURTH NERVE PALSY 81


FIGURE 15-1 Magnetic resonance imaging (MRI) showing a left fourth nerve schwannoma
(arrows).

occasionally give findings on the three-step test that are suggestive of supe-
rior oblique weakness. In these cases, it is useful to lay the patient flat,
because the vertical misalignment from a skew deviation often disappears
when the patient is supine, whereas the vertical misalignment from a fourth
nerve palsy remains unchanged.
The investigation of isolated fourth nerve palsy is controversial. Many
clinicians would not perform neuroimaging for patients with suspected
congenital fourth nerve palsy, isolated fourth nerve palsy due to head
trauma, or isolated acute-onset fourth nerve palsy in association with vascu-
lar risk factors. However, there is evidence from one recent prospective
study that a significant percentage of such patients have a sinister cause for
their fourth nerve palsy that would have been missed without neuroimag-
ing (Chou et al., 2004). Thus, neuroimaging should probably be performed
in most patients with a progressive-onset fourth nerve palsy, including
our patient, and in those with a presumed microvascular etiology who do
not recover within 3 months, although the imaging will rarely be abnormal
if there are no other neurologic signs. An extensive laboratory workup is
usually not fruitful, although inflammatory markers should be checked in
patients with a presumed microvascular fourth nerve palsy who are more
than 50 years old.
Treatment of fourth nerve palsy depends on the underlying cause. Most
patients with a presumed microvascular fourth nerve palsy will completely

82 WHAT DO I DO NOW? NEURO-OPHTHALMOLOGY


recover within 3 months. However, vascular risk factors should be addressed,
and antiplatelet therapy should be considered to reduce their risk of future
vascular events. In our patient, who likely has a long-standing fourth nerve
palsy, treatment of his diplopia depends on the magnitude of his vertical
misalignment. Prisms can be prescribed to alleviate diplopia when the verti-
cal misalignment is small. However, if the vertical misalignment is large,
long-standing, and stable, strabismus surgery would be the most appropri-
ate treatment to alleviate our patient’s diplopia.

K EY P O I N TS TO R E M E M B E R

■ Fourth nerve palsy causes a hypertropia that increases with


adduction and ipsilateral head tilt.
■ Most isolated fourth nerve palsies are congenital, posttraumatic,
or microvascular.
■ Neuroimaging should be performed in patients with progressive-
onset fourth nerve palsy or presumed microvascular fourth nerve
palsy that does not recover within 3 months.
■ Treatment of fourth nerve palsy depends on the etiology and degree
of ocular misalignment.

Further Reading
Chou KL, Galetta SL, Liu GT, et al. Acute ocular motor mononeuropathies: prospective
study of the roles of neuroimaging and clinical assessment. J Neurol Sci.
2004;219:35–39.
Elmalem VI, Younge BR, Biousse V, et al. Clinical course and prognosis of trochlear nerve
schwannomas. Ophthalmology. 2009;116:2011–2016.
Mollan SP, Edwards JH, Price A, Abbott J, Burdon MA. Aetiology and outcomes of adult
superior oblique palsies: a modern series. Eye (Lond). 2009;23:640–644.
Murchison AP, Gilbert ME, Savino PJ. Neuroimaging and acute ocular motor
mononeuropathies: a prospective study. Arch Ophthalmol. 2011;129:301–305.
Parulekar MV, Dai S, Buncic JR, Wong AM. Head position-dependent changes in ocular
torsion and vertical misalignment in skew deviation. Arch Ophthalmol.
2008;126:899–905.
Tamhankar MA, Kim JH, Ying GS, Volpe NJ. Adult hypertropia: a guide to diagnostic
evaluation based on review of 300 patients. Eye (Lond). 2011;25:91–96.

15. FOURTH NERVE PALSY 83


16 Sixth Nerve Palsy

A 52-year-old woman presents to the emergency


department after waking with binocular horizontal
diplopia and pain around her left eye. She has poorly
controlled diabetes and hypertension. She denies recent
head trauma. Examination reveals normal visual acuities,
color vision, confrontation visual fields, pupils, and
eyelids. Ocular motor examination reveals a large-angle
esotropia and a complete left abduction deficit.
Funduscopic examination reveals nonproliferative
diabetic retinopathy.

What do you do now?

84
T he sixth cranial nerve innervates the lateral rectus muscle, which abducts
the eye. Lateral rectus weakness produces binocular horizontal diplopia
that is worse when looking into the distance and when looking to the affected
side, with esotropia (crossed eyes) and an abduction deficit on the affected side.
It is a mistake, however, to assume that all patients with these symptoms and
deficits have a sixth nerve palsy. An abduction deficit can also be produced by
orbital disease (e.g., thyroid eye disease with medial rectus restriction or blow-
out fracture with medial rectus entrapment), by neuromuscular disease (e.g.,
ocular myasthenia or Miller Fisher syndrome), or by a central nervous system
lesion (e.g., fascicular lesion or “pseudoabducens” palsy). Clinical assessment
for orbital, neuromuscular, and brainstem disease is therefore the first step in
the evaluation of the patient described in this case scenario. If there are no
symptoms or signs to suggest any of these, a sixth nerve palsy is likely.
Common causes of sixth nerve palsy include microvascular ischemia,
trauma, multiple sclerosis, neoplasm, and stroke, although a cause cannot
be determined in as many as 25% of cases. The tempo of onset varies
depending on the etiology: sudden onset of sixth nerve palsy suggests a
microvascular etiology; subacute onset suggests demyelination; and slowly
progressive onset suggests compression (e.g., by a neoplasm). The presence
of associated symptoms also varies, with pain being present in the majority
of patients with microvascular sixth nerve palsy. There may be other neuro-
logic symptoms and signs if the sixth nerve palsy is caused by a lesion that
affects other neural structures. Sixth nerve palsies can be caused by intrinsic
brainstem lesions affecting the sixth nerve fasciculus, in which case there
may be associated ipsilateral facial weakness, contralateral hemiparesis, or
sensory symptoms due to involvement of adjacent neural pathways. The
presence of other cranial nerve palsies ipsilateral to a sixth nerve palsy sug-
gests a lesion involving the meninges, cavernous sinus, superior orbital fis-
sure, or orbital apex (see case 34). The presence of pulsatile tinnitus or
orbital signs in an older patient with a sixth nerve palsy should suggest an
arteriovenous fistula. In the case described, the sudden onset of diplopia
with associated pain in a patient with poorly controlled vascular risk factors
suggests a microvascular etiology.
In the past, investigation of suspected microvascular sixth nerve palsy with
neuroimaging was not considered necessary. Two prospective neuroimaging
studies have demonstrated that a significant proportion of patients with

16. SIXTH NERVE PALSY 85


isolated sixth nerve palsy, including those with presumed microvascular
ischemia, have a causative intracranial lesion on imaging (Bendszus et al.,
2001; Chou et al., 2004). It was therefore suggested that neuroimaging be
performed in all patients with isolated sixth nerve palsy, regardless of the
presumed etiology. However, the findings of a recent study suggest that
such an approach is not cost-effective and increases the chances of discover-
ing “incidentalomas” (Murchison, Gilbert, & Savino, 2011). No large, pro-
spective, multicenter study has been performed to evaluate the role of
neuroimaging in the investigation of isolated sixth nerve palsy. Our approach
is to image only those patients whose symptoms, signs, or course is atypical
for microvascular sixth nerve palsy. The patient in this case scenario has had
a typical presentation for microvascular sixth nerve palsy and has several
vascular risk factors. In the absence of any other symptoms, signs, or atypi-
cal features, it would be reasonable not to obtain neuroimaging in this
patient. However, if her subsequent course is not consistent with a micro-
vascular etiology (e.g., if there is ongoing progression or failure to recover
within 3–6 months), neuroimaging should be obtained.
Treatment of sixth nerve palsy depends on the underlying cause. Most
patients with a microvascular sixth nerve palsy will completely recover
within 3–6 months. Management of vascular risk factors should be opti-
mized, and antiplatelet therapy could be considered to reduce the risk of
future vascular events. In the acute setting, monocular occlusion should be
advised to alleviate diplopia if the horizontal misalignment is large. If the
misalignment is small, a Fresnel prism can be prescribed. If there is a failure
to recover and neuroimaging excludes other etiologies, diplopia from small
horizontal misalignments can be managed with prisms, whereas large, stable
horizontal misalignments can be managed with strabismus surgery.

K EY P O I N TS TO R E M E M B E R

■ Esotropia with an abduction deficit suggests a sixth nerve palsy,


but can also occur with orbital, neuromuscular, and central nervous
system disease.
■ Sixth nerve palsy is commonly due to microvascular ischemia,
trauma, multiple sclerosis, neoplasm, or stroke, although an
etiology cannot be established in as many as 25% of patients.

86 WHAT DO I DO NOW? NEURO-OPHTHALMOLOGY


■ Neuroimaging should be performed in patients who do not have a
typical presentation for microvascular sixth nerve palsy or who do
not recover within 3–6 months.
■ Microvascular sixth nerve palsy typically recovers completely within
3–6 months.

Further Reading
Bendszus M, Beck A, Koltzenburg M, et al. MRI in isolated sixth nerve palsies.
Neuroradiology. 2001;43:742–745.
Chou KL, Galetta SL, Liu GT, et al. Acute ocular motor mononeuropathies: prospective
study of the roles of neuroimaging and clinical assessment. J Neurol Sci.
2004;219:35–39.
Murchison AP, Gilbert ME, Savino PJ. Neuroimaging and acute ocular motor
mononeuropathies: a prospective study. Arch Ophthalmol. 2011;129:301–305.
Patel SV, Mutyala S, Leske DA, Hodge DO, Holmes JM. Incidence, associations, and
evaluation of sixth nerve palsy using a population-based method. Ophthalmology.
2004;111:369–375.
Wilker SC, Rucker JC, Newman NJ, Biousse V, Tomsak RL. Pain in ischaemic ocular motor
cranial nerve palsies. Br J Ophthalmol. 2009;93:1657–1659.

16. SIXTH NERVE PALSY 87


17 Ocular Myasthenia

A 72-year-old man presents to your clinic with a 6-month


history of intermittent left-sided ptosis and binocular
diplopia. The symptoms tend to occur toward the end of
the day. He has no other neurologic symptoms.
Examination reveals normal visual acuities, color vision,
confrontation visual fields, and pupils. There is partial
left-sided ptosis. His ductions and versions are full, but
he develops exotropia and complete left-sided ptosis with
prolonged upward gaze. Funduscopic examination is
normal. The remainder of his neurologic examination is
normal. Acetylcholine-receptor antibodies were not
detected on a blood sample sent by his referring doctor.

What do you do now?

88
T he combination of intermittent binocular diplopia and fatigable ptosis
is highly suggestive of ocular myasthenia. Myasthenia gravis is an auto-
immune disease in which there are antibodies to the acetylcholine receptor
on the postsynaptic membrane of the neuromuscular junction. Consequently,
there is abnormal neuromuscular transmission, which is manifest clinically
as fatigable muscle weakness. Many patients present with ocular symptoms,
such as binocular diplopia and ptosis. Approximately half of these patients
do not develop symptomatic systemic disease and remain purely ocular.
Examination typically reveals fluctuating asymmetric ptosis that fatigues
with prolonged upward gaze. If there is unilateral ptosis, there can be con-
tralateral lid retraction caused by attempted neural compensation for the
ptosis; the lid retraction often decreases with manual elevation of the ptotic
lid. There may be a brief upper-eyelid overshoot during gaze shifts from
down to straight ahead, which is known as a Cogan’s lid twitch. The extraoc-
ular muscle weakness in ocular myasthenia is variable and asymmetric,
although complete bilateral ophthalmoplegia can occasionally occur (see
case 18). Almost any pattern of weakness can be seen. Ocular myasthenia
often gives ocular motor findings that mimic other central and peripheral
ocular motor disorders, such as internuclear ophthalmoplegia and third
nerve palsy, but the pupils are normal. Orbicularis oculi weakness is
common and very suggestive of the diagnosis when seen in association with
ptosis or extraocular muscle weakness. When severe, weakness of the orbic-
ularis oculi can produce lower lid ectropion (eversion of the lid) or the
“peekaboo” sign, which is characterized by gradual eye opening during pro-
longed forceful eyelid closure.
While ocular myasthenia is easily suspected, it is often difficult to diag-
nose definitively; the average sensitivities, specificities, and likelihood ratios
for commonly used diagnostic tests are listed in Table 17-1. Indeed,
the situation described in this case scenario is commonly encountered in
clinical practice: the clinical presentation suggests ocular myasthenia, but
acetylcholine-receptor antibodies were not detected. The absence of acetyl-
choline-receptor antibodies cannot be used to exclude the diagnosis, because
only 50% of patients with ocular myasthenia have the antibodies detected
in serum. Performing an acetylcholine-receptor antibody panel, looking for
binding, blocking, and modulating antibodies, increases the diagnostic
yield only slightly. Although muscle-specific kinase (MuSK) antibodies are

17. OCULAR MYASTHENIA 89


TABLE 17-1 Accuracy of Diagnostic Tests for Ocular Myastheniaa

Test Sensitivity Specificity LR+ LR–

Ice test 94% 97% 31 0.06

Rest test 99% 91% 11 0.01

Edrophonium (Tensilon) test 92% 97% 31 0.08

Acetylcholine-receptor antibody 66% 99% 66 0.34

Repetitive nerve stimulationb 29% 94% 4.8 0.76

Single-fiber electromyographyc 97% 92% 12 0.03

Abbreviations: LR+, likelihood ratio for disease with positive test; LR–, likelihood ratio for
disease with negative test.
a
Adapted from Benatar M. A systematic review of diagnostic studies in myasthenia gravis.
Neuromuscul Disord. 2006;16:464.
b
Testing of various nerve–muscle pairs (including facial nerve–orbicularis oculi).
c
Testing of orbicularis oculi or frontalis.

sometimes detected in patients with generalized myasthenia gravis, there


are few reported cases of ocular myasthenia with positive MuSK antibodies.
Routine testing for MuSK antibodies in ocular myasthenia patients is there-
fore not recommended.
Several bedside tests can be performed to evaluate for ocular myasthenia.
Intravenous administration of edrophonium (Tensilon), which inhibits ace-
tylcholinesterase and thereby prolongs the action of acetylcholine, can
improve the symptoms and signs of ocular myasthenia. Sequential doses of
edrophonium are administered over a several-minute period (e.g., in 2-mg
increments) until a total of 10 mg is given. The test is considered positive if
there is unequivocal improvement in muscle weakness (e.g., improvement
in ptosis or extraocular muscle weakness). The test can be safely performed
in the office in most patients, although many institutions now require car-
diac monitoring during the test, given the potential for serious muscarinic
side effects (e.g., bradycardia and syncope). Atropine should always be avail-
able to reverse such side effects. Since edrophonium is not always available,
other clinical tests can be used to evaluate for ocular myasthenia. The ice
test is performed by placing a latex-free ice pack over the eyes for several
minutes, after which the clinician evaluates for an improvement in signs
(see Figure 17-1). The rest test (also known as the sleep test) is performed

90 WHAT DO I DO NOW? NEURO-OPHTHALMOLOGY


FIGURE 17-1 Ice test in ocular myasthenia demonstrating improvement in ptosis.

by having the patient lie with eyes closed for about 30 minutes, after which
the clinician evaluates for an improvement in signs. Both the ice and rest
tests are considered positive if there is an unequivocal improvement in
either ptosis or ophthalmoplegia. The findings of these tests need to be
interpreted with caution, however, because their sensitivities and specifici-
ties have not been evaluated in large studies. Furthermore, the interobserver
reliability of these tests remains unknown.
Electrophysiology testing can sometimes be useful in the evaluation of
patients with suspected ocular myasthenia. Repetitive nerve simulation test-
ing is specific but not sensitive for ocular myasthenia. Single-fiber electro-
myography (sfEMG), looking for increased “jitter,” is more sensitive for
ocular myasthenia, especially if the orbicularis oculi is studied. However,
sfEMG can be reliably performed only by examiners with specific expertise
in the technique and therefore may not be readily available.
In our patient with a suggestive presentation but no acetylcholine-
receptor antibodies, a diagnosis of ocular myasthenia would be supported
by positive results on ice, rest, edrophonium, and electrophysiology testing.

17. OCULAR MYASTHENIA 91


Attention should then be turned to treatment to reduce symptoms and,
ideally, induce disease remission. Symptoms and signs often improve with
pyridostigmine, an acetylcholinesterase inhibitor that is longer-acting than
edrophonium. Increases in dose and frequency may be required but must be
titrated to muscarinic side effects (e.g., cramps and diarrhea). Occasional
patients (e.g., those with severe bilateral ptosis) require immunosuppressive
therapy to control symptoms. Although corticosteroids are the mainstay of
immunosuppressive therapy for myasthenia gravis, their use for ocular
myasthenia remains controversial. Steroid-sparing agents (e.g., azathioprine
and mycophenolate) can be effective in patients who develop intolerable
side effects from corticosteroids. Thymectomy is not routinely recom-
mended unless there is a thymoma. Eyelid or strabismus surgery could be
considered in patients with stable ptosis or ocular misalignment.

K EY P O I N TS TO R E M E M B E R

■ Ocular myasthenia usually presents with intermittent or fluctuating


ptosis and diplopia.
■ About 50% of patients with ocular myasthenia do not have
acetylcholine-receptor antibodies.
■ Positive findings on ice, rest, edrophonium (Tensilon), and sfEMG
testing support a diagnosis of ocular myasthenia.
■ Symptoms and signs of ocular myasthenia improve with
pyridostigmine, but some patients require systemic treatment with
corticosteroids or other immunosuppressive agents to control
symptoms or achieve disease remission.

Further Reading
Benatar M. A systematic review of diagnostic studies in myasthenia gravis. Neuromuscul
Disord. 2006;16:459–467.
Daroff RB. The office Tensilon test for ocular myasthenia gravis. Arch Neurol. 1986;43:
843–844.
Evoli A, Tonali P, Bartoccioni E, Lo Monaco M. Ocular myasthenia: diagnostic and
therapeutic problems. Acta Neurol Scand. 1988;77:31–35.
Kaminski HJ, Daroff RB. Treatment of ocular myasthenia: steroids only when compelled.
Arch Neurol. 2000;57:752–753.
Luchanok U, Kaminski HJ. Ocular myasthenia: diagnostic and treatment
recommendations and the evidence base. Curr Opin Neurol. 2008;21:8–15.

92 WHAT DO I DO NOW? NEURO-OPHTHALMOLOGY


18 Complete Bilateral External
Ophthalmoplegia

A 36-year-old woman presents to your clinic with a


several-year history of progressively increasing bilateral
ptosis and intermittent binocular diplopia. She is
otherwise well and denies vision loss. Examination
reveals normal visual acuities, color vision, confrontation
visual fields, pupils, and fundi. There is partial
nonfatigable bilateral ptosis. Her eye movements are
severely limited in all directions. You are unable to
overcome this limitation with head movements.

What do you do now?

93
W hen a patient with globally limited ductions is encountered, the
first step is to determine if the lesion is supranuclear or nuclear–
infranuclear. Supranuclear lesions affect the saccadic, pursuit, optokinetic,
or vergence inputs to the ocular motor nuclei. Nuclear–infranuclear lesions
affect the ocular motor nuclei, nerves, neuromuscular junction, or extraoc-
ular muscles themselves. The distinction can easily be made at the bedside
by checking if the limitation in ductions can be overcome using the
vestibulo-ocular reflex (VOR), as elicited with caloric stimulation or head
movements (the “doll’s eyes” maneuver). Provided that the vestibular appa-
ratus is intact, the limitation can be overcome using the VOR in patients
with a supranuclear lesion, whereas it cannot in patients with a nuclear–
infranuclear lesion. The patient in this case scenario has a global limitation
of ductions that cannot be overcome with head movements, suggesting a
nuclear–infranuclear lesion. In the absence of pupil involvement (so-called
internal ophthalmoplegia), the deficit is consistent with complete bilateral
external ophthalmoplegia.
The differential diagnosis of complete bilateral ophthalmoplegia varies
depending on the tempo of onset (see Table 18-1). Common causes of
acute complete bilateral ophthalmoplegia include Miller Fisher syndrome,
Guillain-Barré syndrome, posterior-circulation (brainstem) stroke, and
myasthenia gravis; note that several of these conditions can affect the pupils

TABLE 18-1 Causes of Complete Bilateral Ophthalmoplegia

Acute onset Chronic progressive onset

Miller Fisher syndrome Mitochondrial myopathies (e.g., Kearnes-Sayre


Guillain-Barré syndrome syndrome, mitochondrial encephalopathy
Brainstem stroke with lactic acidosis and strokelike episodes,
Myasthenia gravis myoclonic epilepsy with ragged red fibers)
Pituitary apoplexy Oculopharyngeal dystrophy
Botulism Myotonic dystrophy
Drug toxicity Other congenital myopathies
(e.g., phenytoin) Cranial dysinnervation syndromes
Nasopharyngeal carcinoma Neurodegenerative diseases (e.g., progressive
Invasive fungal sinusitis supranuclear palsy, spinocerebellar ataxia
Wernicke’s encephalopathy type 2)
Trauma Thyroid eye disease
Myasthenia gravis

94 WHAT DO I DO NOW? NEURO-OPHTHALMOLOGY


and thereby give combined external and internal ophthalmoplegia. Causes
of chronic progressive external ophthalmoplegia (CPEO) include the mito-
chondrial myopathies, oculopharyngeal dystrophy, myotonic dystrophy,
cranial dysinnervation syndromes, and certain neurodegenerative diseases,
such as progressive supranuclear palsy (see case 21). Complete external oph-
thalmoplegia can also occur in patients with myasthenia gravis (see case 17)
and thyroid eye disease (see case 36), which sometimes coexist in the same
patient (see Figure 18-1). Our patient has had symptoms for several years
and her pupils are normal. Thus, it is likely that she has CPEO. Because the
differential diagnosis is broad, the next step is to take a careful history,
including family history, and to perform a thorough neurologic and sys-
temic examination looking for signs that suggest a specific diagnosis.
Given the age of our patient and lack of other neuro-ophthalmic signs,
apart from bilateral nonfatigable ptosis, she could have CPEO due to a
mitochondrial myopathy. CPEO can occur as part of Kearnes-Sayre syn-
drome, the syndrome of mitochondrial encephalopathy with lactic acidosis
and strokelike episodes (MELAS), and the syndrome of myoclonic epilepsy
with ragged red fibers (MERRF). There should be a careful inquiry about

FIGURE 18-1 Orbital magnetic resonance imaging (MRI; top) demonstrating enlarged
extraocular muscle bellies, and chest computed tomography (CT; bottom) demonstrating
goiter and thymoma (arrows), in a patient with complete bilateral external ophthalmoplegia
due to coexisting thyroid eye disease and seropositive ocular myasthenia.

18. COMPLETE BILATERAL EXTERNAL OPHTHALMOPLEGIA 95


similar symptoms in relatives on the maternal side of the family, because
these syndromes are caused by inherited mutations of mitochondrial DNA.
Interestingly, despite the inability to move their eyes, many patients do not
complain of constant diplopia or oscillopsia during head movements, pre-
sumably because there are adaptive changes in the visual system with the
slow onset of the ophthalmoplegia. However, patients are often symptom-
atic from ptosis, especially when the visual axis is obstructed. When encoun-
tering a patient with suspected CPEO, such as our patient, it is important
to ask about symptoms suggesting multisystem involvement (e.g., hearing
loss or cardiac arrhythmias). It is advisable to look for other neuro-ophthal-
mic signs of mitochondrial disease, such as pigmentary retinopathy and
optic neuropathy. However, the mitochondrial myopathies are notorious
for having variable manifestations, even with the same mutation. Thus,
genetic testing or muscle biopsy is required for definitive diagnosis. Several
therapies have been proposed for mitochondrial myopathies that cause
CPEO, including vitamins and coenzyme Q10, but the efficacy of these
therapies is uncertain. Symptomatic management, however, is often helpful.
Those patients with constant diplopia often benefit from prisms or strabis-
mus surgery. Symptomatic ptosis can be treated with eyelid surgery.

K EY P O I N TS TO R E M E M B E R

■ Supranuclear ophthalmoplegia can be overcome using the VOR,


whereas nuclear–infranuclear ophthalmoplegia cannot.
■ Causes of acute complete bilateral ophthalmoplegia include Miller
Fisher syndrome, Guillain-Barré syndrome, brainstem stroke,
myasthenia gravis, and pituitary apoplexy.
■ Causes of chronic progressive complete bilateral ophthalmoplegia
include mitochondrial myopathies, oculopharyngeal dystrophy,
myotonic dystrophy, cranial dysinnervation syndromes,
neurodegenerative diseases, thyroid eye disease, and myasthenia
gravis.
■ Patients with CPEO do not usually complain of diplopia but can have
symptomatic ptosis.

96 WHAT DO I DO NOW? NEURO-OPHTHALMOLOGY


Further Reading
Keane JR. Acute bilateral ophthalmoplegia: 60 cases. Neurology. 1986;36:279–281.
Keane JR. Bilateral ocular paralysis: analysis of 31 inpatients. Arch Neurol. 2007;64:
178–180.
Schoser BG, Pongratz D. Extraocular mitochondrial myopathies and their differential
diagnoses. Strabismus. 2006;14:107–113.
Thurtell MJ, Halmagyi GM. Complete ophthalmoplegia: an unusual sign of bilateral
paramedian midbrain-thalamic infarction. Stroke. 2008;39:1355–1357.

18. COMPLETE BILATERAL EXTERNAL OPHTHALMOPLEGIA 97


19 Superior Oblique Myokymia

A 52-year-old woman presents to your clinic complaining


of intermittent “twitching vision” in her left eye.
Examination reveals normal visual acuities, color vision,
confrontation visual fields, pupils, and eyelids. Her eye
movements appear to be normal. There are no obvious
saccadic intrusions and there is no nystagmus. However,
you see irregular, small-amplitude torsional movements
of her left fundus during the dilated funduscopic
examination.

What do you do now?

98
T he patient in this scenario has presented with the somewhat unusual
complaint of “twitching vision” in her left eye. In the absence of an
objective examination finding, it might be tempting to diagnose benign
eyelid fasciculations, which are very common and can give a sensation of
eye twitching, or perhaps to dismiss her symptoms as being nonorganic.
However, there are several organic conditions that can cause monocular
oscillations and thereby give illusory motion of the visual world (oscillop-
sia) when viewing through one eye. The first step in this scenario is to ask
the patient if she has covered each eye in turn during the episodes to con-
firm that the oscillopsia is indeed monocular, because the differential diag-
nosis of monocular oscillopsia differs from that of binocular oscillopsia.
Causes for monocular oscillopsia include superior oblique myokymia,
monocular pendular nystagmus in an eye with vision loss, and, rarely,
the Heimann-Bielschowsky phenomenon, in which there are variable-
amplitude low-frequency vertical oscillations in an eye with severe vision
loss. Monocular oscillations can also be seen in the spasmus nutans syn-
drome, a benign, transient condition of infants that is characterized by the
triad of nystagmus, head nodding, and anomalous head position. Given our
patient’s age and the fact that she has normal vision otherwise, she most
likely has superior oblique myokymia.
Superior oblique myokymia typically causes brief recurrent attacks of
monocular visual blurring or oscillopsia. The attacks can be precipitated by
adducting and depressing the affected eye. Although very annoying to the
patient, the eye oscillations are often subtle and difficult to detect on exam-
ination. Careful observation of the conjunctival vessels, however, may reveal
irregular, low-amplitude vertical-torsional oscillations of the affected eye
when the patient is symptomatic. These oscillations are often more obvious
on funduscopic examination, as in our patient. When the diagnosis is sus-
pected but no oscillations are seen, they can sometimes be elicited by asking
the patient to adduct and depress the affected eye.
The pathogenesis of superior oblique myokymia remains uncertain.
Since the condition is benign and affected patients do not have other neu-
rologic deficits, the pathogenesis is probably analogous to that of other par-
oxysmal cranial nerve disorders, such as trigeminal neuralgia. Indeed, it has
been suggested, based on the findings of imaging studies, that superior
oblique myokymia might arise because of vascular compression at the fourth

19. SUPERIOR OBLIQUE MYOKYMIA 99


nerve root entry zone causing focal demyelination (Yousry et al., 2002). It
would therefore be reasonable to obtain magnetic resonance imaging (MRI)
of the brain in our patient, looking for vascular compression of the fourth
nerve. With the exception of eye movement recordings, other investigations
are unlikely to be informative and therefore are not routinely required.
Although short-lived, the attacks of superior oblique myokymia are very
annoying. Consequently, patients often request treatment. Many patients
respond to medical therapy with carbamazepine, gabapentin, phenytoin, or
β-blockers (e.g., propranolol) without experiencing significant side effects.
Each of these medications could be tried in turn, with the dose titrated
according to beneficial response and side effects. Occasional patients have a
hyperphoria corresponding to the affected superior oblique muscle; pris-
matic correction for the hyperphoria can sometimes have a salutary effect
on the superior oblique myokymia. In those who do not respond to medical
treatment or prisms, strabismus surgery on the superior oblique tendon can
sometimes be effective. In those with vascular compression of the fourth
nerve who remain unresponsive to treatment, decompression of the nerve
root entry zone could be considered. However, given the invasive nature of
the procedure and the benign nature of this condition, decompression
should be considered as a last resort in only severely symptomatic patients.

K EY P O I N TS TO R E M E M B E R

■ Superior oblique myokymia can cause brief, recurrent attacks of


monocular visual blurring or oscillopsia.
■ The eye oscillations are often subtle but can sometimes be detected
with careful observation of the conjunctival vessels or on
funduscopic examination.
■ Superior oblique myokymia might arise because of vascular
compression of the fourth nerve root entry zone.
■ Superior oblique myokymia often responds to medical treatment
with carbamazepine, gabapentin, phenytoin, or b-blockers.
■ Severe intractable cases of superior oblique myokymia can respond
to strabismus surgery or decompression of the fourth nerve root
entry zone.

100 WHAT DO I DO NOW? NEURO-OPHTHALMOLOGY


Further Reading
Kosmorsky GS, Ellis BD, Fogt N, Leigh RJ. The treatment of superior oblique myokymia
utilizing the Harada-Ito procedure. J Neuroophthalmol. 1995;15:142–146.
Leigh RJ, Tomsak RL, Seidman SH, Dell’Osso LF. Superior oblique myokymia: quantitative
characteristics of the eye movements in three patients. Arch Ophthalmol.
1991;109:1710–1713.
Tomsak RL, Kosmorsky GS, Leigh RJ. Gabapentin attenuates superior oblique myokymia.
Am J Ophthalmol. 2002;133:721–723.
Williams PE, Purvin VA, Kawasaki A. Superior oblique myokymia: efficacy of medical
treatment. J AAPOS. 2007;11:254–257.
Yousry I, Dieterich M, Naidich TP, Schmid UD, Yousry TA. Superior oblique myokymia:
magnetic resonance imaging support for the neurovascular compression hypothesis.
Ann Neurol. 2002;51:361–368.

19. SUPERIOR OBLIQUE MYOKYMIA 101


20 Internuclear Ophthalmoplegia

A 28-year-old woman presents to your clinic with a 1-day


history of blurred vision while walking. She has had
transient diplopia previously but has not sought medical
attention. Examination reveals normal visual acuities in
both eyes. However, her visual acuity in both eyes
decreases to 20/60 during vertical head movements.
Her color vision, confrontation visual fields, and pupils
are normal. She is unable to adduct either eye past the
midline. Her eye movements are otherwise normal and
there is no ptosis. Funduscopic examination is normal,
and the remainder of her neurologic examination is
unrevealing.

What do you do now?

102
T he combination of bilateral adduction deficits and decreased visual
acuity with vertical head movements is highly suggestive of bilateral
internuclear ophthalmoplegia (INO) due to a lesion affecting both medial
longitudinal fasciculi. The medial longitudinal fasciculus (MLF) conveys
the conjugate horizontal eye movement command from the sixth nerve
nucleus in the pons to the medial rectus subdivision of the contralateral
third nerve nucleus in the midbrain. A lesion affecting the MLF causes an
ipsilateral adduction deficit, because the internuclear neurons in the MLF
ascend on the contralateral side of the brainstem. The MLF also carries
signals that have a role in vertical gaze holding, vertical smooth pursuit, and
the vertical vestibulo-ocular reflex (VOR). Consequently, a lesion affecting
the MLF can also impair these eye movements. Indeed, the patient in this
scenario is reporting blurred vision with vertical head movements, suggest-
ing that her vertical VOR is impaired.
The classic finding in patients with INO is an adduction deficit that can
be overcome with convergence, because the convergence input from the
midbrain to the medial rectus subdivision of the third nerve nucleus is usu-
ally not affected. Therefore, the first step in this patient is to confirm that
her bilateral adduction deficits can be overcome with convergence. Patients
with INO often have “dissociated” abducting nystagmus in the contralat-
eral eye, which is thought to represent an adaptive neural response to the
inability to adduct the ipsilateral eye during attempted versions. Patients
with INO often have a skew deviation and dissociated vertical-torsional
nystagmus, caused by involvement of central otolithic and semicircular
canal pathways, as well as impaired vertical smooth pursuit and impaired
vertical VOR. When a unilateral lesion also involves the adjacent sixth nerve
nucleus, there will also be an ipsilateral conjugate gaze palsy; the combina-
tion of a unilateral INO with an ipsilateral conjugate gaze palsy comprises
the “one-and-a-half ” syndrome.
Common causes of INO include demyelination due to multiple sclerosis
(MS), brainstem stroke, brainstem tumors, fourth-ventricular tumors, and
hindbrain anomalies, such as Chiari malformation. Bilateral INO is most
commonly caused by MS, whereas unilateral INO is most commonly
caused by brainstem stroke. Thus, the investigation of choice in our patient
is magnetic resonance imaging (MRI) of the brain looking for a pontine
tegmentum lesion, although such a lesion may be subtle and missed unless

20. INTERNUCLEAR OPHTHALMOPLEGIA 103


FIGURE 20-1 T2-weighted magnetic resonance imaging (MRI) demonstrating a tiny
brainstem lesion (arrow) causing an isolated left internuclear ophthalmoplegia.

specifically sought (see Figure 20-1). Although our patient has no other
symptoms or signs to suggest MS, apart from a prior history of transient
diplopia, MRI of the brain is also required to assess for white matter
lesions.
Other causes of adduction deficit should be kept in mind when evaluat-
ing a patient with an apparent INO, especially if the MRI is unrevealing.
Ocular myasthenia can give an adduction deficit that is clinically indistin-
guishable from that seen in INO (“pseudo-INO”); in contrast with INO,
the adduction deficit in pseudo-INO cannot be overcome with conver-
gence. There will often be accompanying ptosis or other signs that suggest
the correct diagnosis. Eye movement recordings can be used to distinguish
INO from pseudo-INO in difficult cases.
In most patients with INO due to MS or brainstem stroke, the deficit
spontaneously recovers in time. A 3-day course of intravenous methylpred-
nisone could be considered in our patient with bilateral INO if there is
evidence of demyelination on MRI, because treatment for an exacerbation
of MS may speed her recovery. Many patients with INO do not report

104 WHAT DO I DO NOW? NEURO-OPHTHALMOLOGY


diplopia. If diplopia is present, it is best managed acutely with monocular
occlusion rather than prisms. The long-term management of our patient
depends on the cause of the INO.

K EY P O I N TS TO R E M E M B E R

■ INO is caused by a lesion affecting the ipsilateral MLF.


■ The classic sign of INO is an adduction deficit that can be overcome
with convergence.
■ Common causes of INO include brainstem demyelination due to MS,
brainstem stroke, brainstem tumor, fourth-ventricular tumor, and
hindbrain anomalies.
■ Ocular myasthenia can give an adduction deficit that may be
indistinguishable from INO.
■ INO from brainstem demyelination and stroke usually recovers
spontaneously.

Further Reading
Kim JS. Internuclear ophthalmoplegia as an isolated or predominant symptom of
brainstem infarction. Neurology. 2004;62:1491–1496.
Schmidt F, Kastrup A, Nägele T, Krapf H, Küker W. Isolated ischemic internuclear
ophthalmoplegia: toward the resolution limits of DW-MRI. Eur J Neurol. 2004;11:
67–68.
Serra A, Liao K, Matta M, Leigh RJ. Diagnosing disconjugate eye movements: phase-plane
analysis of horizontal saccades. Neurology. 2008;71:1167–1175.
Zee DS. Internuclear ophthalmoplegia: pathophysiology and diagnosis. Baillieres Clin
Neurol. 1992;1:455–470.

20. INTERNUCLEAR OPHTHALMOPLEGIA 105


21 Progressive Supranuclear
Palsy

A 78-year-old man presents to your clinic with a 6-month


history of falls and difficulty reading. Examination reveals
visual acuities of 20/25 in both eyes. Color vision,
confrontation visual fields, pupils, and fundi are normal.
There is bilateral upper-eyelid retraction. His vertical eye
movements are limited, with slowing of vertical saccades.
His horizontal eye movements are normal, although he is
unable to converge.

What do you do now?

106
D ifficulty reading is a common complaint that can result from a number
of deficits, of which the most frequent is uncorrected presbyopia.
While correction of presbyopia may be all that is required to cure this
patient’s reading difficulty, his inability to converge and his limited vertical
eye movements are likely to be playing a role. For example, the patient’s
inability to look down when attempting to read might make it difficult for
him to look through the reading portion of bifocal or progressive-lens
glasses. There are several other features of concern in this patient, including
his falls, upper-eyelid retraction, and slowed vertical saccades. Indeed, the
combination of limited and slowed vertical saccades, upper-eyelid retrac-
tion, and convergence insufficiency suggests midbrain pathology (see case
35). It should therefore be determined if the limitation in vertical eye move-
ments is due to a supranuclear or nuclear–infranuclear lesion, by checking
if it can be overcome using the vestibulo-ocular reflex (see case 18). The
presence of a vertical supranuclear ophthalmoplegia narrows the differential
diagnosis considerably; given the history of frequent falls, the most likely
diagnosis is progressive supranuclear palsy (PSP).
PSP is a sporadic akinetic-rigid syndrome that is characterized by
increased axial tone, abnormal posture, and difficulties with swallowing and
speech. It can produce a number of characteristic neuro-ophthalmic signs,
including slowing and limitation of vertical (especially downward) saccades,
convergence insufficiency, square-wave jerks, and a variety of eyelid abnor-
malities, including upper-eyelid retraction with reduced blink rate (which
gives the patient a characteristic stare), apraxia of eyelid opening, and
blepharospasm. In the later stages of the disease, horizontal saccades become
impaired and the patient may eventually develop complete supranuclear
ophthalmoplegia (see case 18). The neurologic examination can reveal dys-
arthria, retrocollis, axial rigidity, bradykinesis, and impaired postural
reflexes, with a tendency to fall backward. Other parkinsonian signs, such
as tremor and cogwheel rigidity, are usually absent. Impaired frontal lobe
function can result in frontal release signs and the “applause” sign, which is
an inability to clap only three times despite being instructed to do so.
Although PSP is a clinical diagnosis, magnetic resonance imaging (MRI)
of the brain can reveal characteristic midbrain atrophy—the “humming-
bird” sign. Eye movement recordings can be helpful for detecting subtle
slowing of vertical saccades in the early stages of the disease. Several other

21. PROGRESSIVE SUPRANUCLEAR PALSY 107


rare diseases can give a similar clinical presentation, including Creutzfeldt-
Jakob disease, Whipple’s disease (in which there can be pathognomonic
verging–diverging eye oscillations with synchronous jaw movements called
oculomasticatory myorhythmia), and anti-Ma2 brainstem encephalitis.
However, each of these diseases has a subacute onset and inexorably pro-
gressive course over months, whereas PSP is usually slowly progressive over
years.
PSP is very difficult to manage medically. The extrapyramidal features
are poorly responsive to dopaminergic agents. Nevertheless, these agents
may provide some benefit to our patient and should be tried. Our patient’s
complaint of difficulty reading could be addressed by ensuring that he has
appropriate correction for presbyopia. He should be given separate single-
vision reading glasses, so that he does not have to look down through the
reading portion of bifocal or progressive-lens glasses to read. He may also
benefit from base-in prisms in his reading glasses, to compensate for his
convergence insufficiency. Regular application of artificial tears should be
advised if he has dry eye due to a decreased blink rate.

K EY P O I N TS TO R E M E M B E R

■ PSP is an akinetic-rigid syndrome characterized by dysarthria, axial


rigidity, and postural instability, with a tendency to fall backward.
■ Neuro-ophthalmic findings in PSP include supranuclear vertical
gaze palsy, convergence insufficiency, square-wave jerks, and eyelid
abnormalities.
■ PSP is a clinical diagnosis and is poorly responsive to treatment
with dopaminergic agents.
■ Reading difficulties in PSP can be improved with correction of
presbyopia, separate reading glasses, prisms, and regular
application of artificial tears.

Further Reading
Chen AL, Riley DE, King SA, et al. The disturbance of gaze in progressive supranuclear
palsy: implications for pathogenesis. Front Neurol. 2010; doi: 10.3389/fneur.2010.
00147.
Dubois B, Slachevsky A, Pillon B, Beato R, Villalponda JM, Litvan I. “Applause sign” helps
to discriminate PSP from FTD and PD. Neurology. 2005;64:2132–2133.

108 WHAT DO I DO NOW? NEURO-OPHTHALMOLOGY


Nath U, Ben-Shlomo Y, Thomson RG, Lees AJ, Burn DJ. Clinical features and natural
history of progressive supranuclear palsy: a clinical cohort study. Neurology.
2003;60:910–916.
Stamelou M, Knake S, Oertel WH, Höglinger GU. Magnetic resonance imaging in
progressive supranuclear palsy. J Neurol. 2011;258:549–558.

21. PROGRESSIVE SUPRANUCLEAR PALSY 109


22 Gaze-Evoked Nystagmus

A 52-year-old man presents for a routine eye


examination. He has a long history of epilepsy but is
otherwise well. Examination reveals normal visual
acuities, color vision, confrontation visual fields, pupils,
and fundi. His ductions and versions are full. There is no
nystagmus when he is looking straight ahead. However,
there is leftbeat nystagmus with leftward gaze, rightbeat
nystagmus with rightward gaze, and upbeat nystagmus
with upward gaze. The neurologic examination reveals an
unsteady tandem gait but no other abnormal signs. He
does not have any visual symptoms.

What do you do now?

110
T he patient in this case scenario has gaze-evoked nystagmus, which is a
jerk-waveform nystagmus that is present only during attempted eccen-
tric fixation, such that leftward gaze evokes leftbeat nystagmus, rightward
gaze evokes rightbeat nystagmus, and upward gaze evokes upbeat nystag-
mus. Gaze-evoked nystagmus is probably the most common type of nystag-
mus encountered in clinical practice. It results from impaired function of
the ocular motor gaze-holding mechanism, which is better known as the
“neural integrator.” In health, the neural integrator generates a position
signal to hold the eyes in eccentric orbital positions by opposing elastic
restoring forces that are acting to bring the eyes back toward the center of
the orbit. When neural integrator function is impaired, the position signal
is not adequate to oppose those elastic restoring forces. As a result, the eyes
drift toward the center of the orbit and corrective saccades (quick phases)
are required to direct them back to the desired eccentric position. In con-
trast with other forms of acquired nystagmus, patients usually do not have
visual symptoms from gaze-evoked nystagmus.
Before proceeding any further with this patient, it is important to be
convinced that he does indeed have gaze-evoked nystagmus and not physi-
ologic “end-point” nystagmus. Physiologic end-point nystagmus looks
similar to gaze-evoked nystagmus, but is low-amplitude, low-frequency,
and occurs only on adopting extremely eccentric gaze positions. It is usually
poorly sustained and is not accompanied by abnormal ocular motor or cer-
ebellar signs. In contrast, gaze-evoked nystagmus usually appears with less
extreme eccentric gaze positions and is often accompanied by other abnor-
mal ocular motor signs, such as impaired (“saccadic”) smooth pursuit, or
cerebellar signs. Given that our patient has an unsteady tandem gait, the
concern for gaze-evoked nystagmus should be greater.
Gaze-evoked nystagmus has limited localizing value. Rarely, it can result
from focal lesions affecting components of the neural integrator in the
medial medulla or midbrain reticular formation. More often, it is caused by
medications or neurologic conditions that affect neural integrator function
(see Box 22-1). Anticonvulsant medications, such as phenytoin and car-
bamazepine, are a frequent cause of gaze-evoked nystagmus. Indeed, the
presence of gaze-evoked nystagmus and an unsteady tandem gait in patients

22. GAZE-EVOKED NYSTAGMUS 111


BOX 22-1 CO M M O N CAU S ES O F GA Z E- EVO K E D N YSTAG M U S

Anticonvulsants (e.g., phenytoin, carbamazepine)


Other drugs (e.g., lithium, alcohol, marijuana)
Cerebellar degenerations
Multiple sclerosis
Episodic ataxias

taking these medications were considered clinical signs of drug toxicity


before monitoring of plasma drug levels came into routine practice. Of
note, acute intoxication with phenytoin can occasionally induce cerebellar
atrophy and thereby cause persistent gaze-evoked nystagmus, even after the
drug is discontinued. Given that our patient has a long history of epilepsy,
it is therefore important to inquire about previous and current anticonvul-
sant use. If the patient is indeed taking an anticonvulsant drug, the next
steps are to check the plasma level of the drug (to confirm that it is indeed
elevated) and to liaise with the patient’s neurologist about adjusting the
dose. Transient withdrawal of the anticonvulsant or a reduction in its dose
is likely to be all that is required for the gaze-evoked nystagmus and gait
ataxia to resolve.
In the event that the patient is not taking an anticonvulsant or is not
intoxicated with another implicated drug, such as alcohol or marijuana, a
more thorough history and workup is required. It will be important to
inquire about symptoms that might suggest the etiology of the gaze-evoked
nystagmus as well as about his past medical history and family history.
A complete neurologic examination might reveal other signs that suggest
the etiology. In terms of investigations, the next step, in most cases, would
be to obtain magnetic resonance imaging (MRI) of the brain to look for a
structural abnormality (e.g., cerebellar atrophy). Genetic testing might
be required if the patient reports a family history of progressive or
episodic ataxia. Regardless of the cause, gaze-evoked nystagmus does not
usually produce visual symptoms and therefore does not require specific
treatment. However, treatment should be directed toward the underlying
condition.

112 WHAT DO I DO NOW? NEURO-OPHTHALMOLOGY


K EY P O I N TS TO R E M E M B E R

■ Gaze-evoked nystagmus is the most common type of nystagmus


encountered in clinical practice.
■ Gaze-evoked nystagmus should be differentiated from physiologic
end-point nystagmus, which is of no concern.
■ Common causes of gaze-evoked nystagmus include anticonvulsant
medications, other drugs, cerebellar degenerations, multiple
sclerosis, and episodic ataxias.
■ Gaze-evoked nystagmus does not usually cause visual symptoms,
and therefore treatment should be directed toward the underlying
cause.

Further Reading
Baier B, Dieterich M. Incidence and anatomy of gaze-evoked nystagmus in patients with
cerebellar lesions. Neurology. 2011;76:361–365.
Büttner U, Grundei T. Gaze-evoked nystagmus and smooth pursuit deficits: their
relationship studied in 52 patients. J Neurol. 1995;242:384–389.
Remler BF, Leigh RJ, Osorio I, Tomsak RL. The characteristics and mechanisms of visual
disturbance associated with anticonvulsant therapy. Neurology. 1990;40:791–796.
Shallo-Hoffmann J, Schwarze H, Simonsz HJ, Mühlendyck H. A reexamination of end-point
and rebound nystagmus in normals. Invest Ophthalmol Vis Sci. 1990;31:388–392.

22. GAZE-EVOKED NYSTAGMUS 113


23 Downbeat Nystagmus

A 72-year-old woman with a past history of breast cancer


presents to your clinic with a several-month history of
progressively increasing vertical oscillopsia. Examination
reveals visual acuities of 20/30 in both eyes. Her color
vision, confrontation visual fields, pupils, and eyelids are
normal. Her ductions and versions are full. However, you
notice downbeat nystagmus that is most prominent on
down-lateral gaze. Her funduscopic examination is
normal. The remainder of her neurologic examination
reveals a wide-based, unsteady gait, but no other
abnormal signs.

What do you do now?

114
D ownbeat nystagmus is a vertical jerk-waveform nystagmus with upward
slow phases and downward quick phases, which commonly causes an
illusory motion of the visual world known as oscillopsia. The nystagmus is
usually present when the patient is looking straight ahead, but it can be dif-
ficult to detect without magnification. It typically increases with down
and lateral gaze. Convergence often exacerbates it, but can convert it to
upbeat nystagmus. The nystagmus often increases when the patient is placed
in a head-hanging or prone position. The nystagmus is usually accompa-
nied by other ocular motor abnormalities, such as gaze-evoked nystagmus,
impaired smooth pursuit, and impaired suppression of the vestibulo-ocular
reflex.
Downbeat nystagmus results from lesions affecting the flocculonodular
lobe of the cerebellum (i.e., the vestibulocerebellum). Common causes
include the inherited cerebellar degenerations and congenital hindbrain
anomalies, such as the Chiari malformation (see Box 23-1). However, no
cause will be found in up to 40% of patients with downbeat nystagmus,
despite extensive investigations. In our patient, the initial concern should
be for cerebellar metastases from breast cancer. These can be easily diag-
nosed with magnetic resonance imaging (MRI) of the brain. If they are
present, urgent consultation with an oncologist is required. If they are not
present, the MRI should be carefully scrutinized for signs of cerebellar atro-
phy, because the patient might have paraneoplastic cerebellar degeneration.
Paraneoplastic cerebellar degeneration has a subacute onset and is most
commonly associated with cancers of the lung, ovary, and breast. Downbeat
nystagmus is a frequent finding in affected patients. When there is an
underlying breast cancer, there may be anti-Yo or anti-Ri antibodies in

BOX 23 -1 CO M M O N CAU S ES O F D OW N BE AT N YSTAG M U S

Cerebellar degenerations (e.g., inherited, paraneoplastic)


Hindbrain anomalies (e.g., Chiari malformation)
Cerebellar stroke
Cerebellar mass lesions
Toxins (e.g., anticonvulsants, lithium, alcohol)

23. DOWNBEAT NYSTAGMUS 115


serum or cerebrospinal fluid. A significant percentage of patients with sus-
pected paraneoplastic cerebellar degeneration do not have antineuronal
antibodies or a history of cancer. Full-body positron emission tomography
(PET) should be obtained in these patients, looking for an occult neoplasm.
Full-body PET should also be obtained in patients with suspected paraneo-
plastic cerebellar degeneration who have a history of cancer but no other
evidence of relapse.
If our patient is ultimately diagnosed with paraneoplastic cerebellar degen-
eration based on finding an antineuronal antibody or evidence of relapse,
treatment should be directed toward the neoplasm itself. Unfortunately,
the paraneoplastic syndrome rarely improves with such treatment, although
it may stabilize. There are, however, reports of the paraneoplastic syn-
drome improving in patients who are also treated with plasmapheresis,
intravenous immunoglobulin, or immunosuppressant medications, especially
if these treatments are commenced soon after the onset of the neurologic
symptoms.
Regardless of the cause, many patients with downbeat nystagmus request
medical treatment to alleviate the associated visual symptoms. Clonazepam,
a GABAA-agonist, can improve downbeat nystagmus in some patients,
but its beneficial effect has not been proven in a controlled trial. Many
other drugs have been proposed as treatments for downbeat nystagmus
(e.g., baclofen, gabapentin, and anticholinergic drugs), but beneficial
effects have again not been demonstrated in controlled trials. Several recent
trials have suggested that potassium channel blockers can effectively sup-
press downbeat nystagmus (Kalla et al., 2004; Strupp et al., 2003).
Both 3,4-diaminopyridine and 4-aminopyridine can suppress it, but
they are more effective in patients with downbeat nystagmus caused by
cerebellar degenerations and less effective in those with focal lesions. Both
medications are well tolerated, although they can cause epileptic seizures
when given in high doses or cardiac arrhythmias if the patient has a pro-
longed QT interval. If our patient’s nystagmus and visual symptoms do not
improve with treatment of the underlying cause, a therapeutic trial with
4-aminopyridine, 3,4-diaminopyridine, or clonazepam should therefore be
considered.

116 WHAT DO I DO NOW? NEURO-OPHTHALMOLOGY


K EY P O I N TS TO R E M E M B E R

■ Downbeat nystagmus results from lesions of the flocculonodular


lobe of the cerebellum.
■ Common causes of downbeat nystagmus include cerebellar
degenerations (e.g., hereditary or paraneoplastic), hindbrain
anomalies (e.g., Chiari malformation), cerebellar stroke, cerebellar
mass lesions, and toxins.
■ Downbeat nystagmus often produces disabling vertical oscillopsia
or blurred vision.
■ Downbeat nystagmus can respond to 4-aminopyridine,
3,4-diaminopyridine, or clonazepam.

Further Reading
Kalla R, Glasauer S, Schautzer F, et al. 4-Aminopyridine improves downbeat nystagmus,
smooth pursuit, and VOR gain. Neurology. 2004;62:1228–1229.
Ko MW, Dalmau J, Galetta SL. Neuro-ophthalmologic manifestations of paraneoplastic
syndromes. J Neuroophthalmol. 2008;28:58–68.
Strupp M, Schüler O, Krafczyk S, et al. Treatment of downbeat nystagmus with
3,4-diaminopyridine: a placebo-controlled study. Neurology. 2003;61:165–170.
Wagner JN, Glaser M, Brandt T, Strupp M. Downbeat nystagmus: aetiology and
comorbidity in 117 patients. J Neurol Neurosurg Psychiatry. 2008;79:672–677.
Yoshida S, Takahashi H. Cerebellar metastases in patients with cancer. Surg Neurol.
2009;71:184–187.

23. DOWNBEAT NYSTAGMUS 117


24 Upbeat Nystagmus

You are asked to see a 62-year-old man who has


presented to the emergency department following the
sudden onset of vertical oscillopsia. He has had chronic
vomiting and has lost a large amount of weight following
gastric bypass surgery 6 months ago. Examination
reveals visual acuities of 20/40 in both eyes. He has
bilateral abduction deficits and upbeat nystagmus in all
positions of gaze. The upbeat nystagmus disappears with
convergence. Color vision, confrontation visual fields,
pupils, eyelids, and fundi are normal.

What do you do now?

118
T he patient in this scenario has vertical oscillopsia due to upbeat nystag-
mus. Upbeat nystagmus is a vertical jerk-waveform nystagmus with
downward slow phases and upward quick phases. It is distinct from gaze-
evoked upbeat nystagmus, which is a type of gaze-evoked nystagmus that is
seen exclusively on upgaze and is not present when the patient is looking
straight ahead (see case 22). Unlike downbeat nystagmus, upbeat nystag-
mus is not usually increased on lateral gaze, but it typically increases on
upward gaze. Convergence can exacerbate the nystagmus in some patients,
suppress it in others, and may even convert it to downbeat nystagmus. Like
downbeat nystagmus, upbeat nystagmus can also be modulated by head
posture.
Upbeat nystagmus results from lesions affecting the central structures
that process compensatory upward eye movement signals, such as the peri-
hypoglossal nuclei in the medial medulla. It can also result from lesions
affecting the central pathways that convey these signals between the ves-
tibular nuclei in the medulla and the ocular motor nuclei in the midbrain,
including the crossing ventral tegmental tract, brachium conjunctivum
(superior cerebellar peduncle), and medial longitudinal fasciculus.
Consequently, upbeat nystagmus can result from lesions in a variety of loca-
tions in the brainstem and cerebellum. While it might be tempting to
obtain imaging to look for a brainstem or cerebellar lesion, the first step in
our patient is to complete the neurologic examination, as there may be
other signs that indicate the location of the lesion. For example, the patient
might also have a bulbar palsy, suggesting a medial medullary lesion.
However, there might be signs that do not help with localization of the
lesion but that do suggest the correct diagnosis. We might find that our
patient has confusion and gait ataxia in addition to upbeat nystagmus and
bilateral abduction deficits. Given the history of chronic vomiting follow-
ing gastric bypass surgery with this combination of neurologic signs, our
primary concern should be for Wernicke’s encephalopathy.
While upbeat nystagmus can have a variety of causes (see Box 24-1), the
combination of upbeat nystagmus and abduction deficits is highly sugges-
tive of Wernicke’s encephalopathy. Suppression of upbeat nystagmus with
convergence can also occur in patients with Wernicke’s encephalopathy.
The decreased visual acuities in our patient are almost certainly a conse-
quence of his upbeat nystagmus, but they could also be due to optic nerve

24. UPBEAT NYSTAGMUS 119


BOX 24-1 CO M M O N CAU S ES O F U P BE AT N YSTAG M U S

Brainstem and cerebellar stroke


Wernicke’s encephalopathy
Multiple sclerosis
Brainstem and cerebellar tumors
Cerebellar degenerations

involvement, which can produce central visual field defects and, in some
cases, mild optic disc edema in patients with Wernicke’s encephalopathy.
Given that our patient had normal color vision, confrontation visual fields,
and fundi, optic nerve involvement is unlikely to be the cause for his
decreased visual acuities.
Although the diagnosis of Wernicke’s encephalopathy is clinical, mag-
netic resonance imaging (MRI) can show highly characteristic abnormali-
ties, including signal change in the mammillary bodies, medial thalami,
periaqueductal gray, floor of the fourth ventricle, and cranial nerve nuclei.
Although MRI might be helpful, treatment of our patient with intravenous
thiamine should not be delayed while awaiting imaging. Timely treatment is
not only important to prevent the development of Korsakoff’s syndrome,
which is characterized by irreversible amnesia and confabulation, but will
maximize recovery of his ocular motor deficits. Indeed, treatment with intra-
venous thiamine will often result in a rapid and complete resolution of the
ocular motor deficits. Consequently, the upbeat nystagmus does not usually
require specific treatment in patients with acute Wernicke’s encephalopathy.
However, it can persist if there is a delay in thiamine treatment. Patients with
persistent upbeat nystagmus will usually request treatment, because of severe
disabling oscillopsia. Unfortunately, there have been few clinical trials of
medical treatment for upbeat nystagmus. One uncontrolled trial reported a
beneficial effect with baclofen (Dieterich et al., 1991). A recent double-
masked trial reported suppression of upbeat nystagmus with memantine
(Thurtell et al., 2010), whereas another trial reported suppression with
4-aminopyridine (Glasauer et al., 2005). If our patient’s nystagmus does not
disappear with thiamine treatment, a therapeutic trial with memantine,
4-aminopyridine, or baclofen could be considered. Given that all of these

120 WHAT DO I DO NOW? NEURO-OPHTHALMOLOGY


medications have side effects, the choice of medication and dosage will
depend on what other residual neurologic signs the patient has.

K EY P O I N TS TO R E M E M B E R

■ Since upbeat nystagmus can result from lesions in a variety of


locations in the brainstem and cerebellum, other localizing signs
should be sought.
■ Common causes of upbeat nystagmus include stroke, Wernicke’s
encephalopathy, multiple sclerosis, brainstem and cerebellar
tumors, and cerebellar degenerations.
■ The combination of upbeat nystagmus and abduction deficits is
highly suggestive of Wernicke’s encephalopathy.
■ Persistent upbeat nystagmus might respond to memantine,
4-aminopyridine, or baclofen.

Further Reading
Aasheim ET. Wernicke encephalopathy after bariatric surgery: a systematic review.
Ann Surg. 2008;248:714–720.
Dieterich M, Straube A, Brandt T, Paulus W, Büttner U. The effects of baclofen and
cholinergic drugs on upbeat and downbeat nystagmus. J Neurol Neurosurg
Psychiatry. 1991;54:627–632.
Fisher A, Gresty M, Chambers B, Rudge P. Primary position upbeating nystagmus: a
variety of central positional nystagmus. Brain. 1983;106:949–964.
Glasauer S, Kalla R, Büttner U, Strupp M, Brandt T. 4-Aminopyridine restores visual ocular
motor function in upbeat nystagmus. J Neurol Neurosurg Psychiatry. 2005;76:
451–453.
Thurtell MJ, Joshi AC, Leone AC, et al. Crossover trial of gabapentin and memantine as
treatment for acquired nystagmus. Ann Neurol. 2010;67:676–680.
Zuccoli G, Pipitone N. Neuroimaging findings in acute Wernicke’s encephalopathy.
Am J Roentgenol. 2009;192:501–508.

24. UPBEAT NYSTAGMUS 121


25 Acquired Pendular
Nystagmus

A 32-year-old woman with a long history of multiple


sclerosis (MS) presents to your clinic for evaluation of
blurred and “wobbly” vision. She has had multiple
attacks of optic neuritis in both eyes, but she does not
currently have pain with eye movement. Examination
reveals visual acuities of 20/50 in both eyes. She has red
desaturation in both eyes, but confrontation visual fields
are full. Her pupils are equal in size but sluggishly
reactive to light, and there is a trace right relative
afferent pupillary defect. Her eye movements are full, but
there are low-amplitude and high-frequency elliptical eye
oscillations, without quick phases, more prominent in the
right eye. Funduscopic examination reveals optic disc
pallor, more prominent in the right eye.

What do you do now?

122
P atients with MS commonly have visual complaints, which can result
from involvement of the optic nerves, optic chiasm, retrochiasmal
visual pathways, or brainstem structures and pathways important for eye
movements. The patient in this scenario has had multiple attacks of optic
neuritis in both eyes (see case 1) and has signs suggesting bilateral asym-
metric optic neuropathies, including red desaturation, a relative afferent
pupillary defect, and optic disc pallor. Thus, optic neuropathy is almost
certainly contributing to her complaint of blurred vision. In addition, she
has elliptical eye oscillations suggesting the presence of acquired pendular
nystagmus (APN). APN is a form of nystagmus with a sinusoidal waveform
such that there are slow phases in both directions without corrective quick
phases. The nystagmus can have variable horizontal, vertical, and torsional
components. In patients with APN due to MS, the oscillations are often
low-amplitude and may be difficult to see without careful observation of
the eyes. However, the oscillations usually have a high frequency, which is
why they often cause severe oscillopsia and blurring of vision. It is therefore
likely that our patient’s blurred vision is due to the unfortunate combina-
tion of optic neuropathy and APN.
The pathogenesis of APN in MS patients is not well understood. One
possibility is that the APN arises because of delays in the conduction of
visual information, since it is often more prominent in the eye with greater
vision loss, as in our patient’s case. However, this does not fully explain why
APN occurs, since it remains unchanged in darkness and experimental
delay of visual feedback does not change its characteristics. A more likely
explanation is that APN arises due to an unstable “neural integrator.” The
neural integrator has an important role in ensuring steady gaze. Components
of the neural integrator are located in various parts of the brainstem.
Whereas impaired neural integrator function produces gaze-evoked nystag-
mus (see case 22), APN might result from loss of normal feedback to the
neural integrator, thereby producing instability, perhaps due to plaques of
demyelination affecting the paramedian tracts in the pons.
APN can also occur as a component of the syndrome of oculopalatal
tremor (OPT), which is characterized by irregular synchronous oscillations
of the eyes, palate, and other branchial muscles. The APN of OPT is usually
vertical-torsional and dysconjugate (i.e., the nystagmus differs in the two
eyes), with variable amplitude and frequency components. Unlike the

25. ACQUIRED PENDULAR NYSTAGMUS 123


APN of MS, the APN of OPT is thought to occur because of the disruption
of connections between the dentate nucleus and the contralateral inferior
olivary nucleus via the superior cerebellar peduncle, central tegmental tract,
and inferior cerebellar peduncle. OPT is most commonly a delayed conse-
quence of strokes in the brainstem tegmentum or cerebellum. Other
common causes of APN are listed in Box 25-1.
Given our patient’s history and clinical signs, her APN is likely to be due
to MS. The first step in her management is to optimize her vision by ensur-
ing that easily treated causes of blurred vision, such as refractive error, are
addressed. It is possible to screen for uncorrected refractive error by recheck-
ing visual acuity with a pinhole device. Given that this may prove to be
challenging in the presence of nystagmus, ophthalmic consultation should
also be considered, to check if the patient’s vision can be improved with a
manifest refraction and to screen for other easily treated causes of vision
loss. Treatment for acute optic neuritis with steroids is not indicated at this
time, because she is not reporting acute vision loss or pain with eye move-
ments. The second step in the management of this patient is to decrease her
oscillopsia by treating her APN. Several clinical trials have demonstrated
that gabapentin and memantine can partially suppress APN in MS patients,
thereby reducing oscillopsia and increasing visual acuity (Averbuch-Heller
et al., 1997; Thurtell et al., 2010). However, both can cause significant side
effects; gabapentin can cause unsteadiness, which can be problematic in MS
patients who have ataxia, whereas memantine can cause a reversible exacer-
bation of many MS symptoms (Villoslada et al., 2009). Consequently,
gabapentin may be the preferred first-line treatment for APN in MS
patients, although its dose will need to be carefully titrated according to
both beneficial and adverse effects. The final step in the management of this

BOX 25 -1 CO M MO N CAU S ES O F ACQ U I R E D


P E N D U L A R N YSTAG MU S

Vision loss
Multiple sclerosis
Oculopalatal tremor
Acute brainstem stroke

124 WHAT DO I DO NOW? NEURO-OPHTHALMOLOGY


patient is to minimize her risk of further vision loss due to MS exacerba-
tions by ensuring that she is being treated with disease-modifying therapy,
such as beta-interferon. Consultation with an MS neurologist may ulti-
mately be required to optimize her disease-modifying therapy.

K EY P O I N TS TO R E M E M B E R

■ APN is characterized by slow phases in both directions without


corrective quick phases.
■ APN can cause disabling oscillopsia and blurred vision.
■ Common causes of APN include vision loss, MS, OPT, and acute
brainstem stroke.
■ APN in MS often occurs in association with optic neuropathy and is
often more prominent in the eye with poorer vision.
■ APN can be partially suppressed with gabapentin or memantine, but
the doses must be titrated according to side effects.

Further Reading
Averbuch-Heller L, Tusa RJ, Fuhry L, et al. A double-blind controlled study of gabapentin
and baclofen as treatment for acquired nystagmus. Ann Neurol. 1997;41:818–825.
Averbuch-Heller L, Zivotofsky AZ, Das VE, DiScenna AO, Leigh RJ. Investigations of the
pathogenesis of acquired pendular nystagmus. Brain. 1995;118:369–378.
Thurtell MJ, Joshi AC, Leone AC, et al. Crossover trial of gabapentin and memantine as
treatment for acquired nystagmus. Ann Neurol. 2010;67:676–680.
Villoslada P, Arrondo G, Sepulcre J, Alegre M, Artieda J. Memantine induces reversible
neurologic impairment in patients with MS. Neurology. 2009;72:1630–1633.

25. ACQUIRED PENDULAR NYSTAGMUS 125


26 Infantile Nystagmus
Syndrome

An 16-year-old man presents to your clinic complaining


of blurred vision in both eyes. He has had extensive
investigations for nystagmus that has been present since
infancy, but no ocular, neurologic, or endocrine
abnormality was found. He now wants to apply for a
driver’s license but has been told that his visual acuity
is not adequate. Examination reveals visual acuities of
20/60 in both eyes. Color vision, confrontation visual
fields, pupils, and fundi are normal. His ductions are full
and there is no ocular misalignment. However, there is a
large-amplitude horizontal pendular nystagmus that is
present in all positions of gaze but less prominent when
he converges. He asks if there is anything that can be
done to improve his vision.

What do you do now?

126
T he patient described in this scenario has infantile nystagmus syndrome
(INS). Formerly known as congenital nystagmus, INS is usually spo-
radic but can be an inherited trait; X-linked INS has recently been associ-
ated with mutations in the FRMD7 gene, which regulates neuronal
outgrowth and development. INS usually occurs in isolation, but can be
associated with other visual or neurologic abnormalities (see Box 26-1). Up
to a third of patients with INS have coexisting strabismus.
The nystagmus of INS is usually first noted in infancy but is occasionally
not detected until adulthood. It can have a sinusoidal (pendular) or jerk
waveform with accelerating slow phases. In contrast with acquired forms of
nystagmus, the waveform is often complex and punctuated by brief “fove-
ation” periods, during which the eyes are transiently still. Given that patients
are able to see clearly during the foveation periods, their visual acuity is
often better than might be expected. Surprisingly, INS patients with well-
developed foveation periods rarely complain of oscillopsia, despite almost
continuous movement of their eyes. The nystagmus is predominantly hori-
zontal in most patients, but some also have small torsional and vertical
components to their nystagmus. It is usually conjugate (i.e., the nystagmus
is similar in the two eyes), more prominent when attempting to fixate a
distant target, and less prominent when converging to fixate a near target.

BOX 26 -1 CONDITIONS ASSOCIATED WITH INFANTILE


N YSTAG M U S SY N D RO M E

Ocular and oculocutaneous albinism


Achromatopsia (“color blindness”)
Congenital cataract
Optic atrophy
Optic nerve hypoplasia
Septo-optic dysplasia
Achiasma (absent optic chiasm)
Retinopathy of prematurity
Cone–rod dystrophies
Leber’s congenital amaurosis
Congenital stationary night blindness

26. INFANTILE NYSTAGMUS SYNDROME 127


The nystagmus may be minimized (and visual acuity maximized) when the
eyes adopt a position known as the “null” position or zone. Patients may
unconsciously adopt a head turn to keep their eyes in this position.
The first step in the evaluation of a patient with suspected INS is a com-
plete neuro-ophthalmic assessment, because the response to treatment
depends on the severity of any afferent visual system abnormality. It can be
very helpful to record the patient’s eye movements, because identification of
a characteristic INS waveform confirms the diagnosis. Most patients do not
require further investigations unless other visual, neurologic, or systemic
abnormalities are suspected. Magnetic resonance imaging (MRI) can be
used to detect structural abnormalities of the anterior visual pathways,
brain, and pituitary gland. Electrophysiology testing (e.g., an electroretino-
gram) may be required to detect retinal disease. Endocrine evaluation is
required in patients with suspected pituitary dysfunction (e.g., in septo-
optic dysplasia).
Our patient has already had a thorough workup for associated ocular,
neurologic, and endocrine conditions. Thus, no further investigations are
required and you should focus on discussing treatment options during the
clinic visit. A number of effective treatments for INS are now available,
including optical, medical, and surgical treatments. In this patient, oph-
thalmic consultation should be obtained to screen for easily treated causes
of blurred vision, such as refractive error. If the patient has uncorrected
refractive error, prescription of contact lenses should be considered, because
they can partially suppress INS as well as provide optical correction
(Dell’Osso et al., 1988). Another optical treatment approach to consider in
this patient would be prescription of base-out prisms to induce convergence
and thereby suppress his nystagmus during distance viewing; minus correc-
tion should be added to compensate for any induced accommodation.
Correction of refractive error, use of contact lenses, or use of base-out prisms
may be all that is required to improve the patient’s visual acuity so that he
can get his driver’s license.
If optical therapies are inadequate to improve our patient’s visual acuity,
medical and surgical approaches could be considered. Medical therapy
using gabapentin and memantine has recently been shown to reduce nystag-
mus intensity, prolong foveation periods, and improve visual acuity in patients
with INS, although patients with afferent visual system abnormalities are

128 WHAT DO I DO NOW? NEURO-OPHTHALMOLOGY


less likely to respond (McLean et al., 2007). Medical therapy with acetazo-
lamide could also be considered but can cause unpleasant side effects (see
case 6). Several surgical treatments might also be effective. Some patients
benefit from a tenotomy-and-reattachment procedure in which the hori-
zontal extraocular muscles are severed from the eye at their tendinous
attachments and then reattached at their original positions. Tenotomy-
and-reattachment has been shown to suppress nystagmus, prolong fove-
ation periods, and increase the range of the null zone (Wang et al., 2006).
The mechanism by which it works is uncertain, although it has been pro-
posed that it might modulate proprioceptive feedback signals from the
extraocular muscles (Dell’Osso, Tomsak, & Thurtell, 2009). Tenotomy-
and-reattachment can be combined with conventional strabismus surgery if
there is coexisting strabismus, an eccentric null position, or an anomalous
head position. It could also be combined with a divergence procedure in
our patient, so that viewing of distant targets will induce convergence and
thereby further suppress his nystagmus. Because optical, medical, and surgi-
cal therapies act by different mechanisms, therapies could be combined if
the improvement in visual acuity is insufficient with one therapy alone.

K EY P O I N TS TO R E M E M B E R

■ INS is usually sporadic, but can be an inherited (e.g., X-linked) trait.


■ INS can be associated with abnormalities of the visual and central
nervous systems.
■ Patients with INS sometimes report blurred vision but rarely
complain of oscillopsia.
■ Eye movement recordings may be required to definitively
diagnose INS.
■ INS can be successfully treated with optical, medical, and surgical
approaches.

Further Reading
Betts-Henderson J, Bartesaghi S, Crosier M, et al. The nystagmus-associated FRMD7 gene
regulates neuronal outgrowth and development. Hum Mol Genet. 2010;19:342–351.
Dell’Osso LF, Tomsak RL, Thurtell MJ. Two hypothetical nystagmus procedures:
augmented tenotomy and reattachment and augmented tendon suture (sans
tenotomy). J Pediatr Ophthalmol Strabismus. 2009;46:337–344.

26. INFANTILE NYSTAGMUS SYNDROME 129


Dell’Osso LF, Traccis S, Erzurum SI. Contact lenses and congenital nystagmus. Clin Vis Sci.
1988;3:229–232.
McLean R, Proudlock F, Thomas S, Degg C, Gottlob I. Congenital nystagmus: randomized,
controlled, double-masked trial of memantine/gabapentin. Ann Neurol. 2007;61:
130–138.
Serra A, Dell’Osso LF, Jacobs JB, Burnstine RA. Combined gaze-angle and vergence
variation in infantile nystagmus: two therapies that improve the high-visual-acuity
field and methods to measure it. Invest Ophthalmol Vis Sci. 2006;47:2451–2460.
Thurtell MJ, Dell’Osso LF, Leigh RJ, Matta M, Jacobs JB, Tomsak RL. Effects of
acetazolamide on infantile nystagmus syndrome waveforms: comparisons to contact
lenses and convergence in a well-studied subject. Open Ophthalmol J. 2010;4:42–51.
Wang Z, Dell’Osso LF, Jacobs JB, Burnstine RA, Tomsak RL. Effects of tenotomy on
patients with infantile nystagmus syndrome: foveation improvement over a
broadened visual field. J AAPOS. 2006;10:552–560.

130 WHAT DO I DO NOW? NEURO-OPHTHALMOLOGY


27 Saccadic Intrusions
and Dysmetria

A 42-year-old man presents to your clinic complaining of


difficulty reading. Examination reveals normal distance
visual acuities, color vision, confrontation visual fields,
pupils, and eyelids. His ductions and versions are full,
but he has ocular dysmetria: he consistently overshoots
the visual target when he makes horizontal saccades.
The overshooting is so severe that he makes a series of
horizontal saccades to either side of the new visual
target before he is finally able to fixate on it. You note
that he has difficulty reading a short paragraph because
of these overshoots. You also note that he has occasional
inappropriate horizontal saccades during attempted
fixation. These saccades are also evident on funduscopic
examination.

What do you do now?

131
S accades are rapid eye movements that assist vision by redirecting the
line of sight from one target of interest to another, so that the image of
the new target falls onto the fovea of the retina. A variety of saccadic disor-
ders can be recognized clinically, including disorders of saccadic initiation,
speed, and amplitude. When saccadic amplitudes are inappropriately small
or large, the patient is said to have saccadic (ocular) dysmetria; saccades that
are inappropriately small are hypometric, whereas those that are inappropri-
ately large are hypermetric. The patient in this scenario has saccadic hyper-
metria, because his horizontal saccades consistently overshoot the visual
target. Saccadic dysmetria does not usually cause visual symptoms unless it
is severe. Our patient’s saccadic hypermetria is so severe that he makes sev-
eral overshooting saccades about a visual target before he is finally able to
fixate on it (see Figure 27-1). Such severe saccadic hypermetria makes read-
ing very difficult, because reading is a task that requires small saccades to be
made accurately and in quick succession. Whether symptomatic or not,
saccadic dysmetria, especially hypermetria, is highly suggestive of cerebellar
disease (see Box 27-1 for a list of causes).
Our patient also has intermittent saccadic intrusions during attempted
fixation. Saccadic intrusions are inappropriate, involuntary saccades that
disrupt steady fixation. Several types of saccadic intrusion can be recognized
clinically, including square-wave jerks, macrosaccadic oscillations, ocular
flutter, and opsoclonus. Square-wave jerks are small involuntary horizontal
saccades that take the eye off target and are followed, after about 250 mil-
liseconds, by another saccade that brings the eye back onto target. They can
occur in individuals without neurologic disease, but can be very prominent
in patients with certain neurodegenerative diseases, such as progressive
supranuclear palsy (see case 21) and Friedreich’s ataxia.

FIGURE 27-1 Profile of macrosaccadic oscillations (black line), with eye position plotted
against time, when attempting to fixate on a target (gray line).

132 WHAT DO I DO NOW? NEURO-OPHTHALMOLOGY


BOX 27-1 CO M M O N CAU S ES O F SACCA D I C DYS M E T R I A

Cerebellar degenerations (e.g., inherited, paraneoplastic)


Hindbrain anomalies (e.g., Chiari malformation)
Cerebellar and brainstem (e.g., lateral medullary) stroke
Cerebellar mass lesions
Multiple sclerosis

Square-wave jerks do not usually cause visual symptoms. In contrast,


macrosaccadic oscillations often cause patients to have difficulty reading.
They are crescendo–decrescendo runs of horizontal saccades, with an inter-
saccadic interval of about 200 milliseconds, which often occur following a
gaze shift but can also occur during attempted fixation. Macrosaccadic
oscillations arise because of extreme saccadic hypermetria; the oscillations
consist of repeated hypermetric saccades about a visual target, with eventual
fixation of the target. They usually occur in patients with cerebellar disease
but can also occur in patients with pontine lesions.
In contrast to other saccadic intrusions, ocular flutter and opsoclonus
consist of back-to-back saccades without an intersaccadic interval; in ocular
flutter the saccades are purely horizontal, whereas in opsoclonus they are
multidimensional. When frequent or continuous, these back-to-back sac-
cades can cause distressing and disabling visual symptoms. Ocular flutter
and opsoclonus can occur in association with many conditions, including
brainstem encephalitis and paraneoplastic syndromes, as well as with drug
intoxications and abnormal metabolic states.
Our patient has saccadic hypermetria and macrosaccadic oscillations,
which are highly suggestive of cerebellar disease. The first step is to obtain a
detailed medical history asking about other neurologic symptoms, other
medical conditions, and family history. The next step is to complete the
neurologic examination, looking for other neurologic signs that might sug-
gest the diagnosis. Given that he complains of difficulty reading, it is also
important to check his near visual acuity, to exclude presbyopia. Investigations
such as imaging and genetic studies will then be required to determine the
cause of his cerebellar syndrome. Although treatment should then be
directed toward the underlying cause, it might not affect the saccadic abnor-
malities. There have been no clinical trials evaluating medical treatment for

27. SACCADIC INTRUSIONS AND DYSMETRIA 133


saccadic hypermetria or macrosaccadic oscillations. However, memantine
has been reported to decrease the frequency of saccadic intrusions, and
thereby improve reading, in patients with certain forms of cerebellar degen-
eration (Serra et al., 2008). Thus, in our patient with difficulty reading due
to macrosaccadic oscillations, a trial of memantine treatment should be
offered.

K EY P O I N TS TO R E M E M B E R

■ Saccadic disorders include abnormalities in saccadic initiation,


speed, and amplitude.
■ Saccadic disorders are often asymptomatic but can be a sign of
neurologic disease.
■ When severe, saccadic intrusions can give rise to difficulty reading
or oscillopsia.
■ Saccadic dysmetria and macrosaccadic oscillations are features of
cerebellar disease.
■ The frequency of macrosaccadic oscillations can be reduced with
memantine.

Further Reading
Averbuch-Heller L, Kori AA, Rottach KG, Dell’Osso LF, Remler BF, Leigh RJ. Dysfunction of
pontine omnipause neurons causes impaired fixation: macrosaccadic oscillations
with a unilateral pontine lesion. Neuroophthalmology. 1996;16:99–106.
Serra A, Liao K, Martinez-Conde S, Optican LM, Leigh RJ. Suppression of saccadic
intrusions in hereditary ataxia by memantine. Neurology. 2008;70:810–812.
Swartz BE, Li S, Bespalova I, et al. Pathogenesis of clinical signs in recessive ataxia with
saccadic intrusions. Ann Neurol. 2003;54:824–828.
Thurtell MJ, Tomsak RL, Leigh RJ. Disorders of saccades. Curr Neurol Neurosci Rep.
2007;7:407–416.

134 WHAT DO I DO NOW? NEURO-OPHTHALMOLOGY


SECTION III

EYELID DISORDERS
This page intentionally left blank
28 Eyelid Ptosis

A 48-year-old woman presents to your clinic with a


several-month history of left-sided ptosis. She denies
diplopia but mentions that she has been wearing rigid
contact lenses for many years. Examination reveals
normal corrected visual acuities, color vision,
confrontation visual fields, pupils, and eye movements.
There is 2 millimeters of nonfluctuating and nonfatigable
left-sided ptosis, with a high upper-lid crease.

What do you do now?

137
P tosis is a lowering of the upper-eyelid margin that is commonly
encountered in clinical practice. It has a broad differential diagnosis,
because the causative lesion can involve any one of several anatomically
distinct structures: preseptal structures; levator palpebrae superioris muscle
(see case 18); Müller’s muscle or oculosympathetic pathway (see case 31);
neuromuscular junction (see case 17); third nerve, third nerve fascicle,
or third nerve nucleus (see case 14); or supranuclear inputs. Thus, the first
step in the evaluation of this patient is to obtain a thorough history. There
should be a careful inquiry about the tempo of onset, precipitating factors
(e.g., trauma or recent eye surgery), and associated symptoms (e.g., diplo-
pia, unequal pupils, or neurologic symptoms), because these may suggest
the etiology (e.g., third nerve palsy, ocular myasthenia, or Horner’s syn-
drome). Although fluctuating ptosis suggests ocular myasthenia, it is not
specific; ptosis often gets worse toward the end of the day regardless of
its cause. It is worthwhile to ask if the ptosis is long-standing and to inspect
old photographs (e.g., driver’s license) if there is uncertainty about its dura-
tion. The presence of systemic diseases or a family history might also be
relevant.
The next step in the evaluation of our patient is to perform an eyelid
examination, as this may be all that is required to make the correct diagno-
sis. Ptosis should be distinguished from dermatochalasis, in which there is
excessive upper-eyelid tissue that can hang over the eyelid margin without
lowering the eyelid margin itself. Dermatochalasis is a common condition
that can cause visual impairment if the pupil is obscured, but it is not caused
by neurologic disease and therefore does not require neurologic workup.
Once it has been established that the upper eyelid does indeed have a lower
position, there should be a careful observation of the eyelid position, shape,
and movement. Several eyelid measurements should also be obtained. The
width of the palpebral fissure is measured with the patient looking straight
ahead. Upper-eyelid position is determined by measuring the marginal–
reflex distance, which is the distance between the midpupil corneal light
reflex and the upper-eyelid margin. Levator function is determined by mea-
suring the difference in the position of the upper eyelid with downward
versus upward gaze. Measurement of levator function is especially impor-
tant in the evaluation of patients with isolated ptosis, because the presence
of normal levator function indicates that the cause of ptosis is not weakness

138 WHAT DO I DO NOW? NEURO-OPHTHALMOLOGY


of the levator muscle or a lesion affecting its innervation. The position of
the upper-eyelid skin crease relative to the upper-eyelid margin is also useful
to measure. Because the upper-eyelid skin crease is created by the pull of the
levator on the skin, levator weakness makes the crease less prominent (see
Figure 28-1). When the upper-eyelid skin crease is prominent and higher
than normal (see Figure 28-1), the ptosis is likely to be due to dehiscence of
the levator aponeurosis rather than levator weakness. Other signs that are
important to look for include fatigability and Cogan’s lid twitch (see case 17).
It is also crucial to assess orbicularis oculi strength, because this is often
decreased in neuromuscular disorders that cause ptosis. The pupils and eye
movements should also be assessed, because either or both of these might be
abnormal in patients with third nerve palsy or Horner’s syndrome.
The presence of a high upper-lid crease with normal pupils and eye
movements suggests that our patient does not have neurogenic or myo-
pathic ptosis. If her levator function is normal and there are no other signs
to suggest myopathy or neuromuscular disease, the diagnosis is levator
dehiscence and no further investigation is required. Levator dehiscence is
the most common cause of acquired ptosis in adults and is often due to
long-term use of contact lenses; repeated stretching of the upper eyelid
during insertion of the contact lens, as well as constant rubbing of the lens
against the eyelid, results in thinning and eventual dehiscence of the levator
aponeurosis. Direct trauma to the eyelid is another common cause for leva-
tor dehiscence. For example, the levator aponeurosis can easily be damaged
by injections of anesthetic into the eyelid or by the use of eyelid retractors.
Consequently, levator dehiscence is often iatrogenic.
When the pupil is encroached by the upper eyelid in a patient with leva-
tor dehiscence, the effect on vision can be assessed by obtaining kinetic or

FIGURE 28-1 Absent upper-eyelid creases, backward head tilt, and frontalis activation in
bilateral myopathic ptosis (left). High upper-eyelid crease in left-sided levator dehiscence
(right).

28. EYELID PTOSIS 139


static automated perimetry and then repeating it with the eyelid taped up
into a normal position. If there is a significant superior visual field defect
that disappears when the eyelid is taped up, the most suitable treatment is
eyelid surgery, which is simple, safe, and effective. Patients who do not have
visual compromise do not require surgery, although any contributing fac-
tors, such as contact lens use, should be minimized. Eyelid surgery can also
be considered in patients who are bothered by the cosmetic appearance of
the ptosis.

K EY P O I N TS TO R E M E M B E R

■ Levator dehiscence is the most common cause of acquired ptosis


in adults.
■ Signs of levator dehiscence include normal levator function and a
high upper-eyelid crease.
■ Levator dehiscence is commonly due to long-term contact lens use
or direct trauma (e.g., from use of eyelid retractors during surgery)
and does not require neurologic workup.
■ Eyelid surgery is a simple, safe, and effective treatment for levator
dehiscence.

Further Reading
Ahmad SM, Della Rocca RC. Blepharoptosis: evaluation, techniques, and complications.
Facial Plast Surg. 2007;23:203–215.
Kersten RC, de Conciliis C, Kulwin DR. Acquired ptosis in the young and middle-aged adult
population. Ophthalmology. 1995;102:924–928.
Riemann CD, Hanson S, Foster JA. A comparison of manual kinetic and automated static
perimetry in obtaining ptosis fields. Arch Ophthalmol. 2000;118:65–69.

140 WHAT DO I DO NOW? NEURO-OPHTHALMOLOGY


29 Benign Essential
Blepharospasm

A 62-year-old woman complains of difficulty keeping her


eyes opened, especially when she is exposed to bright
lights. Examination reveals normal visual acuities, color
vision, confrontation visual fields, pupils, and eye
movements. There is no eyelid ptosis. Upper-eyelid
excursion is normal and symmetric. Funduscopic
examination is normal, but provokes orbicularis oculi
spasms that continue intermittently throughout the rest
of the consultation. The remainder of her neurologic
examination is unremarkable.

What do you do now?

141
B lepharospasm is an involuntary and inappropriate closure of the eyes that
results from spasm of the orbicularis oculi muscles. It can occur in asso-
ciation with many ophthalmic and neurologic diseases, such as progressive
supranuclear palsy (PSP; see case 21), but it usually occurs in isolation. When
it occurs in association with other abnormal facial movements, it is called
oromandibular dystonia or Meige’s syndrome. When occurring in isolation, it
is called benign essential blepharospasm (BEB). BEB most commonly occurs
in women who are over 50 years of age. The blepharospasm is often triggered
by exposure to bright lights or other relatively benign stimuli and can be dis-
abling to the point where affected patients are functionally blind.
The etiology of BEB remains uncertain. It was previously thought to be a
manifestation of nonorganic disease, but it probably arises because of dysfunc-
tion within the basal ganglia or brainstem. Despite this, investigations are
usually unrevealing and therefore are not routinely required in the evaluation
of BEB. Many patients with BEB have photophobia that is similar in severity
to that observed in migraine sufferers, but an ophthalmic cause for their pho-
tophobia is rarely identified. Some patients have other ocular complaints, such
as dryness or irritation. While such complaints warrant a careful ophthalmic
examination, their treatment (e.g., with artificial tears) rarely results in a sig-
nificant improvement in the blepharospasm. Consequently, these conditions
are not thought to play a direct role in the pathogenesis of BEB.
BEB can sometimes be difficult to distinguish from other eyelid disor-
ders, such as apraxia of eyelid opening (AEO). AEO is characterized by
transient inability to open the eyes in the absence of orbicularis oculi con-
traction. Patients with AEO often have frontalis activation during attempts
to open the eyes but do not have any other neuro-ophthalmic deficits to
suggest a neurologic or neuromuscular cause for the inability to open their
eyes. AEO occurs in a variety of neurologic diseases, such as PSP and
Parkinson’s disease. The distinction between BEB and AEO is based solely
on clinical characteristics. AEO often occurs in association with BEB,
although the pathogenesis of these two disorders is thought to be distinct.
The patient in this scenario has a normal examination, with the excep-
tion of photophobia and blepharospasm. She should be carefully observed
for oral or mandibular movements, which would suggest a diagnosis of
Meige’s syndrome. She should be examined to look for ophthalmic causes
of photophobia (e.g., dry eye), as treatment of these may lead to some

142 WHAT DO I DO NOW? NEURO-OPHTHALMOLOGY


improvement in her photophobia. Other investigations are not required,
because they are unlikely to be revealing.
Once satisfied that the patient’s blepharospasm does not have another
cause, the focus should turn to its management. The mainstay of treatment
for BEB is injection of botulinum toxin A into the orbicularis oculi and sur-
rounding muscles. For comfort, ice is applied over the patient’s eyes prior to
the injections. The injections are usually given in several places around the
eyes, and their effects are typically sustained for 2–4 months. Higher doses
than average may be required in some patients, depending on response. The
most common complications of the treatment include bleeding, bruising,
ptosis, and diplopia, but these are transient and usually mild. Many patients
report an improvement in their photophobia and blepharospasm with the
use of tinted lenses (e.g., FL-41) that filter out blue-wavelength light. Medical
treatments can be tried (e.g., clonazepam and gabapentin), but these are
often not effective. Surgical treatments can sometimes be effective, especially
in those patients who have associated dermatochalasis or are unresponsive to
other treatments. Although surgical treatments might reduce the severity of
the blepharospasm in some patients, they should not be considered the first
line of therapy. Thus, the most appropriate initial treatment for our patient
is botulinum toxin A injections into the orbicularis oculi every 3 months,
with escalating doses depending on response. She should also be prescribed
tinted (e.g., FL-41) lenses for her photophobia. Surgical therapy should be
considered only if she is unresponsive to these interventions.

K EY P O I N TS TO R E M E M B E R

■ Blepharospasm is a symmetric involuntary closure of the eyes that


is caused by spasm of the orbicularis oculi.
■ BEB most often occurs in older women, in the absence of neurologic
or ophthalmic disease.
■ Investigations are usually unrevealing and therefore not routinely
required in BEB.
■ BEB is most effectively treated with botulinum toxin A injections
every 2-4 months.
■ Associated photophobia can often be reduced by wearing tinted
(e.g., FL-41) lenses.

29. BENIGN ESSENTIAL BLEPHAROSPASM 143


Further Reading
Adams WH, Digre KB, Patel BC, Anderson RL, Warner JE, Katz BJ. The evaluation of light
sensitivity in benign essential blepharospasm. Am J Ophthalmol. 2006;142:82–87.
Blackburn MK, Lamb RD, Digre KB, et al. FL-41 tint improves blink frequency, light
sensitivity, and functional limitations in patients with benign essential
blepharospasm. Ophthalmology. 2009;116:997–1001.
Cillino S, Raimondi G, Guépratte N, et al. Long-term efficacy of botulinum toxin A for
treatment of blepharospasm, hemifacial spasm, and spastic entropion: a multicentre
study using two drug-dose escalation indexes. Eye (Lond). 2010;24:600–607.
Peckham EL, Lopez G, Shamim EA, et al. Clinical features of patients with blepharospasm:
a report of 240 patients. Eur J Neurol. 2011;18:382–386.

144 WHAT DO I DO NOW? NEURO-OPHTHALMOLOGY


SECTION IV

PUPIL DISORDERS
This page intentionally left blank
30 Physiologic Anisocoria

You are asked to see a 30-year-old woman who was


incidentally found to have a difference in the size of her
pupils (anisocoria) by her optometrist. She has not had
diplopia, ptosis, or headaches. Examination reveals
normal visual acuities, color vision, confrontation visual
fields, eye movements, and eyelids. There is 1 mm of
anisocoria; the left pupil is larger than the right. The
degree of anisocoria is unchanged in light and dark. Both
pupils react briskly to light and near. There is no relative
afferent pupillary defect.

What do you do now?

147
T he size of the pupil is determined by the net tone between the iris
sphincter and iris dilator muscles. The iris sphincter muscle is inner-
vated by the parasympathetic division of the autonomic nervous system and
produces pupil constriction (miosis) when it contracts. The iris dilator
muscle is innervated by the sympathetic division of the autonomic nervous
system and produces pupil dilation (mydriasis) when it contracts. Unilateral
or asymmetric disruption of the iris muscles or their innervation by a struc-
tural or pharmacologic lesion can produce anisocoria, which is defined as a
difference of 0.4 mm or more in the size of the pupils.
The cause of the anisocoria (see Table 30-1) can often be determined on
the basis of the clinical signs and findings on pharmacologic pupil testing
(see Figure 30-1). However, the first step in the evaluation of a patient with
anisocoria is to obtain a history. When the anisocoria was noted inciden-
tally by a relative, friend, or health care professional, it is helpful to inspect
old photographs to determine if it is long-standing; it is also often helpful
to use an ophthalmoscope to magnify the pupils in the photographs. The
patient should be asked about symptoms that could be related to the aniso-
coria, such as light sensitivity in one eye (e.g., with changes in lighting level)
or loss of accommodation (e.g., difficulty focusing with one eye at near),
because the ciliary muscle is also innervated by the parasympathetic divi-
sion of the autonomic nervous system. The presence of diplopia or ptosis
may suggest third nerve palsy (see case 14) as the cause for the anisocoria.
A history of trauma, certain medical problems (e.g., migraine), medication
use (e.g., topical or nebulized medications), or occupational exposure may
also be relevant.
The next step in the evaluation of the patient is a careful examination.
Pupil size should be assessed in room light with the patient fixating on a

TABLE 30-1 Common Causes of Anisocoria

Anisocoria greater in darkness Anisocoria greater in bright light

Horner’s syndrome Third nerve palsy


Iris adhesions (e.g., posterior synechiae) Tonic pupil
Pharmacologic (e.g., pilocarpine) Iris trauma or ischemia
Physiologic anisocoriaa Pharmacologic (e.g., mydriatics)

a
The anisocoria is usually similar in bright light and darkness, but can be more prominent in
darkness.

148 WHAT DO I DO NOW? NEURO-OPHTHALMOLOGY


Anisocoria

Greater in dark Greater in light

Apraclonidine Slit-lamp
test examination

No sector
No reversal Reversal Iris
palsy or iris Sector palsy
of anisocoria of anisocoria abnormality
abnormality

0.1%
Physiologic Horner’s
pilocarpine
anisocoria syndrome
test

Hydroxyamphetamine Pupil does not Pupil


test constrict constricts

Pupil does 1% pilocarpine Tonic pupil or


Pupil dilates test Third nerve
not dilate
palsy

Pre-ganglionic Post-ganglionic Pupil Pupil does not


Horner’s Horner’s constricts constrict
syndrome syndrome

Third nerve Pharmacologic Iris trauma or


palsy mydriasis ischemia

FIGURE 30-1 Algorithm for the evaluation of anisocoria (adapted from Kawasaki A. Disorders of pupillary function, accommodation, and lacrimation. In:
Miller NR, Newman NJ, Biousse V, Kerrison JB, eds. Walsh & Hoyt’s clinical neuro-ophthalmology. 6th ed. Philadelphia, PA: Lippincott Williams & Wilkins;
2005:739–805).
distant target, because near viewing will elicit the near triad (convergence,
accommodation, and pupillary miosis). If anisocoria is present, pupil size
should then be measured in bright light and darkness using a pupil gauge
(e.g., on a Rosenbaum near card). Pupil size should also be measured while
the patient is fixating on a near target. The relative speeds of pupil constric-
tion and redilation should be assessed, and there should be a comparison
between the speed of pupil response to light and near. The presence or absence
of a relative afferent pupillary defect should be documented. An ophthal-
mologist should assess the anterior segments with the slit-lamp to look for
relevant abnormalities (e.g., posterior synechiae, iris sphincter muscle tear,
transillumination defects, or segmental iris palsy). Abnormalities of ocular
alignment, ductions, and the eyelids should also be specifically sought.
The differential diagnosis and subsequent investigation of anisocoria
depends on the degree of anisocoria in bright light compared with that in
darkness (see Table 30-1 and Figure 30-1). The smaller pupil is abnormal
when anisocoria is greater in darkness, whereas the larger pupil is abnormal
when it is greater in bright light. In our patient, however, the anisocoria is
unchanged in bright light and darkness. She has no diplopia, ptosis, or
headaches, and her examination is otherwise normal. Her anisocoria is
therefore likely to be physiologic and of no concern.
Physiologic anisocoria (also known as benign, essential, or simple
anisocoria) is common in the general population, with up to 20% having
0.4–1 mm of anisocoria when their pupils are viewed in dim light. The
prevalence of physiologic anisocoria is not influenced by sex, age, or iris
color. The degree of anisocoria is usually the same in light and darkness,
although it can be slightly more prominent in darkness. The degree of ani-
socoria often varies within an individual, but it does not usually reverse.
Physiologic anisocoria is important to recognize, so as to avoid inappropri-
ate diagnostic testing. Thus, our patient does not require further investiga-
tions and can be reassured.
A diagnostic dilemma can arise when physiologic anisocoria occurs in
combination with levator dehiscence (see case 28) on the side of the smaller
pupil, mimicking Horner’s syndrome; this is known as pseudo-Horner’s syn-
drome. In such patients and in those with physiologic anisocoria that is more
prominent in darkness, pharmacologic pupil testing with 0.5% apraclonidine
can help to exclude a true Horner’s syndrome (see Figure 30-1 and case 31).

150 WHAT DO I DO NOW? NEURO-OPHTHALMOLOGY


K EY P O I N TS TO R E M E M B E R

■ The differential diagnosis and investigation of anisocoria depends


on the degree of anisocoria in bright light compared with darkness.
■ Physiologic anisocoria is characterized by 0.4–1 mm of anisocoria
that is similar in bright light and darkness with an otherwise normal
examination.
■ Physiologic anisocoria does not require further investigation, and
the patient can be reassured.

Further Reading
Kawasaki A. Disorders of pupillary function, accommodation, and lacrimation.
In: Miller NR, Newman NJ, Biousse V, Kerrison JB, eds. Walsh & Hoyt’s clinical
neuro-ophthalmology. 6th ed. Philadelphia, PA: Lippincott Williams & Wilkins;
2005:739–805.
Lam BL, Thompson HS, Corbett JJ. The prevalence of simple anisocoria. Am J Ophthalmol.
1987;104:69–73.
Martin TJ. Horner’s syndrome, pseudo-Horner’s syndrome, and simple anisocoria.
Curr Neurol Neurosci Rep. 2007;7:397–406.
Thompson BM, Corbett JJ, Kline LB, Thompson HS. Pseudo-Horner’s syndrome.
Arch Neurol. 1982;39:108–111.

30. PHYSIOLOGIC ANISOCORIA 151


31 Horner’s Syndrome

You are called to see a 24-year-old man who was found to


have anisocoria following a motor vehicle accident. He is
in the emergency room and is complaining of sharp
left-sided neck pain. Examination reveals normal visual
acuities, color vision, and confrontation visual fields.
There is 1.5 mm of anisocoria. The left pupil is smaller
than the right. The anisocoria is most prominent in
darkness. Both pupils show normal reactions to light and
near, and there is no relative afferent pupillary defect.
There is subtle left-sided ptosis. Eye movements are
normal, and the remainder of his neurologic examination
is unremarkable.

What do you do now?

152
T he combination of left-sided miosis and ptosis in this patient suggests
Horner’s syndrome. Horner’s syndrome results from a lesion affecting
the ipsilateral oculosympathetic pathway, which innervates the iris dilator
muscle and the smooth muscle of the eyelids. The oculosympathetic path-
way consists of three serial neurons: the first-, second-, and third-order neu-
rons. The first-order neuron descends from the hypothalamus through the
lateral brainstem and cervical spinal cord to synapse with the second-order
neuron at the C8–T1 level. The second-order neuron passes out of the
spinal cord, across the lung apex, and up the neck to synapse with the third-
order neuron in the superior cervical ganglion, near the bifurcation of the
common carotid artery. The third-order neuron travels up along the inter-
nal carotid artery until it reaches the cavernous sinus, where it briefly joins
with the sixth nerve before joining the ophthalmic division of the trigemi-
nal nerve to enter the orbit.
The first step in the evaluation of this patient is to confirm that he does
indeed have Horner’s syndrome. The classic signs are miosis and ptosis on
the affected side. Denervation of the iris dilator muscle produces miosis
that is most obvious in darkness (see Figure 31-1), because the action of
the iris dilator muscle is impaired. The anisocoria is less obvious in bright
light (see Figure 31-1), because the iris sphincter muscle is working
normally in both eyes. When the lights are turned out, a normal pupil
dilates rapidly (usually within 5 seconds) because of the simultaneous
contraction of the iris dilator (agonist) muscle and relaxation of the iris

FIGURE 31-1 Horner’s syndrome in a patient with right neck trauma. The anisocoria is less
prominent in bright light (top) than in darkness (bottom). Note that there is little if any ptosis.

31. HORNER’S SYNDROME 153


sphincter (antagonist) muscle. When there is sympathetic denervation,
however, the pupil dilates slowly (over 15–20 seconds), because the iris dila-
tor muscle is paretic. Thus, a characteristic finding in Horner’s syndrome is
dilation lag of the miotic pupil. The upper-eyelid ptosis in Horner’s syn-
drome results from loss of the sympathetic innervation of the eyelid smooth
muscle (Müller’s muscle). The ptosis is often mild and can easily be over-
looked (see Figure 31-1). Loss of sympathetic innervation to the smooth
muscle in the lower eyelid often results in a slight elevation known as reverse
or “upside-down” ptosis. The palpebral fissure narrowing that results from
combined upper-eyelid ptosis and lower-eyelid elevation can mimic the
appearance of enophthalmos. Facial anhidrosis can occur with lesions
involving the first- and second-order neurons, as a result of sympathetic
denervation of the sweat glands in the face. When Horner’s syndrome is
congenital, the ipsilateral iris is depigmented, giving rise to iris heterochro-
mia. Thus, our patient should be carefully examined again, looking for dila-
tion lag of the left pupil, elevation of the left lower eyelid, facial anhidrosis,
and iris heterochromia. If these signs are not present, pharmacologic pupil
testing should be pursued to confirm the diagnosis.
In the past, the cocaine test was the gold-standard test for diagnosis of
Horner’s syndrome. Cocaine blocks reuptake of norepinephrine into sym-
pathetic nerve terminals and therefore will cause a normal pupil to dilate.
When there is sympathetic denervation, norepinephrine is not being
released from sympathetic nerve terminals, and therefore there will be little
or no dilation of the affected pupil following instillation of cocaine. The
cocaine test is performed by instilling two drops of 10% cocaine into both
eyes and then evaluating for anisocoria in darkness 45 minutes later; aniso-
coria of 0.8 mm or more is diagnostic of Horner’s syndrome.
The apraclonidine test has recently been proposed as an alternative to the
cocaine test, because cocaine is a controlled substance and is often not read-
ily available. Apraclonidine is often used for the treatment of glaucoma and
is an α-receptor agonist, with strong α2 action and weak α1 action. It has
minimal effect on a normal pupil. However, it will dilate a sympathetically
denervated pupil, because the α1 receptors on the iris dilator muscle become
supersensitive to apraclonidine in the absence of normal sympathetic tone.
The test is performed by instilling two drops of 0.5% apraclonidine into
each eye and then evaluating for anisocoria in darkness 45 minutes later.

154 WHAT DO I DO NOW? NEURO-OPHTHALMOLOGY


The anisocoria reverses, because only the abnormal pupil dilates. Although
the apraclonidine test is reported to be close to 100% sensitive and specific
for the diagnosis of Horner’s syndrome, false-negative results can occur in
the acute setting because α1 denervation supersensitivity has not yet devel-
oped. Given our patient’s acute presentation, the possibility of a false-
negative result should be considered if an apraclonidine test is performed.
A lesion anywhere along the ipsilateral oculosympathetic pathway can
cause Horner’s syndrome (see Table 31-1). The history and examination
findings often help to localize the lesion. The presence of brainstem or
cerebellar signs suggests a first-order neuron lesion, the presence of arm
pain, weakness, or sensory loss suggests a second-order neuron lesion, and

TABLE 31-1 Causes of Horner’s Syndromea

First-order neuron Second-order neuron Third-order neuron

Hypothalamus/ Cervical cord: Superior cervical ganglion:


brainstem: Trauma Trauma
Stroke Tumor Jugular ectasia
Demyelination Syrinx Iatrogenic
AVM
Cervical cord: Internal carotid artery:
Cervical spondylosis
Trauma Dissection
Tumor Brachial plexus: Trauma
Syrinx Trauma Thrombosis
AVM Tumor
Pulmonary apex:
Iatrogenic Skull base:
Tumor (e.g., Pancoast) Trauma
Infection (e.g., apical TB) Tumor
Cervical rib
Cavernous sinus:
Vascular anomalies
Tumor
Neck: Inflammation
Iatrogenic Aneurysm
Trauma Carotid–cavernous
Tumor fistula
Thrombosis

Other:
Cluster headache

Abbreviations: AVM, arteriovenous malformation; TB, tuberculosis


a
Adapted from Kawasaki A, Kardon RH. Disorders of the pupil. Ophthalmol Clin North Am.
2001;14:149–168.

31. HORNER’S SYNDROME 155


the presence of neck or facial pain suggests a third-order neuron lesion. If
localizing signs are identified, targeted imaging studies can be performed.
When the location of the lesion is in doubt, the hydroxyamphetamine test
can be used to distinguish between a preganglionic (first- or second-order
neuron) and postganglionic (third-order neuron) lesion. However, the test
cannot be interpreted if it is performed within 24 hours of the cocaine or
apraclonidine test and therefore is not practical in our patient. Given the his-
tory of recent trauma, new-onset neck pain, and absence of other signs, our
patient could have an internal carotid artery (ICA) dissection. Thus, further
investigations should be obtained while he is still in the emergency room.
Acute isolated painful Horner’s syndrome is the most common finding
in patients with ICA dissection. Given the increased risk of stroke following
ICA dissection, emergent imaging of the head and neck vasculature is
required. Our patient should have magnetic resonance angiography (MRA)
of the head and neck vessels along with magnetic resonance imaging (MRI)
of the head and neck with fat suppression (see Figure 31-2). If a dissection
is identified, vascular risk factors should be addressed and he should be
started on aspirin, because current evidence does not suggest a benefit from
anticoagulation with warfarin. Patients who go on to have a hemispheric
stroke may require thrombolysis or definitive endovascular intervention for
their ICA dissection.

FIGURE 31-2 Left internal carotid artery dissection (arrows) on magnetic resonance
angiography (MRA; left) and magnetic resonance imaging (MRI) with fat suppression (right).

156 WHAT DO I DO NOW? NEURO-OPHTHALMOLOGY


K EY P O I N TS TO R E M E M B E R

■ Horner’s syndrome is caused by a lesion affecting the


oculosympathetic pathway.
■ It is characterized by upper-eyelid ptosis and lower-eyelid elevation
with miosis and dilation lag.
■ It can be diagnosed when there is reversal of anisocoria on
apraclonidine testing, but false-negative results can occur acutely
before denervation supersensitivity has developed.
■ Acute isolated painful Horner’s syndrome should be investigated
with MRI and MRA of the head and neck, to exclude ICA dissection.

Further Reading
Biousse V, Touboul PJ, D’Anglejan-Chatillon J, Lévy C, Schaison M, Bousser MG.
Ophthalmologic manifestations of internal carotid artery dissection. Am J
Ophthalmol. 1998;126:565–577.
Georgiadis D, Arnold M, von Buedingen HC, et al. Aspirin vs anticoagulation in carotid
artery dissection: a study of 298 patients. Neurology. 2009;72:1810–1815.
Kardon RH, Denison CE, Brown CK, Thompson HS. Critical evaluation of the cocaine test in
the diagnosis of Horner’s syndrome. Arch Ophthalmol. 1990;108:384–387.
Kawasaki A, Kardon RH. Disorders of the pupil. Ophthalmol Clin North Am. 2001;14:
149–168.
Koc F, Kavuncu S, Kansu T, Acaroglu G, Firat E. The sensitivity and specificity of 0.5%
apraclonidine in the diagnosis of oculosympathetic paresis. Br J Ophthalmol.
2005;89:1442–1444.
Trobe JD. The evaluation of Horner syndrome. J Neuroophthalmol. 2010;30:1–2.

31. HORNER’S SYNDROME 157


32 Tonic Pupil

A 24-year-old woman presents to your clinic complaining


of increased sensitivity to light when looking through her
left eye. She has no other symptoms and has no
significant medical history. Examination reveals normal
distance visual acuities, color vision, confrontation visual
fields, eyelids, and eye movements. There is 3 mm of
anisocoria. The left pupil is larger than the right,
especially in bright light. The right pupil shows normal
direct and consensual reactions to light. The left pupil
shows sluggish direct and consensual reactions to light.
Both pupils constrict equally with convergence, but the
left pupil is slow to redilate.

What do you do now?

158
I ncreased sensitivity to light (photophobia) is a frequent complaint that
can result from ophthalmic disease (e.g., corneal ulcer, dry eye syndrome,
or uveitis) or neurologic disease (e.g., migraine or meningitis). When it is
monocular, as in the patient described in this scenario, it is likely to be due
to unilateral ophthalmic disease, and therefore a thorough ophthalmic
examination is required, with special attention to the anterior segments.
Occasionally, increased sensitivity to light can directly result from a dilated
pupil, because more light is being let into that eye. Our patient has been
found to have anisocoria that is more prominent in bright light, suggesting
that the dilated left pupil is the abnormal pupil. Indeed, the left pupil shows
sluggish direct and consensual reactions to light, which likely explains her
increased sensitivity to light in that eye. However, the pupil shows a tonic
response to near stimuli. Together, these examination findings are highly
suggestive of a tonic pupil. Note that there is no ptosis or ophthalmoplegia
on the examination, making partial third nerve palsy very unlikely.
A tonic pupil is caused by a lesion that involves the postganglionic para-
sympathetic innervation of the pupil (i.e., ciliary ganglion or short ciliary
nerves in the retrobulbar space), resulting in palsies of the iris sphincter and
ciliary muscles. The classic signs of tonic pupil include poor pupillary reac-
tion to light, segmental palsy of the iris sphincter muscle, accommodation
palsy, and a tonic pupillary reaction to near. Segmental palsy of the iris
sphincter muscle is best appreciated on slit-lamp examination and occurs
because the causative lesion does not affect all of the postganglionic fibers
equally. Identification of segmental palsy on slit-lamp examination is
extremely important from a diagnostic perspective, because it does not
occur with preganglionic parasympathetic lesions (e.g., third nerve palsy;
see case 14) or pharmacologic mydriasis (see case 33). Segmental palsy can
occasionally be seen following iris trauma or ischemia, in which case there
will be iris atrophy and transillumination defects on slit-lamp examination.
The tonic response to near is another characteristic sign that is thought to
occur because there is aberrant reinnervation of the iris sphincter muscle by
fibers that previously innervated the ciliary muscle.
A tonic pupil can be caused by local factors, such as viral infections,
trauma, or surgery, or by a neurologic lesion (see Table 32-1). In a young
woman without any other medical problems or abnormal findings on exam-
ination, it is most likely to be an Adie’s pupil. Nevertheless, it would be

32. TONIC PUPIL 159


TABLE 32-1 Causes of Tonic Pupil

Local Neuropathic

Viral infections (e.g., zoster, syphilis) Syphilis


Orbital tumors Chronic alcoholism
Sarcoidosis Diabetes mellitus
Vasculitis (e.g., giant cell arteritis) Spinocerebellar ataxias
Trauma Guillain-Barré syndrome
Iatrogenic (e.g., orbital surgery) Miller Fisher syndrome

worthwhile to obtain further history. She should be asked if the anisocoria


has been noticed previously, and old photographs should be inspected.
She should be asked about a history of eye trauma or surgery. She should
have a slit-lamp examination looking for segmental palsy of the iris sphincter
muscle. If segmental palsy is identified, iris atrophy and transillumination
defects should be sought, because their presence suggests iris trauma or
ischemia as the cause for the mydriasis. The anterior segments should be
carefully assessed for findings that would suggest an alternative cause for
her monocular photosensitivity. Lastly, a neurologic examination should be
performed: the presence of hyporeflexia or areflexia suggests Holmes-Adie’s
syndrome or, if there is also ataxia or ophthalmoplegia, Miller Fisher
syndrome.
Adie’s pupil, the most likely cause of our patient’s tonic pupil, is sporadic
and of unknown etiology. It usually occurs in women between 20 and 50
years of age. It is unilateral in the majority of cases, but bilateral Adie’s
pupils can develop sequentially, sometimes years apart. The tonic pupil per-
sists indefinitely but gets progressively smaller with time.
Pharmacologic pupil testing can assist in the diagnosis of tonic pupil
from any cause (see Figure 30-1), because the denervated iris sphincter
muscle becomes supersensitive to cholinergic agents. Instillation of dilute
(0.1%) pilocarpine will result in intense constriction of a tonic pupil but
little if any effect on a normal or pharmacologically dilated pupil. A poten-
tial caveat with the dilute pilocarpine test is that denervation supersensitiv-
ity can also develop with preganglionic parasympathetic lesions (e.g., third
nerve palsy; see case 14). Consequently, there should be a low threshold for
close follow-up or for obtaining further investigations if the presentation is
atypical for tonic pupil.

160 WHAT DO I DO NOW? NEURO-OPHTHALMOLOGY


An Adie’s pupil does not require specific treatment. Our patient should
be reassured and advised to wear sunglasses when exposure to bright light is
expected. Dilute pilocarpine eye drops can be prescribed for use in situa-
tions where cosmetic appearance is important (e.g., job interviews or wed-
ding photographs), but they should not be used on a regular basis because
of the risk of ocular complications (e.g., retinal detachment or acute angle-
closure glaucoma). She should be advised that there is a small risk of her
eventually developing bilateral Adie’s pupils.

K EY P O I N TS TO R E M E M B E R

■ A tonic pupil shows poor reaction to light, with segmental iris


sphincter muscle palsy, and a tonic response to near, with a slow
subsequent redilation.
■ A tonic pupil can be caused by local disease or a neurologic lesion.
When idiopathic, it is known as an Adie’s pupil.
■ Instillation of dilute pilocarpine will produce intense constriction of
a tonic pupil, regardless of cause, but little if any constriction of a
normal or pharmacologically dilated pupil.
■ Given that dilute pilocarpine can constrict a dilated pupil due to
third nerve palsy, the patient should be carefully examined for
ptosis and ophthalmoplegia.
■ Tonic pupil does not require specific treatment, but use of
sunglasses can help to minimize light sensitivity.

Further Reading
Bourgon P, Pilley FJ, Thompson HS. Cholinergic supersensitivity of the iris sphincter in
Adie’s tonic pupil. Am J Ophthalmol. 1978;85:373–377.
Jacobson DM, Vierkant RA. Comparison of cholinergic supersensitivity in third nerve
palsy and Adie’s syndrome. J Neuroophthalmol. 1998;18:171–175.
Kardon RH, Corbett JJ, Thompson HS. Segmental denervation and reinnervation of the
iris sphincter as shown by infrared videographic transillumination. Ophthalmology.
1998;105:313–321.

32. TONIC PUPIL 161


33 Pharmacologic Mydriasis

You are called to see a 30-year-old female nurse who


noticed anisocoria when she looked in the mirror at the
end of a night shift. She has not had diplopia, ptosis, or
headaches, but her vision is blurred at near in the left
eye. Examination reveals distance visual acuities of
20/20 in both eyes. Color vision, confrontation visual
fields, eye movements, and eyelids are normal. There is
5 mm of anisocoria. The left pupil is larger than the right,
especially in bright light. The right pupil shows normal
reactions to light and near. However, the left pupil shows
no direct or consensual reaction to light, and no reaction
to near.

What do you do now?

162
T he patient in this case scenario has suddenly become aware of anisoco-
ria after looking into a mirror. The anisocoria is more prominent in
bright light, suggesting that the dilated left pupil is the abnormal pupil. Our
patient’s left pupil does not show a direct or consensual reaction to light or a
reaction to near. The differential diagnosis includes third nerve palsy, acute
tonic pupil, iris trauma or ischemia, and pharmacologic mydriasis.
To narrow the differential diagnosis, it would be worthwhile to obtain
further history. The patient should be asked if she has noticed the anisocoria
previously, and old photographs should be inspected. If the anisocoria is
not preexisting, the patient should be asked about a recent history of eye
trauma or surgery. Her recent medication usage may also be relevant, as the
mydriasis could be pharmacologic (e.g., from scopolamine patches or
inhaled bronchodilators). Pharmacologic agents that can cause mydriasis
include parasympatholytic (anticholinergic) and sympathomimetic (adren-
ergic) drugs (see Table 33-1). Several over-the-counter medications (e.g.,
pseudoephedrine) and illicit drugs (e.g., cocaine and amphetamines) can
cause bilateral pharmacologic mydriasis; exposure to these should be
inquired about in patients with bilateral mydriasis. Given that she is a nurse,
the mydriasis could be due to accidental or intentional instillation of a med-
ication into her eye (e.g., by inadvertently rubbing her eye after handling
the medication). A comprehensive list of the medications that she handled
or had access to during her shift must therefore be obtained.
Although a pupil examination has already been performed, a slit-lamp
examination should be performed to evaluate the iris. The presence of seg-
mental iris palsy is highly suggestive of a postganglionic parasympathetic
lesion (see case 32) but is rarely seen with a preganglionic lesion. Signs of

TABLE 33-1 Drugs that Commonly Cause Pharmacologic Mydriasis

Parasympatholytics (anticholinergics) Sympathomimetics (adrenergics)

Atropine Cocaine
Scopolamine Amphetamines
Homatropine Ephedrine
Tropicamide Pseudoephedrine
Cyclopentolate Phenylephrine
Ipratropium bromide Epinephrine

33. PHARMACOLOGIC MYDRIASIS 163


iris trauma or ischemia (e.g., sphincter muscle tear or transillumination
defects) should also be specifically sought. If the iris appears normal and
there is no segmental palsy, third nerve palsy and pharmacologic mydriasis
remain possible diagnoses. Although the patient does not have diplopia,
ptosis, or headache, and her eye movements are normal, her mydriasis could
be the initial sign of a compressive third nerve palsy (see case 14), because
the pupil fibers are located superficially on the third nerve and might be
the first fibers to be compressed by an enlarging lesion. However, a com-
pressive third nerve palsy whose only manifestation is a dilated nonreactive
pupil is rarely encountered; there are almost always other signs, such as
ptosis or ophthalmoplegia. Fortunately, pharmacologic pupil testing can
help to distinguish between the causes of mydriasis if there is uncertainty
(see Figure 30-1). A long-standing tonic pupil will typically show cholin-
ergic supersensitivity and constrict in response to dilute pilocarpine (see
case 32). Mydriasis due to third nerve palsy will constrict following instilla-
tion of 1% (and sometimes 0.1%) pilocarpine, whereas there will be little
or no effect on a pharmacologic pupil, unless it is mild or resolving. If phar-
macologic testing suggests that the mydriasis could be due to a third nerve
palsy, emergent imaging looking for a compressive lesion is required (see
case 14). No further investigation is required if the findings are consistent
with pharmacologic mydriasis.
Pharmacologic mydriasis does not require specific intervention and the
patient can be reassured. However, the patient should be warned that it may
take hours or even days for the mydriasis to resolve, depending on which
drug was instilled into the eye. The patient should also be advised on how
to avoid further episodes (e.g., washing hands after handling medications).

K EY P O I N TS TO R E M E M B E R

■ Pharmacologic mydriasis can be caused by accidental or intentional


instillation of parasympatholytic (anticholinergic) and
sympathomimetic (adrenergic) drugs into the eye.
■ Pharmacologic mydriasis is not usually reversed by 1% pilocarpine,
unless mild or resolving or caused by sympathomimetic drugs.
■ Pharmacologic mydriasis does not require specific intervention,
but can take hours to days to resolve.

164 WHAT DO I DO NOW? NEURO-OPHTHALMOLOGY


Further Reading
Bartleson JD, Trautmann JC, Sundt TM Jr. Minimal oculomotor nerve paresis secondary
to unruptured intracranial aneurysm. Arch Neurol. 1986;4:1015–1020.
Goldstein JB, Biousse V, Newman NJ. Unilateral pharmacologic mydriasis in a patient with
respiratory compromise. Arch Ophthalmol. 1997;115:806.
McCrary JA III, Webb NR. Anisocoria from scopolamine patches. JAMA. 1982;248:353–354.
Thompson HS, Newsome DA, Loewenfeld IE. The fixed dilated pupil: sudden iridoplegia or
mydriatic drops? A simple diagnostic test. Arch Ophthalmol. 1971;86:21–27.

33. PHARMACOLOGIC MYDRIASIS 165


This page intentionally left blank
SECTION V

COMBINATION SYNDROMES
This page intentionally left blank
34 Syndromes of the Orbital
Apex, Superior Orbital
Fissure, and Cavernous Sinus

A 42-year-old woman with poorly controlled type 1


diabetes presents to your clinic with a 1-day history of
painful vision loss in the right eye. Examination reveals
visual acuities of 20/400 in the right eye and 20/20 in
the left eye. Confrontation visual fields reveal a dense
central scotoma in the right eye. There is 2 mm of
anisocoria; the right pupil is larger than the left. The right
pupil reacts sluggishly to light and near, the left pupil
reacts normally to light and near, and there is a right
relative afferent pupillary defect. There is partial ptosis
and complete ophthalmoplegia on the right. Funduscopic
examination is unremarkable. The remainder of the
neurologic examination reveals decreased sensation over
the right forehead with an absent right corneal reflex.

What do you do now?

169
T he patient in this scenario has multiple cranial nerve palsies: a dense
central scotoma with a relative afferent pupillary defect indicates
involvement of the optic nerve; partial ptosis and complete external oph-
thalmoplegia with a sluggish dilated pupil indicates involvement of the
third, fourth, and sixth nerves; and decreased forehead sensation with an
absent corneal reflex indicates involvement of the ophthalmic division of
the fifth nerve. Together, these findings localize the lesion to the orbital apex
(see Table 34-1). Although a variety of disease processes can cause an orbital
apex syndrome (see Box 34-1), the rapid onset of symptoms and signs in a
patient with poorly controlled type 1 diabetes immediately suggests rhino-
orbital mucormycosis. Therefore, the patient requires immediate admission
to hospital for urgent investigations and aggressive treatment.
Mucormycosis can be caused by a number of different fungi of the order
Mucorales, which are commonly found in decaying organic matter. It
occurs most commonly in diabetics with ketoacidosis, but it can also occur
in patients who are neutropenic, immunocompromised for solid organ or
stem cell transplantation, or receiving deferoxamine therapy (e.g., for hemo-
chromatosis or thalassemia). It rarely occurs in immunocompetent indi-
viduals. It arises from the paranasal sinuses, where fungal hyphae invade the
sinus mucosa and spread to contiguous structures via blood vessels or nerves.

TABLE 34-1 Structures Involved in Syndromes of the Orbital Apex, Superior


Orbital Fissure, and Cavernous Sinus

Structure Orbital apex Superior orbital fissure Cavernous sinus

CN II + – –a

CN III + + +

CN IV + + +

CN V1 + + +

CN V2 – – +

CN VI + + +

Carotid artery – – +

Abbreviations: CN, cranial nerve; +, involved; –, not involved.


a
Can be indirectly involved (e.g., as a result of ocular hypertension with arteriovenous fistula).

170 WHAT DO I DO NOW? NEURO-OPHTHALMOLOGY


BOX 3 4-1 CAUS ES O F SY N D RO M ES O F T H E O RBI TA L A P E X,
S UPER IO R O RBI TA L F I SS U RE , A N D CAVE R NO U S S I NU S

Inflammation (e.g., idiopathic orbital inflammation, sarcoidosis,


Tolosa-Hunt syndrome)
Infection (e.g., mucormycosis, invasive aspergillosis, herpes zoster,
tuberculosis)
Neoplasm (e.g., lymphoma, nasopharyngeal carcinoma, meningioma,
metastases)
Vascular (e.g., cavernous sinus thrombosis, carotid aneurysm,
carotid–cavernous fistula)
Iatrogenic (e.g., endoscopic sinus surgery, orbital surgery, facial
surgery)
Trauma (e.g., penetrating injury, orbital fracture, retained foreign
body)

Angioinvasion can lead to thrombosis with infarction of distal structures


(see Figure 34-1) or hematogenous dissemination. When the disease is
localized to the sinuses, the symptoms and signs are nonspecific and include
facial pain, nasal congestion, and bloody nasal discharge. When the infec-
tion invades the orbit, there may be a partial or complete orbital apex syn-
drome (see Table 34-1). Often, there are other signs to indicate orbital
involvement, such as periorbital edema (see Figure 34-1), proptosis, and
chemosis; these signs should be specifically sought in our patient. The infec-
tion can spread from the orbit into the cavernous sinus to cause cavernous
sinus thrombosis. The cavernous segment of the internal carotid artery can
also become affected as a result of the spread of hyphae within the blood
vessel lumen, leading to occlusion and cerebral infarction. Other intracra-
nial complications include epidural and subdural abscess, venous sinus
thrombosis, and meningitis. Once the process has spread beyond the sinuses
and orbit, the prognosis is extremely poor.
Accurate and timely diagnosis of mucormycosis requires a high index of
suspicion in predisposed patients. Computed tomography (CT) or mag-
netic resonance imaging (MRI) should be obtained urgently in our patient,
looking for signs of paranasal sinus or orbital involvement. Characteristic
findings include sinus mucosal thickening, sinus opacification, bony erosion

34. SYNDROMES OF ORBITAL APEX, SUPERIOR ORBITAL FISSURE, & CAVERNOUS SINUS 171
FIGURE 34-1 Clinical and radiologic features of mucormycosis in a patient with diabetic
ketoacidosis. Periorbital edema and facial infarction due to angioinvasion with thrombosis
(top, left). Opacification of maxillary and ethmoid sinuses with enlarged right extraocular
muscle bellies on computed tomography (CT) (top, right). Right-sided proptosis with orbital
apex enhancement and enlarged extraocular muscle bellies on magnetic resonance imaging
(MRI) (bottom, left). Occlusion of left internal carotid artery on magnetic resonance
angiography (MRA) (bottom, right).

(only evident on CT), orbital apex enhancement, and enlargement of the


extraocular muscle bellies (see Figure 34-1). There may be cavernous sinus
thrombosis, occlusion of the internal carotid artery, cerebral infarction, or,
once the disease has spread into the cranial cavity, signs of meningitis (see
Figure 34-1). However, an important caveat is that the imaging findings
can initially be subtle or absent despite a dramatic clinical presentation.
Nasal endoscopy with biopsies of sinus mucosa should be obtained urgently
in our patient, because definitive diagnosis requires histopathologic docu-
mentation of fungal invasion. Debridement of necrotic mucosa can be

172 WHAT DO I DO NOW? NEURO-OPHTHALMOLOGY


performed during the same procedure. If the sinus biopsies are unrevealing
and the clinical suspicion remains high, orbital apex biopsy could be con-
sidered, although there is a considerable risk of complications (e.g., vision
loss) with this procedure.
Successful treatment of our patient will depend on rapid reversal of pre-
disposing factors, immediate initiation of empiric antifungal treatment,
and early surgical debridement of infected tissue. The most likely predispos-
ing factor in our patient is diabetic ketoacidosis, which can be quickly diag-
nosed on the basis of blood chemistry results. Diabetic ketoacidosis should
be aggressively treated with intravenous hydration and insulin infusion,
preferably in an intensive care setting. Empiric antifungal treatment should
be commenced in our patient without waiting for confirmation of the diag-
nosis. Liposomal amphotericin B is the agent of choice, although use of
posaconazole can be considered in refractory disease. Unfortunately, the
mortality rate with antifungal treatment alone remains very high (about
70%), and therefore radical surgical debridement should be considered
early in patients with limited disease, to prevent the spread of infection to
adjacent structures. The optimal extent and timing of surgical debridement
remain uncertain. Several adjunctive therapies could also be considered in
our patient, including hyperbaric oxygen and granulocyte–macrophage
colony-stimulating factor, although the evidence for their efficacy remains
circumstantial.

K EY P O I N TS TO R E M E M B E R

■ Mucormycosis can cause an acute-onset orbital apex syndrome in


immunocompromised patients (e.g., neutropenic or transplant
patients) or diabetic patients with ketoacidosis.
■ Urgent investigations, including imaging, nasal endoscopy, and
mucosal biopsy, are required to make the diagnosis.
■ Imaging changes can initially be subtle or absent despite a dramatic
clinical presentation.
■ Successful treatment requires rapid reversal of predisposing
factors, immediate initiation of empiric antifungal therapy
(e.g., amphotericin B), and early radical surgical debridement.

34. SYNDROMES OF ORBITAL APEX, SUPERIOR ORBITAL FISSURE, & CAVERNOUS SINUS 173
Further Reading
Keane JR. Cavernous sinus syndrome: analysis of 151 cases. Arch Neurol. 1996;53:967–971.
Parikh SL, Venkatraman G, DelGaudio JM. Invasive fungal sinusitis: a 15-year review from
a single institution. Am J Rhinol. 2004;18:75–81.
Spellberg B, Edwards J Jr, Ibrahim A. Novel perspectives on mucormycosis:
pathophysiology, presentation, and management. Clin Microbiol Rev. 2005;18:
556–569.
Yeh S, Foroozan R. Orbital apex syndrome. Curr Opin Ophthalmol. 2004;15:490–498.

174 WHAT DO I DO NOW? NEURO-OPHTHALMOLOGY


35 Dorsal Midbrain Syndrome

You are called to evaluate a 24-year-old woman with a


prior history of ventriculoperitoneal shunting for
congenital aqueduct stenosis who has presented to the
emergency department with headaches. She has not had
vision loss, diplopia, or ptosis. Examination reveals
normal visual acuities, color vision, confrontation visual
fields, and fundi. Her pupils are equal in size. They show
sluggish direct and consensual reactions to light, but
constrict briskly when she converges. She also has
bilateral upper-eyelid retraction. She has limited upward
gaze and develops convergence–retraction nystagmus
when she makes upward saccades.

What do you do now?

175
D orsal midbrain lesions give rise to a characteristic syndrome of ocular
motor, eyelid, and pupil abnormalities (see Figure 35-1); this is some-
times called Parinaud’s syndrome but is better termed the dorsal midbrain
syndrome. The classic ocular motor signs of the dorsal midbrain syndrome
include supranuclear vertical gaze palsy, skew deviation, convergence insuf-
ficiency, and convergence–retraction nystagmus. The supranuclear vertical
gaze palsy is due to involvement of the posterior commissure and is worse
for upward eye movements, although downward eye movements can also be
affected. There can sometimes be a tonic downward deviation of the eyes
(the “setting sun” sign; see Figure 35-1), especially in children with an acute
dorsal midbrain syndrome. Skew deviation results from involvement of the
central otolithic pathways. Convergence–retraction nystagmus is highly
localizing to the dorsal midbrain, is characterized by rhythmic convergent–
retraction eye movements during attempted upward gaze, and is best elic-
ited by asking the patient to follow the downward moving stripes on an
optokinetic drum. Eyelid abnormalities include upper-eyelid retraction
(Collier’s sign; see Figure 35-1) and, less often, upper-eyelid ptosis. The
pupils are often mid-dilated and show light–near dissociation, in which
there is a poor pupillary response to light but brisk response to near.
Common causes of the dorsal midbrain syndrome include hydrocepha-
lus, stroke, intrinsic brainstem tumors, and compression by extrinsic
lesions (e.g., pinealomas, germinomas, and third-ventricular tumors).
Malfunction of a ventriculoperitoneal shunt can produce a dorsal midbrain
syndrome as well as other symptoms and signs of intracranial hypertension,
such as headaches, vomiting, and papilledema (see case 6). In our patient,
the first step is to image the brain to determine if she has hydrocephalus due

FIGURE 35-1 Signs of the dorsal midbrain syndrome, including downward eye deviation, skew
deviation, and upper-eyelid retraction (Collier’s sign), following a dorsal midbrain hemorrhage.
The patient developed convergence–retraction nystagmus during attempted upward saccades.

176 WHAT DO I DO NOW? NEURO-OPHTHALMOLOGY


to shunt malfunction; unless contraindicated, magnetic resonance imaging
(MRI) is preferred for the initial investigation of a patient with the dorsal
midbrain syndrome. While the presence of hydrocephalus is a clear indica-
tor of shunt malfunction, the absence of hydrocephalus does not exclude it;
a dorsal midbrain syndrome can sometimes occur without hydrocephalus
in a patient with shunt malfunction. Consequently, there should be a low
threshold for obtaining urgent shunt function studies and for requesting
neurosurgical consultation when the clinical suspicion for shunt malfunc-
tion is high, as in our patient.
Neurosurgical consultation and intervention is indicated when there is
concern for ventriculoperitoneal shunt malfunction, because medical treat-
ments to reduce intracranial pressure are unlikely to be effective. In many
cases, shunt revision is required and will usually lead to a prompt resolution
of the dorsal midbrain syndrome as well as the other symptoms and signs of
intracranial hypertension.

K EY P O I N TS TO R E M E M B E R

■ Dorsal midbrain syndrome is characterized by supranuclear vertical


gaze palsy, skew deviation, convergence–retraction nystagmus,
eyelid retraction, and light–near dissociation of the pupils.
■ Common causes of dorsal midbrain syndrome include
hydrocephalus, stroke, intrinsic brainstem tumors, and compression
by extrinsic lesions (e.g., pinealomas).
■ Ventriculoperitoneal shunt malfunction can cause a dorsal midbrain
syndrome or other symptoms and signs of intracranial
hypertension, sometimes without hydrocephalus.
■ Shunt function studies and neurosurgical consultation should be
urgently obtained when there is evidence of shunt malfunction or
when clinical suspicion for shunt malfunction is high.

Further Reading
Chou SY, Digre KB. Neuro-ophthalmic complications of raised intracranial pressure,
hydrocephalus, and shunt malfunction. Neurosurg Clin N Am. 1999;10:587–608.
Katz DM, Trobe JD, Muraszko KM, Dauser RC. Shunt failure without ventriculomegaly
proclaimed by ophthalmic findings. J Neurosurg. 1994;81:721–725.
Keane JR. The pretectal syndrome: 206 patients. Neurology. 1990;40:684–690.

35. DORSAL MIDBRAIN SYNDROME 177


36 Thyroid Eye Disease

A 52-year-old woman presents with a several-month


history of increasing binocular diagonal diplopia. She has
an unremarkable medical history but is a heavy smoker.
Examination reveals visual acuities of 20/30 in both eyes.
Color vision, confrontation visual fields, and pupils are
normal. Slit-lamp examination reveals conjunctival
injection and punctate corneal epithelial erosions in both
eyes. She has mild bilateral upper-eyelid retraction and
proptosis. There is an esotropia and right hypotropia, and
she has limited abduction and elevation of her right eye.
Funduscopic examination is normal.

What do you do now?

178
T he clinical presentation in this scenario is highly suggestive of thyroid
eye disease (TED; also known as thyroid-associated or Graves’ ophthal-
mopathy). TED is the most common orbital disease encountered in clinical
practice and is most often associated with Graves’ disease. It is typically
bilateral and painless, although it can be asymmetric and associated with
eye discomfort. The inferior and medial rectus muscles are the most com-
monly affected extraocular muscles (see Figure 36-1), resulting in restric-
tion of eye elevation and abduction. Consequently, patients with TED
often present with binocular diplopia. The diplopia is typically worse in the
morning, because of dependent engorgement of the extraocular muscles.
Other signs of orbital disease are typically present and include axial propto-
sis, periorbital edema, conjunctival injection and chemosis, eyelid retrac-
tion, lagophthalmos (inability to close the eye), lid lag (higher eyelid
position than normal in downgaze), and von Graefe’s sign (slowed descent
of the eyelid during movement from upgaze to downgaze) (Gaddipati et al.,
2008). The combination of eyelid retraction and lagophthalmos can result
in decreased visual acuity due to exposure keratopathy, of which punctate
corneal epithelial erosions are a characteristic finding. Vision loss can result
from optic nerve compression at the orbital apex by the enlarged extraocu-
lar muscle bellies (see case 4) and may occur in the absence of significant
proptosis. The vision loss is typically insidious and may not be noticed by
the patient until severe. Dyschromatopsia, often out of proportion to the
amount of vision loss, is a common clinical finding in patients with com-
pressive optic neuropathy due to TED.

FIGURE 36-1 Computed tomography (CT) scan showing enlargement of the extraocular
muscle bellies (most marked for the medial and inferior rectus muscles) and proptosis in
thyroid eye disease.

36. THYROID EYE DISEASE 179


TABLE 36-1 Grading of Thyroid Eye Disease (TED)a

Examination finding Mild TED Moderate-to-severe TED

Eyelid retraction (mm) <2 ≥2

Proptosis (mm) <3 ≥3

Soft tissue involvement Mild Moderate to severe

Diplopia None or intermittent Intermittent or constant

Corneal involvement Absent or mild Moderate

a
Adapted from Bartalena L, Tanda ML. Graves’ ophthalmopathy. N Engl J Med. 2009;360:994–1001.

TED is diagnosed and graded on the basis of clinical findings (see


Table 36-1). However, imaging of the orbits is the most useful investigation
to obtain when the diagnosis is suspected. Axial and coronal computed
tomography (CT) or magnetic resonance imaging (MRI) of the orbit can
be obtained and usually reveals characteristic enlargement of the extraocular
muscle bellies with relative sparing of the tendons (see Figure 36-1). MRI is
sometimes preferred, because there is no radiation dose to the lens, whereas
CT is favored when details of the bony anatomy are required (e.g., prior to
orbital decompression surgery). Thyroid function tests (thyroid-stimulating
hormone, T3, and T4 levels) should be obtained in all patients with TED,
because they can have undiagnosed hyperthyroidism or hypothyroidism.
Thyroid-stimulating antibodies are often present in patients with TED and
can be an important marker of disease in euthyroid patients. Findings from
several recent studies (e.g., de Bellis et al., 2004) have suggested that
extraocular muscle antibodies are a more specific serologic marker of both
disease presence and activity, but assays for these are not yet commercially
available. Visual field testing should be obtained in any patient with TED
who has vision loss. Our patient’s visual acuities are decreased to 20/30 in
both eyes, but given the findings on slit-lamp examination and absence of
other signs to suggest optic neuropathy, this is likely due to exposure ker-
atopathy. Nevertheless, visual field testing should be considered to exclude
a visual field defect from compressive optic neuropathy, especially if there is
crowding at the orbital apex on imaging.

180 WHAT DO I DO NOW? NEURO-OPHTHALMOLOGY


Most alternative diagnoses can be excluded on the basis of imaging find-
ings. However, when there are atypical features (e.g., exotropia rather than
esotropia or ptosis rather than eyelid retraction), the possibility of coexisting
ocular myasthenia should be specifically considered (see cases 17 and 18).
Many patients with TED have only mild disease and do not require
specific treatment. Supportive treatments, such as artificial tears for expo-
sure keratopathy and prisms for diplopia, can improve symptoms and qual-
ity of life. In patients with moderate to severe TED, treatment options
include immunosuppression with systemic corticosteroids, orbital irradia-
tion, and orbital decompression surgery. Orbital decompression surgery is
the treatment of choice in patients with vision loss due to compressive optic
neuropathy, although some clinicians prefer a trial of high-dose intravenous
steroids before recommending surgery. The choice of treatment in our
patient depends on the severity of the clinical signs (see Table 36-1) and
whether or not there is compressive optic neuropathy. She should be
prescribed artificial tears for exposure keratopathy; consultation with a
cornea specialist could be considered if the exposure is severe or does not
respond to simple measures. If she has significant ocular misalignment or
eyelid retraction, strabismus surgery or eyelid surgery could be offered fol-
lowing orbital decompression surgery or once the disease process has stabi-
lized. Radioactive iodine therapy, which is often given to patients with
hyperthyroidism due to Graves’ disease, should be avoided in our patient,
as it can worsen TED. Our patient should also be advised to discontinue
smoking, because it can worsen TED and decrease the beneficial effects of
other treatments.

K EY P O I N TS TO R E M E M B E R

■ TED is the most common orbital disease encountered in clinical


practice.
■ Signs of TED include ophthalmoplegia, proptosis, periorbital edema,
conjunctival injection, chemosis, eyelid retraction, lagophthalmos,
lid lag, and von Graefe’s sign.
■ MRI or CT of the orbits typically reveals enlargement of the
extraocular muscle bellies with relative sparing of the tendons.

36. THYROID EYE DISEASE 181


■ Severe TED can be treated with systemic corticosteroids, orbital
irradiation, or orbital decompression surgery.
■ Diplopia in TED can be managed with prisms for small deviations
and strabismus surgery for larger deviations.

Further Reading
Bartalena L, Tanda ML. Graves’ ophthalmopathy. N Engl J Med. 2009;360:994–1001.
de Bellis A, Perrino S, Coronella C, et al. Extraocular muscle antibodies and the
occurrence of ophthalmopathy in Graves’ disease. Clin Endocrinol (Oxf).
2004;60:694–698.
Gaddipati RV, Meyer DR. Eyelid retraction, lid lag, lagophthalmos, and von Graefe’s sign:
quantifying the eyelid features of Graves’ ophthalmopathy. Ophthalmology.
2008;115:1083–1088.
Thornton J, Kelly SP, Harrison RA, Edwards R. Cigarette smoking and thyroid eye disease:
a systematic review. Eye (Lond). 2007;21:1135–1145.

182 WHAT DO I DO NOW? NEURO-OPHTHALMOLOGY


Index

Note: Page numbers followed by “f ” and “t ” denote figures and tables, respectively.

AAION. See arteritic anterior ischemic anticoagulation, with warfarin, 61


optic neuropathy anticonvulsant medications, 111–13
abduction deficit, 85, 86 anti-Ma2 brainstem encephalitis, 108
aberrant regeneration, 76 APN. See acquired pendular nystagmus
acetazolamide, 34, 129 apraclonidine test, for Horner's syndrome,
acetylcholine-receptor 154–55, 157
antibodies, 89, 90t, 92 apraxia of eyelid opening (AEO), 142
acquired pendular nystagmus (APN), arterial occlusion, 58, 60, 60f, 61
122–25 arteritic anterior ischemic optic neuropathy
with MS, 122, 123, 124 (AAION), 11–15, 18
neural integrator and, 123 ataxia
pathogenesis of, 123 Friedreich's, 132
with strokes, 124 optic, 54t
acute hypoadrenalism, 46 with third nerve palsy, 76
acute isolated Horner's syndrome, 156, 157 atropine, 90
adduction deficits, 103, 104
Adie's pupil, 159–60, 161 baclofen, 120, 121
AEO. See apraxia of eyelid opening bean-shaped scotoma, 64, 66
akinetic-rigid syndrome, 107, 108 benign essential blepharospasm (BEB),
alexia, 54t 141–44
altitudinal onset, of transient visual loss, 58 etiology of, 142
Alzheimer's disease, 53, 55, 56. See also injections for, 143
visual variant of Alzheimer's disease photophobia with, 142–43
4-aminopyridine, 116, 117, 120, 121 beta-interferon, 8, 125
amiodarone, 18 bilateral adduction deficits, 102, 103
amitriptyline, 66 binocular oscillopsia, 99
aneurysm bitemporal hemianopia, 43, 44, 45f, 47
compression by, 76, 77–78, 77f bitemporal visual field defects, 43, 44, 45f, 47
of internal carotid artery, 44, 47 blepharospasm, 141–44. See also benign
angioinvasion, 171 essential blepharospasm
anisocoria, 148–51, 149f. See also β-blockers, 100
physiologic anisocoria botulinum toxin A, 143
pharmacologic mydriasis causing, brainstem stroke, 94, 94t, 96, 103,
162, 163 105, 133
reversal of, 155, 157 bulbar palsy, 119
tonic pupil causing, 158, 159, 160
anterior compressive optic neuropathy, 22, 22f carbamazepine, 100, 111
anterior ischemic optic neuropathy, 11–20 carotid artery stenting, 61

183
carotid Doppler ultrasound, for transient computed tomographic angiography
visual loss, 60 (CTA), 60, 77, 78
carotid endarterectomy, 61 concentric onset, of transient visual loss, 58
catheter angiography, 77 conjunctival injection, 23, 178, 179
cavernous sinus. See orbital apex, superior contralateral hemiparesis, 76
orbital fissure, cavernous sinus, convergence-retraction nystagmus, 175–77
syndromes of cortical blindness, 54t
central scotoma, 5–6, 21–22, 25–26, CPEO. See chronic progressive external
169–70 ophthalmoplegia
central visual field defect, 6, 22, 44 cranial nerve palsies, 75–87, 169–74
cerebellar degeneration, 115, 116, 117, craniopharyngioma, 44, 47
120, 133, 134 Creutzfeldt-Jakob disease, 108
cerebral achromatopsia, 53, 54t CSF. See cerebrospinal fluid
cerebral akinetopsia, 54t CT. See computed tomography
cerebrospinal fluid (CSF), 8, 31, 34, 35 CTA. See computed tomographic
chemosis, 22, 171, 179, 181 angiography
Chiari malformation, 103, 115
chiasmal syndromes, 43–47, 45f decompression
choroidal ischemia, 13f, 14 for pituitary apoplexy, 46, 47
chronic progressive external for superior oblique myokymia, 100
ophthalmoplegia (CPEO), 93–97 demyelination, 6, 9, 103, 105
cilioretinal ischemia, 13f, 14 dermatochalasis, 138
clonazepam, 116, 117 3,4-diaminopyridine, 116, 117
cocaine test, for Horner's syndrome, 154 differential diagnosis
Cogan's lid twitch, 89, 139 of complete bilateral external
complete bilateral external ophthalmoplegia, 94
ophthalmoplegia, 93–97 of papilledema, 37
causes of, 94–95, 94t of pharmacologic mydriasis, 163
differential diagnosis of, 94–95 of physiologic anisocoria, 150, 151
compression dilute pilocarpine, for tonic pupil, 160, 161
by aneurysm, 76, 77f, 78 diplopia, 92, 96, 105, 148
by neoplasm, 76, 78 binocular, 88, 89
optic chiasm, 43–47 diagonal, 75, 178
optic nerve, 21–24 horizontal, 84
compressive optic neuropathy, 21–24 vertical, 79
anterior, 22, 22f torsional, 80
causes of, 23 transient, 13, 102
MRI for, 21, 22f, 23, 24 "disc at risk," 17–18
posterior, 22 disorders, of higher visual function, 52–56
computed tomography (CT), 23, 24, 41 alexia, 54t
for mucormycosis, 171 cerebral achromatopsia, 54t
for optic chiasm compression, 44 cerebral akinetopsia, 54t
for pituitary apoplexy, 46 cortical blindness, 54t
for TED, 179f, 180, 181 neurodegenerative disease as cause of, 54

184 INDEX
ocular motor apraxia, 54t esotropia, 85, 86, 178
optic ataxia, 54t etiology
palinopsia, 54t of BEB, 142
prosopagnosia, 53, 54t of fourth nerve palsy, 80–81
simultanagnosia, 53, 54t of sixth nerve palsy, 85, 86
visual agnosia, 53 of third nerve palsy, 76
visual hallucinations, 54t of TVL, 58–60, 61
dominant optic atrophy (DOA), 26 eyelid
dopaminergic agents, 108 ptosis of, 137–40
dorsal midbrain syndrome, 175–77, 176f causes of, 138
causes of, 176–77 systemic diseases causing, 138
convergence-retraction nystagmus trauma causing, 139
with, 175, 176 retraction of, 106, 107, 175, 176, 176f,
hydrocephalus causing, 177 177, 178, 179, 180t, 181
shunt malfunction causing, 175–77 surgery of, 96, 140
upper-eyelid ptosis with, 176
upper-eyelid retraction with, 176 facial anhidrosis, 154
downbeat nystagmus, 114–17 fatigable muscle weakness, 89
causes of, 115 flocculonodular lobe, lesions of, 115, 117
cerebellar degenerations causing, 115, 117 fluorescein angiography, 14
hindbrain anomalies causing, 115, 117 fogging, 69
medications for, 116 fortification spectrum, of migraine aura, 64
drugs, pharmacologic mydriasis caused by, fourth nerve palsy, 79–83
163, 163t, 164 diagnostic tests for, 81–82
dyschromatopsia, 6, 22, 179 etiology of, 80, 81
dysmetria, 131–34 GCA causing, 80
head trauma causing, 80, 81
echocardiography, for transient visual Parks-Bielschowsky three-step test for,
loss, 60–61 80–81, 81t
edema. See optic disc edema vascular risk factors with, 80
edrophonium, 90, 92 fourth nerve schwannomas, 81, 82f
electrocardiographic monitoring, for fourth-ventricular tumors, 103, 105
transient visual loss, 61 Friedreich's ataxia, 132
electroretinogram testing, for infantile FRMD7 gene, 127
nystagmus syndrome, 128
elevation, of optic nerve head, 36, 37, 39, 41 gabapentin, 100, 124, 125, 128
elliptical eye oscillations, 122, 123 galactorrhea, 43
embolic arterial occlusion, 58, 60, 60f gaze-evoked nystagmus, 110–13
embolism, with transient visual loss, 58, causes of, 111–12
60, 61 genetic testing for, 112
encephalitis, 53, 108 medications as cause of, 111–12
endocrine evaluation, of infantile nystagmus physiologic end-point nystagmus
syndrome, 128 compared to, 111, 113
epilepsy, 110, 112 GCA. See giant cell arteritis

INDEX 185
genetic testing, for gaze-evoked ice test, for ocular myasthenia, 90, 90t,
nystagmus, 112 91f, 92
giant cell arteritis (GCA), 11–15, 13f, ictal phenomena, 66
58, 80 idiopathic intracranial hypertension (IIH),
glioma, 44, 47 29–35, 31f, 33f
Goldmann perimetry, 70 cerebral venous stenosis with, 34
Guillain-Barré syndrome, 94, 94t, 96 diagnostic criteria for, 33
idiopathic optic neuritis, 5–10, 7f, 26
headaches, 29, 36, 37. See also migraine IHS. See International Headache Society
aura; retinal migraine IIH. See idiopathic intracranial
with dorsal midbrain syndrome, 175 hypertension
global, 43 incomplete third nerve palsy, 76
temporal, 11, 12, 13 infantile nystagmus syndrome (INS),
third nerve palsy with, 75 126–30
head tilt, with fourth nerve palsy, 80 conditions associated with, 127
head trauma, fourth nerve palsy and, 80, 81 electroretinogram testing for, 128
Heimann-Bielschowsky phenomenon, 99 endocrine evaluation for, 128
hemianopia. See also homonymous neuro-ophthalmic assessment of, 128
hemianopia surgical treatments for, 129
bitemporal, 43, 44, 45f, 47 X-linked, 127
hemispheric transient ischemic attacks, 61 infarction, of pituitary macroadenoma,
hereditary optic neuropathy, 25–28 44, 47
higher visual function, disorders of, 52–56 inferior altitudinal defect, 16, 17
high-grade internal carotid artery stenosis, injection
61 Botulinum toxin A for BEB, 143
hindbrain anomalies, 103, 105, 115, 117, conjunctival, 23, 178, 179
133 INO. See internuclear ophthalmoplegia
Holmes-Adie's syndrome, 160 INS. See infantile nystagmus syndrome
homonymous hemianopia, 48–51 intermittent leg claudication, 61
prognosis for, 49 internal carotid artery
testing for, 49–50 aneurysms of, 44, 47
homonymous visual field defects, 48–51, 53 dissection of, 156, 156f
horizontal saccades, 131, 132 International Headache Society (IHS), 64
Horner's syndrome, 139, 148t, 149f, 150, internuclear ophthalmoplegia (INO), 89,
152–57, 153f 102–5
acute isolated, 156, 157 intravenous corticosteroids, 46
apraclonidine test for, 154–55, 157 intravenous methylprednisone, 8, 9, 14
causes of, 155t iris dilator muscle, 148, 153
cocaine test for, 154 iris sphincter muscle, 148, 153–54, 160,
hydrocephalus, dorsal midbrain syndrome 161
and, 177 ischemia. See also microvascular ischemia
hyperemic optic disc edema, 12 choroidal, 13f, 14
hypermetria, 132 cilioretinal, 13f, 14
hypertropia, with fourth nerve palsy, 80, 83 vertebrobasilar, 64

186 INDEX
ischemic optic neuropathy. See also arteritic for disorders of higher visual function,
anterior ischemic optic neuropathy; 54, 55, 55f
nonarteritic anterior ischemic optic for dorsal midbrain syndrome, 177
neuropathy for homonymous hemianopia, 49
anterior, 11–20 for Horner's syndrome, 156, 156f, 157
posterior, 12 for idiopathic intracranial
isolated transient visual loss, 61 hypertension, 31
for INO, 103–4, 104f
junctional scotoma, 44 for migraine aura, 65f
for mucormycosis, 171, 172f
lateral rectus weakness, 85 for nystagmus
Leber's hereditary optic neuropathy downbeat, 115
(LHON), 25–28 INS, 128
lesions upbeat, 120
chiasmal, 45f for optic chiasm compression, 44, 46f
with eyelid ptosis, 138 for optic neuropathy
of flocculonodular lobe, 115, 117 compressive, 22f, 23, 24
with Horner's syndrome, 153, hereditary, 26
155–56, 157 for pituitary apoplexy, 46
medial medullary, 119 for TED, 180, 181
nuclear-infranuclear, 94, 96, 107 for third nerve palsy, 77
with nystagmus magnetic resonance venography
downbeat, 115 (MRV), 31, 34
upbeat, 119 medial longitudinal fasciculus (MLF), 103
in occipital lobe, 48, 49, 64 medial medullary lesion, 119
in optic tract, 49 medications
with sixth nerve palsy, 85, 86 anisocoria and, 148, 148t, 162–65
supranuclear, 94, 107 anticonvulsant, 111–12, 113
with syndromes of orbital apex, gaze-evoked nystagmus
superior orbital fissure, cavernous caused by, 111–13
sinus, 170, 171 NAION caused by, 18
with third nerve palsy, 76 for nystagmus, 116, 120–21, 124, 125,
with tonic pupil, 159 128–29
white matter, 7, 7f, 8 for ocular myasthenia, 92
levator dehiscence, 137–40, 150 Meige's syndrome, 142
levator function, 138–39, 140 memantine, 120, 121, 124, 125,
LHON. See Leber's hereditary optic 128, 134
neuropathy meningiomas
lumbar puncture, 31, 32 ONSM, 22f, 23, 24
suprasellar, 44, 47
macrosaccadic oscillations, 132f, 133, 134 meningism, 44
magnetic resonance angiography (MRA), microvascular ischemia
60, 65f, 77, 77f, 78, 156, 157, 172f causing sixth nerve palsy, 85, 86
magnetic resonance imaging (MRI) causing third nerve palsy, 76, 78

INDEX 187
microvascular third nerve palsy, 76, 78 nonarteritic anterior ischemic optic
midbrain atrophy, 107 neuropathy (NAION), 12, 14, 16–20,
migraine aura, 63–67, 65t 18f, 26
fortification spectrum of, 64 medications causing, 18
scintillating zigzag edge with, 64, optic disc edema and, 12, 16, 17, 18f
65t, 66 pathogenesis of, 17–18
headache with, 64 steroids for, 19
nausea with, 64 nonorganic vision loss, 68–71
phonophobia with, 64 fogging for, 69
photophobia with, 64 Goldmann perimetry for, 70
vomiting with, 64 maneuvers for, 69–70
transient visual disturbance optokinetic nystagmus for, 69
caused by, 64 organic disease with, 69, 70
Miller Fisher syndrome, 81, 94, 94t, refraction for, 69
96, 160 stereopsis for, 69
mitochondrial DNA, 26–27, 96 tangent screen for, 70
mitochondrial myopathies, 94t, 95–96 North American Symptomatic Carotid
MLF. See medial longitudinal fasciculus Endarterectomy Trial, 61
monocular nystagmus, 99, 100 nuclear-infranuclear lesion, 94, 96, 107
monocular oscillopsia, 99, 100 nystagmus. See also acquired pendular
monocular vision loss, 6, 12, 13, 58, 59t, nystagmus; downbeat nystagmus;
60, 60f, 61, 69 gaze-evoked nystagmus; infantile
MRA. See magnetic resonance nystagmus syndrome; upbeat nystagmus
angiography convergence-retraction, 175, 176, 177
MRI. See magnetic resonance imaging optokinetic, 69
MRV. See magnetic resonance venography
mucormycosis, 169–74, 172f occipital infarct, 49, 50f
multiple sclerosis (MS), 6, 7, 8, 133 occipital lobe, lesion in, 48, 49, 64
with APN, 122–25 occipital seizures, 64, 65t, 66, 67
with gaze-evoked nystagmus, 112, 113 occipital tumor, 49
with internuclear ophthalmoplegia, ocular dysmetria, 131, 132
103–4, 105 ocular flutter, 132, 133
with sixth nerve palsy, 85, 86 ocular motor apraxia, 54t
myasthenia gravis, 89–92, 90t, 91f, 94, 94t, ocular myasthenia, 88–92
95, 96 diagnostic tests for, 89–91, 90t, 91f, 92
medications for, 92
NAION. See nonarteritic anterior ischemic oculopalatal tremor (OPT), 123–24
optic neuropathy oculosympathetic pathway, 153, 155, 157
nausea, with migraine aura, 64 ONHD. See optic nerve head drusen
negative visual phenomena, 64 ONSF. See optic nerve sheath fenestration
neoplasm ONSM. See optic nerve sheath meningioma
with sixth nerve palsy, 85, 86 ophthalmoplegia, 44, 169. See also complete
with third nerve palsy, 76, 78 bilateral external ophthalmoplegia
neural integrator, 111, 123 CPEO, 93–97

188 INDEX
internuclear, 89, 102–5 organic disease, nonorganic vision
opsoclonus, 132, 133 loss with, 69, 70
OPT. See oculopalatal tremor oromandibular dystonia, 142
optic ataxia, 54t oscillations
optic chiasm, 44 macrosaccadic, 132, 132f,
optic disc edema, 6, 11, 12, 13f, 14, 16, 133, 134
17f, 18, 22, 22f, 25, 26, 29, 30 oscillopsia
bilateral, 29 binocular, 99
hyperemic, 12 monocular, 99, 100
NAION and, 16, 17f, 18 vertical, 114, 117, 118, 119
pallid, 11, 12, 13f
optic nerve compression, 21–24, 26 pain
optic nerve head drusen (ONHD), 39–41, with sixth nerve palsy, 85
40f, 42, 58, 59t with third nerve palsy, 76
optic nerve head, elevation of, 36–42 painless monocular vision loss, 16, 17,
optic nerve sheath fenestration (ONSF), 34 21, 22, 25
optic nerve sheath meningioma (ONSM), palinopsia, 54t
22f, 23, 24 pallid optic disc edema, 11, 12, 13f
optic neuritis, 5–10, 122, 123. See also panhypopituitarism, 46
idiopathic optic neuritis papilledema, 30, 31f, 34
causes of, 6 differential diagnosis of, 37
diagnostic tests for, 6–7, 7f, 8 pseudopapilledema compared with, 37,
retrobulbar, 6 38f, 38t
Optic Neuritis Treatment Trial, 8 TVL and, 58
optic neuropathy. See also compressive papillitis, 6
optic neuropathy; nonarteritic anterior paraneoplastic cerebellar degeneration,
ischemic optic neuropathy 115–16, 117
AAION, 11–15, 18 parieto-occipital hypometabolism, 55
hereditary, 25–28 Parkinson's disease, 142
ischemic Parks-Bielschowsky three-step
anterior, 11–20 test, 80–82, 81t
posterior, 12 partial third nerve palsy, 76
LHON, 25–28 PET. See positron emission
optic tract, lesion in, 49 tomography
optokinetic nystagmus, for nonorganic pharmacologic mydriasis, 162–65
vision loss, 69 differential diagnosis of, 163–64
oral steroids, 8 drugs causing, 163, 163t, 164
orbicularis oculi, 89, 142, 143 phenytoin, 100, 111, 112
orbital apex, superior orbital fissure, phonophobia, with migraine, 64
cavernous sinus, syndromes of, 169–74 phosphodiesterase type-5 inhibitors, 18
causes of, 170, 171 photophobia
cranial nerve palsies with, 170 with BEB, 142–43
mucormycosis causing, 170–73, 172f with migraine, 64
structures of, 170, 170t with tonic pupil, 158, 159, 161

INDEX 189
physiologic anisocoria, 147–51 with saccadic intrusions, 131–32, 134
algorithm for evaluation of, 149f refractive errors, in infantile nystagmus
differential diagnosis of, 148t, 150, 151 syndrome, 128
physiologic end-point nystagmus, 111, 113 relative afferent pupillary defect, 5, 6, 9,
pituitary apoplexy, 43, 44, 46, 47, 94t, 96 11, 16, 21, 22, 25, 49, 69, 122, 123,
pituitary macroadenoma, 44, 46f, 47 169, 170
positive visual phenomena, 63, 64, 65f, 65t rest test, for ocular myasthenia, 90–91, 90t, 92
positron emission tomography retinal folds, 22, 30
(PET), 55, 116 retinal migraine, 64, 67
posterior compressive optic neuropathy, 22 retrobulbar optic neuritis, 6
posterior ischemic optic neuropathy, 12
potassium channel blockers, 116 saccades
prednisone, 14, 19 horizontal, 131–33
presbyopia, 107, 108 vertical, 106, 107
primary visual cortex, 49 saccadic amplitudes, 132–34
prisms, 80, 83, 86 saccadic disorders, 131–34
progressive supranuclear palsy (PSP), saccadic hypermetria, 131–34
106–9, 132, 142 saccadic intrusions
neuro-ophthalmic findings in, 107, 108 and dysmetria, 131–34
reading difficulties with, 105–8 causes of, 132–33
propranolol, 66 square-wave jerks, 132, 133
proptosis, 22, 81, 178, 179 reading difficulties with, 131–32, 134
prosopagnosia, 53, 54t scotoma
Alzheimer's disease causing, 53 bean-shaped, 64, 66
cerebral achromatopsia with, 53 central, 5–6, 21–22, 25–26, 169–70
homonymous visual fields defects junctional, 44
with, 53 segmental palsy, 159, 160, 163–64
viral encephalitis causing, 53 sensitivity to light. See photophobia
visual memory impairment with, 53 sfEMG. See single-fiber electromyography
pseudopapilledema, 36–42 shunting, for cerebrospinal fluid, 34, 35
papilledema compared with, 37, 38f, 38t shunt malfunction, with dorsal midbrain
PSP. See progressive supranuclear palsy syndrome, 175–77
ptosis, 75, 76, 78, 88, 89, 90, 91, 91f, 92, simultanagnosia, 53, 54t
93, 95, 96, 137–40, 139f, 148, 152, 153, single-fiber electromyography (sfEMG),
154, 157, 169, 170, 176. See also eyelid 90t, 91, 92
upside-down, 154, 157 sixth nerve palsy, 84–87
pulsatile tinnitus, 30, 85 diagnostic testing for, 85–86, 87
pupil, tonic, 158–61. See also tonic pupil etiology of, 85, 86
Adie's, 159, 160, 161 microvascular ischemia causing, 85, 86
pyridostigmine, 92 MS causing, 85, 86
neoplasm causing, 85, 86
raised intracranial pressure (ICP), 29–35 pain with, 85
reading difficulties stroke causing, 85, 86
with progressive supranuclear palsy, 106–8 trauma causing, 85, 86

190 INDEX
skew deviation, 81–82 trauma causing, 76, 78
square-wave jerks, 132, 133 tremor with, 76
stereopsis, for nonorganic vision loss, 69 thyroid eye disease (TED), 23, 81, 178–82,
strabismus surgery, 83, 86, 92, 96, 100, 129 179f, 180t
stroke binocular diplopia with, 178, 179
APN with, 124 grading of, 180, 180t
brainstem, 94, 94t, 96, 103, 105, 133 thyroid function tests for, 180
causing sixth nerve palsy, 85, 86 vision loss with, 23, 179
superior oblique myokymia, 98–101 visual field testing for, 180
medical therapy for, 100 thyroid function tests, 180
neurovascular compression causing, 100 tilted optic discs, 39, 44
pathogenesis of, 99–100 tonic pupil, 158–61
superior oblique weakness, 79–83 cause of, 159–60, 160t
superior orbital fissure. See orbital apex, dilute pilocarpine testing for, 160, 161
superior orbital fissure, cavernous sinus, photophobia with, 158, 159, 161
syndromes of topiramate, 66
supranuclear lesion, 94, 107 torsional diplopia, 80
suprasellar meningioma, 44, 47 transient diplopia, 13, 102
surgery transient visual disturbance, migraine aura
of eyelid, 92, 96, 140, 143, 181 and, 64
for INS, 129 transient visual loss (TVL), 11, 13,
strabismus, 83, 86, 92, 96, 100, 129 57–62, 59t
systemic hypoperfusion, 58, 59t, 61 arterial occlusion
systemic hypotension, 58 primary, 58, 61
secondary, 58, 60, 61
tangent screen, 70 arterial stenosis, 58, 60, 61
TED. See thyroid eye disease carotid Doppler ultrasound for, 60
temporal headaches, 11, 12, 13 CTA for, 60
third nerve palsy, 75–78, 89, 139, 163 duration of, 58
aberrant regeneration with, 76 echocardiography for, 60–61
anisocoria with, 148, 148t, 149f, 159, electrocardiographic monitoring for, 61
160, 161, 163, 164 embolism causing, 58, 60, 61
ataxia with, 76 etiology of, 58, 59t, 60, 61
compression causing onset of
by aneurysm, 76, 77–78, 77f altitudinal, 58
by neoplasm, 76, 78 concentric, 58
contralateral hemiparesis with, 76 systemic hypoperfusion, 58, 59t, 61
etiology of, 76, 78 vascular risk factors with, 58
headaches with, 75 trauma
incomplete, 76 causing anisocoria, 148, 148t, 149f
microvascular ischemia causing, 76, 78 causing eyelid ptosis, 139
neurologic symptoms with, 76 causing fourth nerve palsy, 80, 81, 83
pain with, 76 causing sixth nerve palsy, 85, 86
partial, 76 causing third nerve palsy, 76, 78

INDEX 191
tremor, with third nerve palsy, 76 visual agnosia, 53
TVL. See transient visual loss visual aura, 64
twitching vision, 98, 99 visual field defects. See also homonymous
visual field defects
upbeat nystagmus, 118–21 bitemporal, 43, 44, 45f, 47
causes of, 119, 120, 121 central, 6, 22, 44
medications for, 120–21 chiasmal lesions with, 45f
upper-eyelid retraction, 106, 107, 175, 176, incongruent, 49
176f, 177, 178, 179, 180t, 181 pattern of, 51, 58
upper-eyelid skin crease, 137, 139, 139f, visual hallucinations, 54t
140 visual memory impairment, 53
upside-down ptosis, 154, 157 visual-spatial difficulties, 55
visual variant of Alzheimer's disease
vasospasm, 58, 59t, 61 (VVAD), 55, 55f, 56
vertebrobasilar ischemia, 64 vomiting
vertical oscillopsia, 114, 117, 118, 119 with migraine, 64
vertical saccades, limited or slowed, 106, with pituitary apoplexy, 43, 44
107 VOR. See vestibulo-ocular reflex
vestibulo-ocular reflex (VOR), 94, 96, 103 VVAD. See visual variant of Alzheimer's
viral encephalitis, 53 disease
vision loss. See also nonorganic vision loss
binocular, 13, 29, 58, 59t, 61 warfarin, anticoagulation with, 61
central, 22 Wernicke's encephalopathy, 118–21
monocular, 6, 12, 13, 58, 59t, 60f, Whipple's disease, 108
61, 69 white matter lesions, 7, 7f, 8
painless, 16, 17, 21, 22, 25
with TED, 23, 179 X-linked infantile nystagmus syndrome,
visual acuity, decreased, 6, 9, 103, 179 127, 129

192 INDEX

You might also like